Renal Uro

You might also like

Download as pdf or txt
Download as pdf or txt
You are on page 1of 120

04/09/2023, 11:44 mcqbank.co.uk/testhistory.php?

view=1487811

Welcome oasis.miles@gmail.com | Sign out

Home My Account Exam Info Testimonials About us

Test History
You scored: 70%
Total time taken to complete the test: 192 minutes 35 seconds.
Total number of questions answered: 68, out of: 68 possible.
Average time taken per question: 2 minutes 49 seconds.

Question Details:

Single Best Answer Question

Maximum Mark: 1
Mark Awarded: 1
Time taken to answer: 0 minutes 50 seconds.
Main Question

Which one of the following is the MOST appropriate management for a strongly suspected case of testicular torsion seen in A&E?
Answers

Your answer:
C. Arrange urgent scrotal exploration in theatre and warn patient &/or his parents of the possible need to remove the testicle if
non-viable.

Correct Answer:
Your answer is correct!

All Answers:
A. Watch and wait with a trial of analgesia and GP follow-up as most torsions spontaneously "unwind".
B. Arrange ultrasound scan to confirm the diagnosis.
C. Arrange urgent scrotal exploration in theatre and warn patient &/or his parents of the possible need to remove the testicle if non-viable.
D. Discharge with a 14 day course of ciprofloxacin as the testicle most likely is already necrosed.
E. Manually “unwind” the testicle in a counter-clockwise direction under entonox to reduce the necrosis whilst waiting for surgery.

Feedback

Torsion of the testis causes occlusion of testicular blood vessels.

Peak incidence around 13yrs of age.


The left side is more commonly affected than the right with 2% of cases being bilateral.
Risk factors
high insertion of the tunica vaginalis (producing a "bell clapper testis" with a horizontal lie).

undescended testes.
Presentation
History includes a sudden onset of severe unilateral scrotal pain & scrotal swelling.
50% of patients have a history of prior intermittent testicular pain that has resolved spontaneously.
There may be associated nausea & vomiting, abdominal pain, fever and urinary frequency.
Examination reveals a tender testicle frequently elevated in position with a horizontal lie when compared with
the other side.
The scrotum is oedematous and there is ipsilateral loss of the cremasteric reflex.
Pain is not relieved upon elevation of the scrotum (elevation may improve the pain in epididymitis -
Prehn's sign).

https://www.mcqbank.co.uk/testhistory.php?view=1487811 1/120
04/09/2023, 11:44 mcqbank.co.uk/testhistory.php?view=1487811
Management
Testicular torsion is a clinical diagnosis.
Imaging studies usually are not necessary.
If the history and examination suggest testicular torsion, the patient should go directly to theatre.
The patient &/or his parents must be warned of the possible need to remove the testis if non-viable.
Ultrasound may be ordered if there is doubt and shows absence of arterial flow.
Urinalysis is usually normal but WBC can be observed in 30% - therefore, do not rely on urinary WBC presence
to exclude the diagnosis.

Send us your feedback about this MCQ | Add question feedback to My Revision Notes | Rate this MCQ:

Single Best Answer Question

Maximum Mark: 1
Mark Awarded: 1
Time taken to answer: 0 minutes 53 seconds.
Main Question

A 7-year-old boy presents to A&E with bloody diarrhoea after eating a kebab. Urine dip shows blood and protein. Initial bloods
show a thrombocytopaenia. Which of the following is the MOST likely diagnosis?
Answers

Your answer:
C. Haemolytic uraemic syndrome

Correct Answer:
Your answer is correct!

All Answers:
A. Concurrent gastroenteritis and urinary tract infection
B. Nephrotic syndrome
C. Haemolytic uraemic syndrome
D. Disseminated intravascular coagulation
E. Crohn’s disease

Feedback

Haemolytic uraemic syndrome is the triad of:


1. Acute renal failure
2. Microangiopathic haemolytic anaemia
3. Thrombocytopenia

It is the most common cause of acute renal failure in children.


Aetiology
>90% of cases in children are secondary to infection.
E. coli O157:H7 causes haemorrhagic colitis but only about 3-15% of cases progress to HUS.
About 10% of HUS cases are atypical and are not caused by Shiga toxin-producing bacteria or streptococci.
Patients without evidence of underlying infection should be fully investigated, in particular looking for
complement gene mutations.
Presentation
The classical presenting feature is profuse diarrhoea that turns bloody 1 to 3 days later.
It is rare for the diarrhoea to have been bloody from the outset.
There is often fever, abdominal pain & vomiting:
Management
Is based on the prompt recognition of the syndrome, and clear differentiation from diarrhoea & vomiting with
dehydration.
Investigations
Stool sample should be sent for culture & phage typing of the E. coli.
FBC & blood film provide evidence of haemolysis, anaemia & thrombocytopenia.
Packed cell volume of <30%, erythrocyte destruction evident on peripheral blood film & platelet count
<150x109/L are typical.
U&E show the extent of renal involvement.
Treatment

https://www.mcqbank.co.uk/testhistory.php?view=1487811 2/120
04/09/2023, 11:44 mcqbank.co.uk/testhistory.php?view=1487811
Is largely supportive, but often needs to be undertaken in a tertiary centre since dialysis is often necessary to
manage the acute renal failure.

Send us your feedback about this MCQ | Add question feedback to My Revision Notes | Rate this MCQ:

Single Best Answer Question

Maximum Mark: 1
Mark Awarded: 1
Time taken to answer: 0 minutes 22 seconds.
Main Question

A 27-year-old, healthy, non-pregnant woman presents to her GP with a first presentation of an uncomplicated lower urinary tract
infection. According to national guidance, which SINGLE ONE of the following is the most appropriate FIRST-LINE treatment
option assuming no drug allergies and normal renal function?
Answers

Your answer:
B. Nitrofurantoin

Correct Answer:
Your answer is correct!

All Answers:
A. Amoxicllin
B. Nitrofurantoin
C. Azithromycin
D. Cefalexin
E. Co-amoxiclav

Feedback

For women who are not pregnant presenting with an acute lower urinary tract infection, the need for antibiotics should be
considered depending on severity of symptoms, risk of complications, and previous urine culture results and antibiotic use. If
prescribing an immediate antibiotic — treat according to sensitivities from recent urine culture (if available), otherwise treat
empirically taking account of local antimicrobial resistance patterns:

For first choice, consider prescribing:


Nitrofurantoin for 3 days (if eGFR ≥45) or
Trimethoprim for 3 days (if low risk of resistance).
For second choice (if there is no improvement in symptoms when first-choice antibiotic is taken for at least 48 hours or if
first-choice is unsuitable) consider prescribing:
Nitrofurantoin for 3 days (if eGFR ≥45 and not used as first-choice) or
Pivmecillinam (a penicillin) for 3 days or
Fosfomycin 3g as a single dose sachet.

[Reference: NICE Clinical Knowledge Summaries - Urinary tract infection (lower) – women]

Urinary tract infection (lower)

Background
Lower urinary tract infection (UTI) = infection of the bladder.
Cystitis is often used as a synonym for lower UTI (particularly for women), although technically it means 'bladder
inflammation' and there may be non-infectious causes such as radiation & chemical-induced cystitis.
Upper UTI is infection of the upper urinary tract i.e. ureters & kidneys (pyelonephritis).
UTI is very common in women & accounts ~2% of all GP consultations.
50% of women will be treated for a symptomatic UTI during their lifetime.
UTI is not common in men, but incidence is higher in elderly men (likely to have additional risk factors) or
with indwelling urinary catheters.
Usually caused by gut bacteria from GI tract.
Escherichia coli accounts for 80% & Staphylococcus saprophyticus 10% of UTIs. Proteus mirabilis is more common
in men or associated with renal tract abnormalities, particularly calculi.

Definitions

Significant bacteriuria = 105 colony-forming units per ml (CFU/mL).

https://www.mcqbank.co.uk/testhistory.php?view=1487811 3/120
04/09/2023, 11:44 mcqbank.co.uk/testhistory.php?view=1487811
Asymptomatic bacteriuria = significant bacteriuria without clinical infection.
NICE Antenatal Care guidelines state that asymptomatic bacteriuria should be screened for & treated in
pregnancy as associated with premature delivery & low birthweight.
Do not treat asymptomatic bacteriuria in patients with indwelling catheters.
Uncomplicated UTI = typical pathogens with normal urinary tract & kidney function, and no predisposing co-morbidities.
Complicated UTI = increased likelihood of complications such as persistent infection, treatment failure & recurrent
infection. Assoc with ≥1 risk factors:
Abnormal urinary tract (e.g. calculus, vesicoureteric reflux, neurogenic bladder, indwelling catheter, urinary
obstruction, recent instrumentation).
Virulent organism (e.g. Staphylococcus aureus).
Immunosuppression (e.g. poorly controlled diabetes).
Impaired renal function.
Recurrent UTI = Defined as ≥2 UTI in 6mths or ≥3 UTI in 12mths. Can be:
Relapse - same organism
Reinfection - different organism

Diagnosis
Typical features of UTI = dysuria, frequency, urgency, changes in urine appearance or
consistency, nocturia, suprapubic discomfort/tenderness.
Note typical features may be absent in the elderly with underlying cognitive impairment — can present
with delirium, lethargy, ↓ ability to carry out activities of daily living & anorexia.
Women
If uncomplicated UTI in <65y.o, urine dipstick can be used as a diagnostic aid — dipstick is unreliable in women
aged >65y & those who are catheterised.
If dipstick +ve for nitrite or leukocyte & RBC ⟹ UTI is likely.
Send urine for culture if previous antibiotic treatment has failed or possibility of antibiotic resistance.
If dipstick -ve for nitrite & +ve for leukocyte ⟹ UTI is possible.
Send urine for culture to confirm diagnosis.
If dipstick -ve for nitrite, leukocyte & RBC ⟹ UTI is unlikely.
No need to send urine culture — consider other diagnoses.
Send urine for culture if:
Pregnant.
>65y.
Persistent symptoms that do not resolve with antibiotics.
Recurrent UTI.
Urinary catheter in situ or recently catheterised.
Risk factors for resistance or complicated UTI such as GU tract abnormalities, renal impairment,
residence in a long-term care facility, hospitalisation for >7d in last 6m, recent travel to a country with ↑
resistance or previous resistant UTI.
Atypical symptoms.
Visible or non-visible haematuria.
Men
Confirm diagnosis with urine culture before starting empirical drug treatment.
If catheterised - only send urine sample if features of systemic infection.
Do not use urine dipstick or microscopy to diagnose UTI in men:
Who are not catheterized — dipsticks are poor at ruling out infection.
But, may be helpful in some situations, e.g. UTI is unlikely if mild or non-specific symptoms AND -
ve urine dipstick test (i.e. both nitrite & leukocytes -ve).
Note, presence of these markers does not rule in UTI, although +ve nitrite makes UTI more
likely (PPV 96%).
With an indwelling catheter — working diagnosis should be based on clinical judgement.
Aged >65y — dipsticks become more unreliable with increasing age >65y.

Differential diagnosis
Pyelonephritis - urinary symptoms associated with fever &/or loin pain.
Drug-induced cystitis - cyclophosphamide, allopurinol, danazol, or tiaprofenic acid.
Sexually transmitted Infections e.g. chlamydia trachomatis (pyuria without bacteriuria).
Dermatological conditions e.g. psoriasis, dermatitis, lichen sclerosis or lichen planus.
Spondyloarthropathies e.g. reactive arthritis or Bechet’s syndrome.
Malignancy.
Ovarian cancer may present with persistent urinary urgency &/or frequency.
Bladder, or renal cancer may present with haematuria (visible or non-visible).
In women – consider
Urethral syndrome (aka painful bladder syndrome, interstitial cystitis & trigonitis = symptoms of cystitis in
absence of UTI - symptoms relieved by voiding & aggravated by drinking alcohol or caffeinated drinks).

https://www.mcqbank.co.uk/testhistory.php?view=1487811 4/120
04/09/2023, 11:44 mcqbank.co.uk/testhistory.php?view=1487811
Atrophic vaginitis/urethritis (menopausal woman with vaginal discharge or itch, & pain during sexual
intercourse).
In men – consider
Acute prostatitis (fever, irritative urinary symptoms, perineal/suprapubic pain, pain on ejaculation or with bowel
movements, & exquisitely tender prostate on PR).
Epididymitis (scrotal pain & epididymis is oedematous & tender).

Empirical Treatment
Women
If mild symptoms with normal immunity, renal function & renal tract, treatment can be delayed if she wishes
to see if symptoms resolve, especially if -ve urine dip (nitrites & leucocytes) indicating low probability of UTI.
For all other women especially if pregnant prescribe treatment without delay.
1st Choice
Nitrofurantoin for 3d.
If eGFR ≥45.
c/i in G6PD deficiency & acute porphyria.
Trimethoprim for 3d.
BNF advises caution if predisposed to folate deficiency.
Renal impairment:
If eGFR 15–30 - use half normal dose after 3d.
If eGFR <15 - use half normal dose.
c/i in blood dyscrasias.
2nd Choice (if no improvement in symptoms when 1st-choice antibiotic is taken for at least 48h or if first-
choice is unsuitable).
Nitrofurantoin for 3d (if not used as 1st choice).
Pivmecillinam for 3d.
Fosfomycin single dose sachet.
Pregnant
Nitrofurantoin (avoid at term due to risk of neonatal haemolysis) for 7d.
2nd choice
Amoxicillin (only if culture results available & susceptible) for 7d.
Cefalexin for 7d.
BNF states that manufacturer advises avoiding trimethoprim during pregnancy. There is a
teratogenic risk in first trimester (folate antagonist).
Men
Start empirical antibiotic drug treatment with trimethoprim or nitrofurantoin for 7d.

Duration
3d course for women with:
Uncomplicated UTI.
UTI with haematuria.
Recurrent UTI.
5–10d course for women who have:
Abnormal urinary tract.
Immunosuppression.
Impaired renal function.
7d course if:
Pregnant.
Catheterised.
Change catheter before starting antibiotics if catheter has been insitu for >7d.
Consider withholding antibiotics until culture results available to guide treatment if mild symptoms with normal
immunity & renal function.
Male.

References
NICE Clinical Knowledge Summaries - Urinary tract infection (lower) – women
NICE Clinical Knowledge Summaries - Urinary tract infection (lower) – men
British National Formulary

Send us your feedback about this MCQ | Add question feedback to My Revision Notes | Rate this MCQ:

https://www.mcqbank.co.uk/testhistory.php?view=1487811 5/120
04/09/2023, 11:44 mcqbank.co.uk/testhistory.php?view=1487811

Single Best Answer Question

Maximum Mark: 1
Mark Awarded: 1
Time taken to answer: 3 minutes 11 seconds.
Main Question

Which SINGLE ONE of the following is the commonest cause of sterile pyuria in the UK?
Answers

Your answer:
C. Partially or recently treated UTI

Correct Answer:
Your answer is correct!

All Answers:
A. Schistosomiasis
B. Tuberculosis
C. Partially or recently treated UTI
D. Renal or bladder tumour
E. Chlamydia

Feedback

Despite the name, sterile pyuria, in many cases, is caused by an infection of some sort – usually a partially or recently treated
UTI. In younger sexually active people, chlamydial urethritis is a common cause. In patients over the age of 45, renal or
bladder tumours are important considerations.

Genito-urinary TB and schistosomiasis are both rare in the UK but occur disproportionately frequently in exams. Genito-
urinary TB is often asymptomatic, but may present with urinary symptoms (especially frequency and urgency), loin pain (from
upper tract obstruction), haematuria and non-specific symptoms such as fever, weight loss, night sweats and malaise. The
diagnosis should be considered in patients with sterile pyuria who have previous contact with active TB, recent arrival from a
high-risk country, the immuno-compromised or health care workers in settings with high TB prevalence. Schistosomiasis
haematobium infection (a tropical disease also known as bilharzia) may cause sterile pyuria, as well as cystitis like symptoms or
haematuria and a travel history should be taken for persistent unexplained pyuria. High risk areas are Africa (especially Lake
Malawi) and the Mediterranean part of the Middle East, particularly in people who have been swimming in open water. Urinary
symptoms may develop many years after the original infection.

Sterile Pyuria

Background
Sterile pyuria can be broadly defined as the presence of leucocytes in urine in the absence of demonstrable urinary
tract infection.
The British Journal of Family Medicine (BJFM) defines it as urine dipstick +ve for leucocytes without dipstick
haematuria or nitrites, & subsequent MSU showing no bacterial growth.
BJFM advises to ignore a trace of leucocytes & to treat “+” or greater as a positive result.
An MSU is sent to allow urine culture to detect organisms, & not to confirm presence of leucocytes, which may
degrade while in transit to the laboratory, thus creating a false -ve result.
If pyuria is seen on microscopy in the absence of prior dipstick analysis, it is unclear on the exact threshold for
clinical significance, although a value of >5-10 WBC per μl of urine may be sensible.
Important to ensure a urine specimen is taken clean-catch & mid-stream to reduce risk of contamination.
Urine from catheters or urostomies will almost always contain white cells and cannot therefore be
interpreted, except in the context of symptoms, e.g. of suspected infection.
Sterile pyuria is not an uncommon finding in primary care.
~10% of patients presenting to their GP with lower urinary symptoms, & who are suspected of having UTI are found
to have sterile pyuria.

Causes
Sterile pyuria may be classified as infectious or non-infectious.

https://www.mcqbank.co.uk/testhistory.php?view=1487811 6/120
04/09/2023, 11:44 mcqbank.co.uk/testhistory.php?view=1487811

Infectious causes
The very name, sterile pyuria, is potentially misleading, as the commonest cause is infection of some sort.
May be a partially treated UTI (even one dose of antibiotic before urine collection), a recently treated UTI (pyuria
often remains for 1-2w after clearing infection), a UTI with a fastidious (complex or particular nutritional requirements) or
slow growing atypical organisms that fail to grow during standard laboratory culture.
Patients with typical symptoms of UTI & sterile pyuria should be treated empirically with antibiotics & urine retested
after treatment to ensure resolution of the pyuria (i.e. treating for probable UTI with a fastidious organism).
Other, infective causes are also possible.
Urethritis - particularly chlamydia in the asymptomatic sexually active person.
Therefore, a sexual history should always be sought in young patients presenting with lower urinary tract
symptoms.
In the older population, prostatitis, cystouretheritis, & balanitis may present as sterile pyuria but symptoms
should make these diagnoses apparent.
Extrinsic irritation of the ureters or bladder by appendicitis or diverticulitis may cause pyuria.
The clinician should always be vigilant to enquire about recent foreign travel. On a global scale, infection with tuberculosis
(TB) or schistosomiasis are common causes of pyuria, but these are rare in a UK population without other risk factors.
Consider TB in chronic sterile pyuria – although a rare manifestation of TB, its consequences may be disastrous for
the patient if not treated early.
Suspect in patients coming from endemic regions, immunocompromised, & those presenting with
unintentional weight loss.

Non-infectious causes
Renal disease
Causes
Papillary necrosis - where the tips of the renal medulla undergo ischaemic necrosis, occurs most commonly
as part of diabetic nephropathy, in sickle cell disease or in patients with analgesic nephropathy, due to
long term use of analgesics such as co-codamol.
Tubulo-interstitial diseases - interstitial nephritis, lupus nephritis, sarcoidosis.
Structural causes - e.g. polycystic kidney disease.
This should be considered for all patients.
All patients with unexplained sterile pyuria should have an eGFR, urinary albumin: creatinine ratio & BP.
Referral should be considered if sterile pyuria is associated with significant proteinuria (ACR>30mg/mmol) or
deteriorating renal function.
Renal or bladder tumours although exact risk associated with isolated asymptomatic sterile pyuria, while not stated in
the medical literature, is likely to be extremely low.
Interstitial cystitis, or painful bladder syndrome - but will present with typical symptoms – suprapubic pain
(especially with a full bladder), dysuria & urinary frequency. This condition should be suspected on clinical grounds
& requires further urological investigation.
↓ oestrogenisation of the vulva, vagina & bladder in older women can lead to a degree of inflammation, making sterile
pyuria a relatively common finding.
Treatment of symptomatic atrophic vaginitis with topical oestrogen can lead to resolution of sterile pyuria but can
this be justified in asymptomatic women purely to see clearance of white cells from the urine?
Sterile pyuria is a common finding in pregnancy – usually due to contamination by physiological vaginal discharge. It is
vital, however, to confidently exclude UTI, as asymptomatic bacteriuria is such an important preventable
cause of pre-term labour. If pyuria is found, it is worth repeating the urine specimen after cleaning the vulva &
rigorously ensuring a true clean-catch specimen.
Radiotherapy involving pelvis & urinary tract.
Instrumentation of the urinary tract (cystourethroscopy or nephroscopy), or indwelling catheters & stents.

https://www.mcqbank.co.uk/testhistory.php?view=1487811 7/120
04/09/2023, 11:44 mcqbank.co.uk/testhistory.php?view=1487811
Drugs – Olsalazine, nitrofurantoin, penicillin-based antibiotics, NSAIDs, aspirin, proton pump inhibitors &
diuretics have also been involved in acute drug reactions, causing tubulointerstitial nephritis with sterile pyuria.

Management
Investigations
BJFM recommends symptomatic patients with sterile pyuria warrant investigation.
But, asymptomatic patients with sterile pyuria should have urine retested after a short interval such as 2–4w. 2 out
of 3 +ve results warrant further investigation.
Basic primary care investigations:
MSU for MC&S
Chlamydia/gonorrhoea test
eGFR, ACR, BP
Renal U/S
Specialist referral
All symptomatic patients with unexplained sterile pyuria should be referred to secondary care (typically
urology, gynaecology or nephrology) for assessment
Red flags suggestive of malignancy = sterile pyuria in association with visible haematuria, or with persistent new
onset storage symptoms (urgency/frequency/nocturia), or with bladder pain.
The question of whether to refer all patients with persistent unexplained asymptomatic sterile pyuria is more difficult
to answer categorically.
Many urologists will feel all these patients should be investigated, while many GPs, often more comfortable
with “dealing with uncertainty” will take a more pragmatic approach if basic investigations are reassuring.
There is no evidence to tell us which of these is the correct approach, and thus referral cannot be considered
mandatory. However, if in doubt, it is not unreasonable to refer patients for a urological opinion, if only for
reassurance of both patient & doctor.

References

Rees et al. Assessment of sterile pyuria in primary care. British Journal of Family Medicine (2015); 3 (3): 34-37

Glen et al. Sterile pyuria: a practical management guide. British Journal of General Practice (2016); 66 (644): e225-e227

Send us your feedback about this MCQ | Add question feedback to My Revision Notes | Rate this MCQ:

Extended Matching Question

Maximum Mark: 3
Mark Awarded: 3
Time taken to answer: 1 minutes 38 seconds.
Main Question

For each patient below with renal disease, choose the most likely diagnosis.
Select ONE option only from the list.
Each option may be selected once, more than once or not at all.
Answers

Your Answer:
F. Fanconi syndrome
Correct Answer:
Your answer is correct!

All answers:
1. A 12-month-old boy with a history of cystinosis presents with failure to
A. Alport's syndrome
thrive. On examination, he has low muscle tone, frontal bossing and leg
B. Nephrocalcinosis
bowing. His urine shows aminoaciduria, glycosuria and phosphaturia.
C. Amyloidosis
D. Adult Polycystic kidney disease
E. Renal calculi
F. Fanconi syndrome
G. Von Hippel-Lindau disease
H. Tuberous sclerosis

2. A 30-year-old man presents with hypertension and gross haematuria. Your Answer:
Abdominal examination reveals enlarged palpable kidneys. D. Adult Polycystic kidney disease
Correct Answer:
Your answer is correct!

https://www.mcqbank.co.uk/testhistory.php?view=1487811 8/120
04/09/2023, 11:44 mcqbank.co.uk/testhistory.php?view=1487811
All answers:
A. Alport's syndrome
B. Nephrocalcinosis
C. Amyloidosis
D. Adult Polycystic kidney disease
E. Renal calculi
F. Fanconi syndrome
G. Von Hippel-Lindau disease
H. Tuberous sclerosis

Your Answer:
B. Nephrocalcinosis
Correct Answer:
Your answer is correct!

All answers:
3. A 60-year-old man with multiple myeloma is noted to have patchy A. Alport's syndrome
calcification of the kidneys on an abdominal x-ray. B. Nephrocalcinosis
C. Amyloidosis
D. Adult Polycystic kidney disease
E. Renal calculi
F. Fanconi syndrome
G. Von Hippel-Lindau disease
H. Tuberous sclerosis

Feedback

Q1 - F

Fanconi syndrome is a generalised inherited or acquired disturbance of renal tubular transport.


Leads to aminoaciduria, glycosuria, phosphaturia, renal tubular acidosis type 2 (proximal), & hypophosphataemic
rickets (children) or osteomalacia (adults).
Aetiology
Inherited
Primary
Autosomal dominant – chromosome 15
Autosomal recessive
X-linked
Secondary
Cystinosis is one of the most common causes of Fanconi syndrome in children
Acquired
Intrinsic renal disease
acute tubular necrosis, interstitial nephritis, hypokalaemic nephropathy, myeloma, amyloidosis
Drugs
cisplatin, ifosfamide, gentamicin, valproate. heavy metals & glue sniffing
Presentation
Polyuria, polydipsia, bouts of dehydration (sometimes associated with fever), bone deformities (rickets in
children or osteomalacia in adults.
Results from excessive urinary losses of calcium & phosphate and of a defect in the hydroxylation of 25-
hydroxyvitamin D3 into 1,25-dihydroxyvitamin D3, and impaired growth.
Management
Mainly consists of the replacement of substances lost in the urine and specific treatment for the underlying cause.

Q2 - D

Autosomal dominant polycystic kidney disease (ADPKD) is the most frequent genetic cause of renal failure in adults.
Approximately 90% have an abnormality on chromosome 16.
ADPKD is a multisystemic and progressive disorder with formation of cysts in the kidney and other organs (e.g. liver,
pancreas, spleen)
Hepatic cysts may occur in 75% of patients >60y.
They appear to be affected by female steroid hormones, and, although they may occur in both sexes, massive
cysts occur almost exclusively in women.
Non-cystic manifestations include an increased incidence of mitral valve prolapse, intracranial berry aneurysms (in up
to 40% of cases) and an increased frequency of colonic diverticula & renal cell carcinoma.
Presentation
Clinical features usually begin in the 3rd - 4th decade of life, but cysts may be detectable in childhood.
Patients usually present with
haematuria - from cyst rupture

https://www.mcqbank.co.uk/testhistory.php?view=1487811 9/120
04/09/2023, 11:44 mcqbank.co.uk/testhistory.php?view=1487811
flank pain - from cyst enlargement
hypertension
stroke - from subarachnoid haemorrhage from intracranial aneurysms
renal failure
palpable masses in the flanks
increased incidence of UTIs
Investigations
Urinalysis - check for infection, protein (microalbuminuria occurs in about a third but heavy proteinuria is rare)
and haematuria.
Bloods - FBC (polycystic kidneys can produce excess erythropoietin and hence raise Hb), U&E
Ultrasound can detect renal cysts from 1-1.5 cm in diameter.
Sensitivity is 100% over the age of 20 but false negatives can occur below this age.
It is also possible to scan other organs like the liver or pancreas for cysts.
CT is more sensitive in that it can detect cysts down to 0.5 cm diameter but the dose of radiation is quite high.
MRI is an equally sensitive alternative.
Gene testing can be done by linkage or sequence analysis:
Management
Diagnosis of ADPKD has implications both for the patient & their family.
Treatment is currently supportive only.
No treatments have, as yet, been shown to halt or even slow disease progression.

Q3 - B

Nephrocalcinosis is parenchymal calcium deposition in the kidney usually a consequence of hypercalcaemia.


Pathology
There may be intracellular renal damage caused by calcium overload.
Initially, deposits occur in the mitochondria of renal cells, but later whole nephrons become calcified.
The result is patchy atrophy & calcification of the kidney.
Causes
Malignancy (commonest cause)
Multiple myeloma
Hyperparathyroidism
Sarcoidosis
Vitamin D intoxication
Presentation
The underlying aetiology primarily determines the presentation of nephrocalcinosis, although in most cases it is
asymptomatic and is identified as an abnormality on imaging of the renal tract.
May also present with
effects of hypercalcaemia (e.g. polyuria & polydipsia, nausea & vomiting).
features of acute or chronic renal failure.
In medullary nephrocalcinosis, calcium nodules commonly rupture into the calyceal system to form urinary
stones and cause renal colic, haematuria, urinary tract infections or the passage of urinary stones.
Investigations
Blood tests reveal hyercalcaemia & increased creatinine.
Urine tests reveal haematuria, proteinuria, pyuria, hypercalciuria
Abdominal X-ray may reveal renal calculi or nephrocalcinosis.

https://www.mcqbank.co.uk/testhistory.php?view=1487811 10/120
04/09/2023, 11:44 mcqbank.co.uk/testhistory.php?view=1487811

Treatment
Treatment of macroscopic nephrocalcinosis (calcium deposition that is visible without magnification) may include
thiazide diuretics & dietary salt restriction, potassium & magnesium supplementation, and citrate supplementation in
idiopathic hypercalciuria (of unknown cause) & in distal renal tubular acidosis.

Send us your feedback about this MCQ | Add question feedback to My Revision Notes | Rate this MCQ:

Single Best Answer Question

Maximum Mark: 1
Mark Awarded: 1
Time taken to answer: 0 minutes 36 seconds.
Main Question

A 60-year-old man presents with painless, visible haematuria. Rectal examination reveals a mildly enlarged, smooth prostate.
PSA is less than 4.0ng/ml. Which one of the following is the MOST likely diagnosis?
Answers

Your answer:
D. Bladder carcinoma

Correct Answer:
Your answer is correct!

All Answers:
A. Prostate carcinoma
B. Prostatitis
C. Urinary tract infection
D. Bladder carcinoma
E. Renal cell carcinoma

Feedback

The most common type of bladder cancer in the UK is urothelial carcinoma, formerly known as transitional cell
carcinoma (TCC).

The urothelium in the entire urinary tract may be involved, including the renal pelvis, ureter, bladder, & urethra.
Risk factors
1. About half of bladder cancers are caused by smoking.
2. Other risk factors include industrial exposure to aromatic amines in dyes, paints, solvents, leather dust, inks,
combustion products, rubber & textiles.
3. Environmental pollution, e.g. arsenic-contaminated wells, has been identified as a factor.
4. There may be a genetic predisposition.
5. Radiation to the pelvis and cyclophosphamide are risks.

https://www.mcqbank.co.uk/testhistory.php?view=1487811 11/120
04/09/2023, 11:44 mcqbank.co.uk/testhistory.php?view=1487811
6. There is a tenuous link between chronic infection and bladder cancer, e.g. HIV & herpes simplex
7. Coffee may increase the risk of bladder cancer by 20% but some studies have suggested otherwise.
8. Suggestions of risk from artificial sweeteners have not been substantiated & there is no increased risk in those who
dye their hair.
Presentation
Usually affects males in their 60’s.
90% present with painless visible haematuria, which is the classic presentation.
Consider all patients with visible haematuria to have bladder cancer until proven otherwise.
30% present irritative bladder symptoms such as dysuria, urgency, or frequency of urination.
Management
Diagnosis of bladder cancer is transurethral cystoscopy & urine cytology.
Urine cytology can be obtained in voided urine or at the time of the cystoscopy ("bladder washout").
Treatment varies depending on whether or not the tumour has invaded the muscle.
May involve intravesical immuno (BCG) or chemotherapy, transurethral resection of bladder tumour
(TURBT), radical cystectomy, external beam radiotherapy or adjuvant & neoadjuvant chemotherapy.

Send us your feedback about this MCQ | Add question feedback to My Revision Notes | Rate this MCQ:

Single Best Answer Question

Maximum Mark: 1
Mark Awarded: 0
Time taken to answer: 0 minutes 17 seconds.
Main Question

Which one of the following does NOT discolour urine?


Answers

Your answer:
D. Nitrofurantoin

Correct Answer:
E. Trimethoprim

All Answers:
A. Rifampicin
B. Beetroot
C. Myoglobinuria
D. Nitrofurantoin
E. Trimethoprim

Feedback

Colour Pathological causes Food and drug causes


bile pigments levodopa
Brown
myoglobin metronidazole
nitrofurantoin
some antimalarial
agents
fava beans

bile pigments Cascara


Brownish-black
melanin levodopa
methaemoglobin methyldopa
senna

pseudomonal UTI amitriptyline


Green or blue
biliverdin indigo carmine
IV cimetidine
IV promethazine
methylthioninium
chloride
triamterene

bile pigments phenothiazines


Orange

https://www.mcqbank.co.uk/testhistory.php?view=1487811 12/120
04/09/2023, 11:44 mcqbank.co.uk/testhistory.php?view=1487811
phenazopyridine

haematuria beets
Red
haemoglobinuria blackberries
myoglobinuria rhubarb
porphyria phenolphthalein
rifampicin

concentrated urine carrots


Yellow (orange to gold in cascara
dehydration)

Send us your feedback about this MCQ | Add question feedback to My Revision Notes | Rate this MCQ:

Single Best Answer Question

Maximum Mark: 1
Mark Awarded: 1
Time taken to answer: 1 minutes 38 seconds.
Main Question

Which one of the following patients is NOT eligible to receive branded Viagra® on a NHS prescription?
Answers

Your answer:
E. 50-year-old who had a myocardial infarction in the last 6mths.

Correct Answer:
Your answer is correct!

All Answers:
A. 35-year-old with diabetes mellitus.
B. 40-year-old with multiple sclerosis.
C. 70-year-old with prostate cancer.
D. 75-year-old with Parkinson’s disease.
E. 50-year-old who had a myocardial infarction in the last 6mths.

Feedback

Previous MI is not an indication for prescribing branded Viagra® on the NHS and also recent MI is a contraindication.

Erectile dysfunction (ED) is the inability to attain and maintain an erection sufficient for satisfactory sexual
performance.

ED shares risk factors with cardiovascular disease.


History gathering
The following features suggest psychogenic causes:
Sudden onset
Early collapse of erection
Self-stimulated or waking erections
Premature ejaculation or inability to ejaculate
Problems or changes in relationship
Major life events
Psychological problems
The following features suggest organic causes:
Gradual onset
Normal ejaculation
Normal libido (except hypogonadal men)
Risk factor in medical history (cardiovascular, endocrine or neurological)
Operations, radiotherapy, or trauma to pelvis or scrotum
Current drug recognised as associated with erectile dysfunction
Smoking, high alcohol consumption, use of recreational or body-building drugs
Management

https://www.mcqbank.co.uk/testhistory.php?view=1487811 13/120
04/09/2023, 11:44 mcqbank.co.uk/testhistory.php?view=1487811
First-line therapy for erectile dysfunction
Oral agents
Phosphodiesterase inhibitors (sildenafil, tadalafil, & vardenafil) improve the relaxation of smooth
muscle.
Efficacy of the drug is dependent on release of nitric oxide from the nerve terminals of the
cavernosal nerve.
Contra-indications
Patients receiving nitrates
Previous history of non-arteritic anterior ischaemic optic neuropathy
Hypotension
avoid if systolic blood pressure <90 mmHg
Recent stroke or myocardial infarction
Unstable angina
Patients in whom vasodilation or sexual activity are inadvisable
Side-effects
GI s/e
Dyspepsia
Nausea & vomiting
Headaches (including migraine)
Visual disturbances
blue discolouration
non-arteritic anterior ischaemic optic neuropathy has been reported—stop drug if
sudden visual impairment occurs)
Nasal congestion
Flushing
Vacuum devices
External cylinder fitted over the penis to allow air to be pumped out, resulting in engorgement of penis
with blood.
They work best when there is a motivated, interested & understanding partner.
They may be the treatment of choice in well-informed older patients and in those with
comorbidities precluding use of drugs or invasive methods.
Adverse events include pain, petechiae, bruising & numbness.
Second-line therapy for erectile dysfunction
Intraurethral alprostadil (prostaglandin E1) is inserted as a pellet into the urethral meatus & produces an
erection after about 15 minutes.
Intracavernosal alprostadil is given into the corpora cavernosa to produce an erection lasting <1 hour.
Penile pain is usually mild but a significant number of men stop using this method because of penile pain.
Third-line therapy for erectile dysfunction
Penile prosthesis
Semi-rigid, malleable or inflatable devices surgically inserted to produce an erect state.
Prostheses should be considered in patients whose impotence has an organic cause and who are
unwilling to consider, fail to respond to, or are unable to continue with medical treatment or external
devices.

Private or NHS prescription?


In Aug 2014, generic sildenafil (& apomorphine) was taken off the SLS list following a significant reduction in the price of
sildenafil after the expiry of patent protection for Viagra®, however restrictions continue to apply to branded Viagra®.
Other phosphodiesterase inhibitors (e.g. tadalafil, avanafil) & alprostadil can still only be prescribed by GPs on the NHS for
men who:
1. Have diabetes, MS, Parkinson's disease, poliomyelitis, prostate cancer, severe pelvic injury, single-gene
neurological disease, spina bifida, or spinal cord injury.
2. Are receiving dialysis for renal failure.
3. Have had radical pelvic surgery, prostatectomy, or a kidney transplant.
4. Were receiving Caverject®, Erecnos®, MUSE®, Viagra®, or Viridal® for erectile dysfunction, at the expense of the
NHS, on 14 September 1998.
5. Treatment should also be available from specialist services (under local agreement) when the condition is
causing severe distress:
Significant disruption to normal social and occupational activities.
A marked effect on mood, behaviour, social and environmental awareness.
A marked effect on interpersonal relationships.
If the patient does not fit these criteria, a private prescription can be issued.

Send us your feedback about this MCQ | Add question feedback to My Revision Notes | Rate this MCQ:

https://www.mcqbank.co.uk/testhistory.php?view=1487811 14/120
04/09/2023, 11:44 mcqbank.co.uk/testhistory.php?view=1487811

Single Best Answer Question

Maximum Mark: 1
Mark Awarded: 1
Time taken to answer: 1 minutes 40 seconds.
Main Question

A 36-year-old man presents to his GP with subfertility. He and his wife have been trying to conceive for the last 2 years without
success. His wife’s investigations in primary care have all been normal. His semen analysis reveals oligospermia. He does have an
extensive medical history - Crohn’s disease, gout, palpitations, depression, and hypertension. His regular medications include
sulfasalazine, allopurinol, bisoprolol, citalopram and ramipril. Which one of his drugs is the most likely cause of his oligospermia?
Answers

Your answer:
C. Sulfasalazine

Correct Answer:
Your answer is correct!

All Answers:
A. Ramipril
B. Bisoprolol
C. Sulfasalazine
D. Allopurinol
E. Citalopram

Feedback

Of the drugs listed, sulfasalazine is the most likely cause of this man’s infertility. Sulfasalazine (both oral and rectal treatments)
can reduce sperm counts but the effects are reversible on withdrawal of therapy or by switching to mesalazine. This is an
important consideration as inflammatory bowel disease predominantly affects young adults of reproductive age.
[Reference - NICE Clinical Knowledge Summaries – Infertility]

Infertility

Background
NICE Clinical Knowledge Summaries defines Infertility as the period of time people have been trying to conceive
without success after which formal investigation is justified and possible treatment implemented.
Infertility affects 1 in 7 heterosexual couples in the UK.
The main causes of infertility in the UK are:
Unexplained infertility (25%)
Ovulatory disorders (25%)
Tubal damage (20%)
Factors in the male causing infertility (30%)
Uterine or peritoneal disorders (10%)

General advice for couples trying to conceive [Ref 1]


Preparation for pregnancy.
Advise women to take folic acid, check rubella status, and offer cervical screening in accordance with the national
cervical screening programme.
Regular sexual intercourse (2-3x weekly) THROUGHOUT the woman's cycle.
This should ensure that intercourse falls within the fertile period.
With regular intercourse:
84% will conceive within 1 year.
92% will conceive within 2 years.
Do not recommend the use of temperature charts or luteinizing hormone detection methods.
Stress
Stress reduces libido and frequency of intercourse.
Fertility problems and its investigation and treatment can also cause psychological stress.
Offer counselling before, during, and after investigation and treatment irrespective of the outcome.
Smoking reduces fertility in both men and women.
Offer referral to a smoking cessation programme.
Weight

Encourage weight loss if BMI >29 kg/m2.


Advise women with BMI <19 kg/m2 and either amenorrhoea or irregular menstruation that gaining weight is likely to
increase chances of conception.

https://www.mcqbank.co.uk/testhistory.php?view=1487811 15/120
04/09/2023, 11:44 mcqbank.co.uk/testhistory.php?view=1487811
Alcohol
DOH recommends women consume no alcohol at all whilst trying to conceive.
DOH advises men that excessive alcohol consumption can be detrimental to semen quality.
Other risk factors
No evidence that caffeinated beverages affect fertility.
Elevated scrotal temperature is associated with reduced semen quality, but it is uncertain whether wearing loose-
fitting underwear improves semen quality.
Effectiveness of complementary therapies has not been properly evaluated, and these therapies are not
recommended.
Illicit drugs may affect fertility.
Offer referral to a specialist drugs and alcohol service.

Causes of infertility in women:


The most common causes of infertility in women are ovulation failure (amenorrhoea or oligomenorrhoea) & tubal
damage secondary to infection.
Drugs
NSAIDs
Some evidence these may reversibly inhibit ovulation.
Cyclo-oxygenase inhibitors
Can have a negative local effect on ovulation.
Spironolactone
Can cause infertility and menstrual irregularities.
Normal menstruation resumes within 2 months of therapy withdrawal.
Cytotoxic drugs
Can induce ovarian failure, which may be permanent.
Neuroleptic drugs
May cause amenorrhoea and infertility.
Recreational drugs, e.g. marijuana and cocaine
Have been associated with impaired ovulatory and tubal function.

Causes of infertility in men:


In 44% of men with infertility, no underlying cause is found other than idiopathic oligo-astheno-
teratozoospermia (OAT) syndrome.
Physical findings and endocrine investigations (usually done in secondary care) are normal.
Semen analysis shows:
Decreased number of spermatozoa (oligozoospermia).
Decreased sperm motility (asthenozoospermia).
Many abnormal sperm on morphological examination (teratozoospermia).
Primary spermatogenic failure
= any spermatogenic abnormality caused by a condition other than hypothalamic pituitary disease.
Obstructive azoospermia
= absence of both spermatozoa and spermatogenic cells in semen and post-ejaculate urine due to bilateral
obstruction of the seminal ducts.
Hypogonadism
= deficient androgen secretion secondary to hypothalamic or pituitary causes.
Drugs
Sulfasalazine
Causes oligospermia.
Oral and rectal treatments have the same effects,
Reversible on withdrawal of therapy or by switching to mesalazine.
Androgens & anabolic steroids
Causes reduction in testicular volume and azoospermia/oligospermia.
Cytotoxic drugs
Can induce permanent azoospermia.
Recreational drugs, e.g. cocaine and marijuana
Can adversely affect the quality of semen.
Herbal remedies, e.g. root extracts of Tripterygium wilfordii
May cause reduction in testicular volume and azoospermia/oligospermia.

When to do initial investigations?


Start investigations in couples who have not conceived after 1 year of regular unprotected sexual intercourse.
But, offer investigations earlier than 1 year if have factors (see below) that make them less likely to conceive.

Initial investigations in a man


In the male, semen analysis is the only necessary initial investigation:

https://www.mcqbank.co.uk/testhistory.php?view=1487811 16/120
04/09/2023, 11:44 mcqbank.co.uk/testhistory.php?view=1487811
Specimen is produced by masturbation (not into a condom, as they contain spermatocides) and after three days of
abstinence from sexual activity.
The specimen should be kept warm and sent to the laboratory for examination, ideally within an hour from
production.
Normal results based on WHO criteria [Ref2]:
Parameter Lower reference limit
Semen volume ≥1.5ml
Total sperm
≥39 million per ejaculate
number
Sperm
≥15 million per ml
concentration
Progressive ≥40% motile or ≥32% with progressive
motility motility
Sperm morphology ≥4% of normal forms
Vitality ≥58% live spermatozoa
≥7.2
pH

If the result of the first semen sample is normal, there is no need to do a repeat confirmatory test.
If the result of the first semen sample is abnormal, order a repeat test:
Testing should ideally be repeated 3 months after the initial test, to allow time for the cycle of spermatozoa to be
completed.
It may be appropriate to test earlier (2-4wks) if:
Azoospermia or severe oligospermia (<15 million sperm/ml).
Patient preference (e.g. anxious and cannot wait).
If the repeat test result is normal, the semen should be regarded as normal and no further testing is required.
Men who have two abnormal semen examination results should be referred for further assessment.
Screen for chlamydia - first-void urine sample is reported to be as good as a urethral swab and causes less
discomfort in a male.

Initial investigations in a woman


Mid-luteal phase progesterone to confirm ovulation.
Take sample 7d before expected period (i.e. D21 in a 28d cycle).
If prolonged irregular menstrual cycles, depending on the timing of menstrual periods, serum progesterone may
need to be measured later (e.g. on D28 of a 35d cycle) to confirm ovulation, and repeated weekly thereafter until the
next menstrual cycle starts.
The following additional tests may be needed:
If irregular menstrual cycles, measure FSH & LH on D2-4 of cycle.
If symptoms of thyroid disease, perform TFT.
If ovulatory disorder, galactorrhoea, or a suspected pituitary tumour, measure prolactin.
Screen for chlamydia.

When to refer?
Refer to local guidelines as referral criteria may vary between health authorities.
If the woman is <36 yrs:
Consider referral if history/examination, and investigations are normal in both partners and the couple
has not conceived after 1 year.
Consider earlier referral if:
Woman Man
Age ≥36 yrs (refer after 6mths) Previous genital pathology
Amenorrhoea or oligomenorrhoea Previous urogenital surgery
Previous abdominal or pelvic surgery Previous STI
Previous PID Varicocele
Previous STI Significant systemic illness
Abnormal pelvic examination Abnormal genital examination
Known reason for infertility (e.g. prior treatment for cancer)

Take Home Points


Start investigations in couples who have not conceived after 1 year of regular unprotected sexual intercourse.
If the woman is <36 yrs - Consider referral if history/examination, and investigations are normal in both partners and the
couple has not conceived after 1 year.

References
Ref 1 - NICE Clinical Knowledge Summaries - Infertility
Ref 2 - NICE CG156 (2013) – Fertility: Assessment and treatment for people with fertility problems

https://www.mcqbank.co.uk/testhistory.php?view=1487811 17/120
04/09/2023, 11:44 mcqbank.co.uk/testhistory.php?view=1487811

Send us your feedback about this MCQ | Add question feedback to My Revision Notes | Rate this MCQ:

Extended Matching Question

Maximum Mark: 3
Mark Awarded: 3
Time taken to answer: 6 minutes 58 seconds.
Main Question

For each patient below with a renal mass, choose the most likely cause.
Select ONE option only from the list.
Each option may be selected once, more than once or not at all.
Answers

Your Answer:
H. Renal amyloidosis
Correct Answer:
Your answer is correct!

All answers:
1. A 55-year-old woman with a history of rheumatoid arthritis presents with
A. Polycystic kidney disease
hypertension and generalised oedema. Bilateral renal masses are palpable.
B. Renal stones
Urinalysis reveals proteinuria and the renal biopsy fluoresces bright green
C. Wilms' nephroblastoma
under polarized light after Congo red staining.
D. Rhabdomyosarcoma
E. Multiple myeloma
F. Monoclonal gammopathies of uncertain
origin
G. Renal tuberculosis
H. Renal amyloidosis

Your Answer:
C. Wilms' nephroblastoma
Correct Answer:
Your answer is correct!

All answers:
2. A 4-year-old boy presents with malaise, abdominal pain and intermittent A. Polycystic kidney disease
fevers. On examination, an abdominal mass is palpable and haematuria is B. Renal stones
present on urinalysis. C. Wilms' nephroblastoma
D. Rhabdomyosarcoma
E. Multiple myeloma
F. Monoclonal gammopathies of uncertain
origin
G. Renal tuberculosis
H. Renal amyloidosis

Your Answer:
A. Polycystic kidney disease
Correct Answer:
Your answer is correct!

All answers:
3. A 30-year-old man presents with haematuria. On examination, he is
A. Polycystic kidney disease
hypertensive and bilateral renal masses are palpable. Blood tests reveal a urea
B. Renal stones
of 80 mmol/l & a creatinine of 800 μmol/l. He collapses in A&E and a head CT
C. Wilms' nephroblastoma
shows a subarachnoid haemorrhage.
D. Rhabdomyosarcoma
E. Multiple myeloma
F. Monoclonal gammopathies of uncertain
origin
G. Renal tuberculosis
H. Renal amyloidosis

Feedback

Q1 - H

Amyloidosis is a clinical disorder caused by extracellular deposition of insoluble abnormal proteins.

https://www.mcqbank.co.uk/testhistory.php?view=1487811 18/120
04/09/2023, 11:44 mcqbank.co.uk/testhistory.php?view=1487811
All types of amyloid consist of a major fibrillar protein that defines the type of amyloid plus various minor
components.
AA is the most common systemic amyloidosis (formerly termed secondary amyloidosis).
Some of the conditions associated with AA include:
rheumatoid arthritis
juvenile chronic arthritis
ankylosing spondylitis
psoriasis & psoriatic arthritis
Still's disease
Behçet syndrome
familial Mediterranean fever
Crohn's disease
leprosy
osteomyelitis
tuberculosis
chronic bronchiectasis
Hodgkin's disease
renal cell carcinoma
carcinoma of the gut, lung, or urogenital tract
Presentation
Largely determined by the organ or system affected & the picture is often confused by the underlying disease.
Renal dysfunction is one of the most common presenting features of patients with systemic amyloidosis.
Diagnosis
Amyloidosis is diagnosed when Congo red–binding material (bright green fluorescence observed under polarized
light after Congo red staining) is demonstrated in a biopsy specimen.
For many years, rectal & gingival biopsies were considered the gold standard for the diagnosis of systemic
amyloidosis.
Biopsies can be obtained from any affected organ but blood vessel fragility associated with amyloid
deposition carries a risk of bleeding especially with internal organ biopsies so hence the rectal & gingival
sites were used.
However, more recently recognition that the capillaries in the subcutaneous fat are often involved in patients
with systemic amyloidosis and can often provide sufficient tissue for the diagnosis of amyloid.
Treatment
No treatment is yet available that specifically targets the amyloid deposits, and therapy is therefore aimed at
suppressing the underlying plasma cell dyscrasia along with supportive measures to support and possibly preserve
organ function.

Q2 -C

Wilms’ nephroblastoma is the fifth most common paediatric malignancy and the most common type of renal tumour in
children and the commonest intra-abdominal tumour of childhood.
It is characterized by an abnormal proliferation of the metanephric blastema cells, which are believed to be primitive
embryologic cells of the kidney.
It may be sporadic, familial (2%) or associated with Beckwith-Wiedemann syndrome, & WAGR syndrome (Wilms, Aniridia,
GU malformations, Retardation).
Presentation
Wilms’ tumour is diagnosed at a mean age of 3.5 years.
The most common feature at presentation is an abdominal mass.
Abdominal pain occurs in 30%-40% of cases.
Other signs & symptoms of Wilms’ tumour include hypertension, fever caused by tumour necrosis,
haematuria, and anaemia.
Treatment
Highly treatable involving multimodal therapy (i.e. surgery, radiation, chemotherapy).

Q3 - A

Autosomal dominant polycystic kidney disease (ADPKD) is the most frequent genetic cause of renal failure in adults.
Approximately 90% have an abnormality on chromosome 16.
ADPKD is a multisystemic and progressive disorder with formation of cysts in the kidney and other organs (e.g. liver,
pancreas, spleen)
Hepatic cysts may occur in 75% of patients >60y.
They appear to be affected by female steroid hormones, and, although they may occur in both sexes, massive
cysts occur almost exclusively in women.
Non-cystic manifestations include an increased incidence of mitral valve prolapse, intracranial berry aneurysms (in up
to 40% of cases) and an increased frequency of colonic diverticula & renal cell carcinoma.
Presentation

https://www.mcqbank.co.uk/testhistory.php?view=1487811 19/120
04/09/2023, 11:44 mcqbank.co.uk/testhistory.php?view=1487811
Clinical features usually begin in the 3rd - 4th decade of life, but cysts may be detectable in childhood.
Patients usually present with
haematuria - from cyst rupture
flank pain - from cyst enlargement
hypertension
stroke - from subarachnoid haemorrhage from intracranial aneurysms
renal failure
palpable masses in the flanks
increased incidence of UTIs
Investigations
Urinalysis - check for infection, protein (microalbuminuria occurs in about a third but heavy proteinuria is rare)
and haematuria.
Bloods - FBC (polycystic kidneys can produce excess erythropoietin and hence raise Hb), U&E
Ultrasound can detect renal cysts from 1-1.5 cm in diameter.
Sensitivity is 100% over the age of 20 but false negatives can occur below this age.
It is also possible to scan other organs like the liver or pancreas for cysts.
CT is more sensitive in that it can detect cysts down to 0.5 cm diameter but the dose of radiation is quite high.
MRI is an equally sensitive alternative.
Gene testing can be done by linkage or sequence analysis:
Management
Diagnosis of ADPKD has implications both for the patient & their family.
Treatment is currently supportive only.
No treatments have, as yet, been shown to halt or even slow disease progression.

Send us your feedback about this MCQ | Add question feedback to My Revision Notes | Rate this MCQ:

Single Best Answer Question

Maximum Mark: 1
Mark Awarded: 1
Time taken to answer: 0 minutes 37 seconds.
Main Question

Which one of the following organisms is NOT implicated in causing acute epididymo-orchitis?
Answers

Your answer:
B. Group B Streptococcus

Correct Answer:
Your answer is correct!

All Answers:
A. Chlamydia trachomatis
B. Group B Streptococcus
C. Escherichia coli
D. Pseudomonas
E. Neisseria gonorrhoeae

Feedback

Acute epididymo-orchitis is inflammation of the epididymis & adjacent testicle.

Incidence 1:1000 men.


Peaks in 15-30 years and the >60 year olds.
In men <35y.o, pathogens are most likely Chlamydia trachomatis & Neisseria gonorrhoeae.
Those >35y.o, pathogens are usually non-sexually transmitted gram -ve enteric organisms such as E. coli &
Pseudomonas.
If recent instrumentation or catheterisation has occurred then gram -ve enteric organisms are again more
commonly the cause
Presentation
Gradual in nature, with symptoms often peaking within 24 hours of onset.
Initially, the patient may note abdominal or flank pain which then localizes to the scrotum.
There may also be scrotal swelling, urinary frequency, urgency, dysuria, nausea, fever, chills
& urethral discharge.

https://www.mcqbank.co.uk/testhistory.php?view=1487811 20/120
04/09/2023, 11:44 mcqbank.co.uk/testhistory.php?view=1487811
On examination, the scrotum is erythematous & oedematous.
Early on, in cases without significant testicular involvement, tenderness may be clearly localized to the
epididymis.
The scrotum is often fixed to the underlying epididymis.
Classically, scrotal elevation decreases pain in epididymitis & not in torsion (Prehn's
sign).
Investigations
urinalysis & MSU
urethral swab
first void urine: for gonorrhoea & chlamydia
Management
May need to screen patient & contacts for STI and for other patients may need to investigate for anatomical
abnormalities of the urinary tract.
Treatment
Uncomplicated gonorrhoea
cefixime or ciprofloxacin
Uncomplicated genital chlamydial infection, non-gonococcal urethritis & non-specific genital infection
doxycycline or azithromycin
If urine dipstick positive and most likely enteric organisms
treat as for 'complicated UTI' - e.g. trimethoprim or ofloxacin for 14 days

Send us your feedback about this MCQ | Add question feedback to My Revision Notes | Rate this MCQ:

Single Best Answer Question

Maximum Mark: 1
Mark Awarded: 1
Time taken to answer: 0 minutes 18 seconds.
Main Question

A 60-year-old man complains of urinary frequency, dribbling, & reduced force of stream over the last year. Rectal examination
reveals a moderately enlarged prostate. Which of the following drugs would be the MOST appropriate treatment to start?
Answers

Your answer:
B. Tamsulosin

Correct Answer:
Your answer is correct!

All Answers:
A. Trimethoprim
B. Tamsulosin
C. Oxybutynin
D. Duloxetine
E. Desmopressin

Feedback

Benign prostatic hyperplasia (BPH) is increase in size of the prostate.

It is characterized by hyperplasia of prostatic stromal & epithelial cells, resulting in the formation of large discrete
nodules in the periurethral region of the prostate.
An estimated 50% of men have histological evidence of BPH by 50 years & 75% by 75 years.
Presentation
When sufficiently large, the nodules compress the urethral canal leading to symptoms of
urinary hesitancy
frequency
dysuria
↑ risk of urinary tract infections
retention
The digital rectal examination can assess prostate size & contour.
The median sulcus is not obliterated in in BPH unlike prostatic carcinoma.
Symptoms are not related to prostate size.

https://www.mcqbank.co.uk/testhistory.php?view=1487811 21/120
04/09/2023, 11:44 mcqbank.co.uk/testhistory.php?view=1487811
A more precise volumetric determination can be made using transrectal ultrasonography (TRUS).
Management
Investigation of benign prostatic hypertrophy begins with Urinalysis and sending a MSU for MC&S.
Routine blood tests include a FBC, U&E & LFT (to exclude possible prostate malignancy that has
metastasised) & a Prostate specific antigen (PSA).
Medical treatment includes
α1-adrenergic receptor antagonists (e.g. terazosin, tamsulosin)
relaxes smooth muscle in the prostate & bladder neck thereby decreasing the degree of blockage
of urine flow.
5α-reductase inhibitors (e.g. finasteride)
inhibits 5a-reductase, which reduces production of dihydrotestosterone, a hormone responsible
for enlarging the prostate.
When used together with alpha blockers a reduction of BPH progression to acute urinary retention & surgery
has been noted in patients with larger prostates.
Minimally invasive surgical options include:
Transurethral microwave thermotherapy (TUMT)
Transurethral Needle Ablation (TUNA)
which deliver enough energy into the prostate to create sufficient heat to cause cell death & eventual
shrinkage of the prostate.
Surgical options
Transurethral resection of prostate (TURP) which is considered the gold standard in surgical therapy & involves
removing part of the prostate through the urethra.

Send us your feedback about this MCQ | Add question feedback to My Revision Notes | Rate this MCQ:

Single Best Answer Question

Maximum Mark: 1
Mark Awarded: 1
Time taken to answer: 3 minutes 12 seconds.
Main Question

Which of the following BEST describes the type of urinary incontinence where the patient has mental or physical disabilities that
keep them from urinating normally, although the urinary system itself is intact?
Answers

Your answer:
A. Functional incontinence

Correct Answer:
Your answer is correct!

All Answers:
A. Functional incontinence
B. Stress incontinence
C. Urge incontinence
D. Mixed incontinence
E. True incontinence

Feedback

Urinary incontinence is the involuntary leakage of urine. There are several different types:

Functional incontinence occurs when patients have mental or physical disabilities (e.g. Parkinson's disease) that keep
them from urinating normally, although the urinary system itself is intact.
Stress incontinence is the involuntary loss of urine through an intact urethra secondary to an increase in intra-abdominal
pressure and in the absence of detrusor activity.
It is associated with a defect of the urethral sphincter.
Urine loss occurs when the intra-vesical pressure exceeds the maximum urethral pressure.
Presentation
Most patients complain of involuntary loss of urine on coughing or straining.
Often, this accompanied by symptoms of frequency & urgency.
Usually result of
childbirth - pregnancy, labour & delivery may all cause mechanical & denervation injury to the normal
supports of the bladder neck & proximal urethra
menopausal changes - decreased intraurethral pressure due to lack of oestrogen

https://www.mcqbank.co.uk/testhistory.php?view=1487811 22/120
04/09/2023, 11:44 mcqbank.co.uk/testhistory.php?view=1487811
Investigations
The urinary stress test is a simple test for stress incontinence.
The patient is examined in the lithotomy position with a full bladder.
The examiner asks the patient to cough whilst observing the urethral meatus.
Stress incontinence is indicated by short spurts of urinary loss simultaneous with each
cough.
Loss of large volumes of urine or delayed leakage suggests uninhibited bladder contractions.
The test should be repeated in the standing position if the initial result is negative.
Urodynamic studies include urinary flow rate, cystometry, urethral pressure profile & electromyography.
Management
Conservative management
lifestyle interventions such as caffeine reduction, reducing fluid intake or weight loss.
pelvic floor muscle training.
Anticholinergics such as oxybutynin.
Surgical treatment is reserved for those in whom conservative measures have failed.
Urge incontinence or detrusor instability is characterised by uncontrolled contraction of the bladder wall (detrusor
muscle) producing urgency & sometimes leakage, often with frequency & nocturia.
Detrusor overactivity is the second commonest cause of female urinary incontinence behind stress incontinence.
Risk factors include multiple sclerosis & stroke but most cases have no specific cause.
In men, urge incontinence may be due to neurological disease or an enlarged prostate.
Presentation
Involuntary urine leakage accompanied by, or immediately preceded by, urgency of micturition. This
means a sudden & compelling desire to urinate that cannot be deferred.
Urinary frequency is also common.
Investigations
Investigations include mid-stream urine MC&S to exclude urinary tract infection.
Urodynamic studies show involuntary contraction of bladder during filling.
Management
Management of incontinence in adults arising from detrusor instability is by combining:
Conservative management
Bladder training - the goals are to increase the amount of time between emptying the bladder
and the amount of fluids the bladder can hold.
Anticholinergics, e.g. oxybutynin have a direct relaxant effect on urinary smooth muscle. They
reduce involuntary detrusor contractions and increase bladder capacity.
Invasive options
Botulinum toxin A
Injection of the bladder wall with botulinum toxin A is the first-line invasive option.
Sacral nerve stimulation is used in patients who do not respond to other treatments.
Surgery is only indicated for intractable & severe detrusor overactivity.
Laparoscopic augmentation cystoplasty.
Mixed incontinence is involuntary leakage of urine associated with both urgency & exertion, effort, sneezing or coughing.
Overactive bladder syndrome (OAB) is urgency that occurs with or without urge incontinence, and usually with
frequency & nocturia.
It may be called 'OAB wet' or 'OAB dry', depending on whether or not the urgency is associated with incontinence.
The usual cause of this problem is detrusor overactivity.
Overflow incontinence is incomplete bladder emptying secondary to impaired detrusor contractility or bladder outlet
obstruction.
Factors involved in the development of overflow incontinence are
physical obstruction, such as pelvic organ prolapse & enlarged prostate
neurological abnormalities, such as spinal cord injuries
It is also commonly associated with bladder neuropathy as occurs in diabetes mellitus
Patients often complain of continuous small-volume leakage associated with weak urinary stream, dribbling,
hesitancy, frequency, & nocturia.
True incontinence may be due to a fistulous tract between the vagina & either the ureter, bladder, or urethra.
There is continuous leakage of urine.

Reference
Patient.co.uk - Urinary Incontinence

Send us your feedback about this MCQ | Add question feedback to My Revision Notes | Rate this MCQ:

https://www.mcqbank.co.uk/testhistory.php?view=1487811 23/120
04/09/2023, 11:44 mcqbank.co.uk/testhistory.php?view=1487811

Extended Matching Question

Maximum Mark: 3
Mark Awarded: 3
Time taken to answer: 1 minutes 24 seconds.
Main Question

For each patient below with renal trauma, choose the most appropriate catheterisation option.
Select ONE option only from the list.
Each option may be selected once, more than once or not at all.
Answers

Your Answer:
A. Observation only – catheter not required
Correct Answer:
Your answer is correct!

All answers:
A. Observation only – catheter not required
1. A 25-year-old kickboxer has received a kick to his kidneys. His urine
B. Urethral catheterisation
appears clear but dipstick reveals 2+ blood. Subsequent IVU is normal.
C. Suprapubic catheterisation
D. Convene catheterisation
E. In-out catheterisation
F. Guidewire assisted catheterisation
G. Flexible urethroscopy assisted
catheterisation
H. Nephrostomy

Your Answer:
B. Urethral catheterisation
Correct Answer:
Your answer is correct!

All answers:
A. Observation only – catheter not required
2. A 30-year-old roofer falls off a ladder and sustains a pelvic fracture.
B. Urethral catheterisation
Cystogram shows minor extraperitoneal urine extravasation.
C. Suprapubic catheterisation
D. Convene catheterisation
E. In-out catheterisation
F. Guidewire assisted catheterisation
G. Flexible urethroscopy assisted
catheterisation
H. Nephrostomy

Your Answer:
C. Suprapubic catheterisation
Correct Answer:
Your answer is correct!

All answers:
3. An 18-year-old motorcyclist has lost control of his bike at a roundabout. He A. Observation only – catheter not required
has injuries to his perineum and blood is present at the urethral meatus. His B. Urethral catheterisation
bladder is palpable. Foley urethral catheterisation is attempted but fails. C. Suprapubic catheterisation
D. Convene catheterisation
E. In-out catheterisation
F. Guidewire assisted catheterisation
G. Flexible urethroscopy assisted
catheterisation
H. Nephrostomy

Feedback

Q1 - A

Most minor renal trauma can be managed conservatively.

Blunt renal injuries


The severity of blunt kidney trauma varies widely.
When an injury is minor or low velocity, the kidney may only be bruised.
When an injury is more severe or high velocity, the kidney may be lacerated, and urine & blood may leak into the
surrounding tissue.
https://www.mcqbank.co.uk/testhistory.php?view=1487811 24/120
04/09/2023, 11:44 mcqbank.co.uk/testhistory.php?view=1487811
If the kidney is torn from its attachment to blood vessels, bleeding may be profuse, resulting in shock or death.
Most kidney injuries result in haematuria.
Adults who have mild symptoms & microscopic haematuria most likely have a minor bruise that will heal on its own.
Where more serious injury is suspected a contrast CT may be indicated.

Q2 - B

Urethral catheterisation is appropriate here in the absence of evidence of injury to the urethra. Suprapubic catheterisation is not
indicated here and he does not need a cystoscopy in view of the cystogram that he has had.
Extraperitoneal bladder rupture is the most common type or bladder injury, accounting for ~85% of cases, usually the
result of pelvic fractures or penetrating trauma.
Up to a quarter of patients with pelvic fractures also suffer genitourinary injuries, particularly those fractures at or near the
pubic symphysis.
The urinary meatus should be inspected for the presence of blood before inserting an indwelling catheter.
If blood is present, it's likely that the urethra is also injured, and the use of a urethral catheter is contraindicated.
Further radiographic studies will be needed to determine the nature & degree of the injury.
CT cystography is the investigation of choice when evaluating patients with suspected bladder injury which may
reveala a variable path of extravasated contrast material.
Treatment
Most extraperitoneal bladder leaks can be effectively managed with maximal bladder drainage per urethral or
suprapubic catheter.
Approximately 85% of such injuries will heal within 7-10 days, at which point the catheter can be removed and a trial
of voiding completed.
Overall, nearly all extraperitoneal bladder injuries heal within 3 weeks.
However, if surgery is pursued for other indications, extraperitoneal bladder injuries may be repaired surgically in the
same setting if the patient is stable.
[Reference - Medscape - Bladder Trauma Treatment & Management]

Q3 - C

This patient is considered to have a transected urethra until considered otherwise.

Urethral injuries can be classified into 2 categories based on the anatomical site of the trauma.

Posterior urethral injuries are located in the membranous & prostatic urethra.
These injuries are most commonly related to major blunt trauma such as RTA & falls and are commonly associated
with pelvic fractures.
Anterior urethral injuries are located distal to the membranous urethra.
Most anterior urethral injuries come from blunt trauma to the perineum (straddle injuries), and many have delayed
manifestation, appearing years later as a stricture.
Presentation
Look for perineal bruising & blood at the external urethral meatus
And perform a rectal examination - abnormally high riding prostate or inability to palpate the prostate imply
urethral injury
Management
If you suspect urethral injury do not attempt urethral catheterisation but refer urgently to the Urologists.
Some Urologists perform a retrograde urethrogram.
Others prefer suprapubic catheterisation & subsequent imaging.

Send us your feedback about this MCQ | Add question feedback to My Revision Notes | Rate this MCQ:

https://www.mcqbank.co.uk/testhistory.php?view=1487811 25/120
04/09/2023, 11:44 mcqbank.co.uk/testhistory.php?view=1487811

Single Best Answer Question

Maximum Mark: 1
Mark Awarded: 1
Time taken to answer: 0 minutes 47 seconds.
Main Question

Which one of the following is NOT a recognised treatment for benign prostatic hyperplasia?
Answers

Your answer:
B. α1-adrenergic receptor agonists

Correct Answer:
Your answer is correct!

All Answers:
A. 5α-reductase inhibitors
B. α1-adrenergic receptor agonists
C. Transurethral resection of prostate (TURP)
D. Transurethral Needle Ablation (TUNA)
E. Transurethral microwave thermotherapy (TUMT)

Feedback

Benign prostatic hyperplasia (BPH) is increase in size of the prostate.

It is characterized by hyperplasia of prostatic stromal & epithelial cells, resulting in the formation of large discrete
nodules in the periurethral region of the prostate.
An estimated 50% of men have histological evidence of BPH by 50 years & 75% by 75 years.
Presentation
When sufficiently large, the nodules compress the urethral canal leading to symptoms of
urinary hesitancy
frequency
dysuria
↑ risk of urinary tract infections
retention
The digital rectal examination can assess prostate size & contour.
The median sulcus is not obliterated in in BPH unlike prostatic carcinoma .
Symptoms are not related to prostate size.
A more precise volumetric determination can be made using transrectal ultrasonography (TRUS).
Management
Investigation of benign prostatic hypertrophy begins with Urinalysis and sending a MSU for MC&S.
Routine blood tests include a FBC, U&E & LFT (to exclude possible prostate malignancy that has
metastasised) & a Prostate specific antigen (PSA).
Medical treatment includes
α1-adrenergic receptor antagonists (e.g. terazosin, tamsulosin)
relaxes smooth muscle in the prostate & bladder neck thereby decreasing the degree of blockage
of urine flow.
5α-reductase inhibitors (e.g. finasteride)
inhibits 5a-reductase, which reduces production of dihydrotestosterone, a hormone responsible
for enlarging the prostate.
When used together with alpha blockers a reduction of BPH progression to acute urinary retention & surgery
has been noted in patients with larger prostates.
Minimally invasive surgical options include:
Transurethral microwave thermotherapy (TUMT)
Transurethral Needle Ablation (TUNA)
which deliver enough energy into the prostate to create sufficient heat to cause cell death & eventual
shrinkage of the prostate.
Surgical options
Transurethral resection of prostate (TURP) which is considered the gold standard in surgical therapy & involves
removing part of the prostate through the urethra.

Send us your feedback about this MCQ | Add question feedback to My Revision Notes | Rate this MCQ:

https://www.mcqbank.co.uk/testhistory.php?view=1487811 26/120
04/09/2023, 11:44 mcqbank.co.uk/testhistory.php?view=1487811

Extended Matching Question

Maximum Mark: 3
Mark Awarded: 2
Time taken to answer: 4 minutes 54 seconds.
Main Question

For each patient below with urinary tract obstruction, choose the most likely cause.
Select ONE option only from the list.
Each option may be selected once, more than once or not at all.
Answers

Your Answer:
B. Prostatic carcinoma
Correct Answer:
E. Retroperitoneal fibrosis

1. A 50-year-old man with advanced gastric carcinoma presents with vague


pains in his back for the last 4 months. On examination, his blood pressure is All answers:
160/110mmHg. Blood tests show a urea 10mmol/l, creatinine 110μmol/l, ESR A. Retroperitoneal haematoma
70mm & CRP 60mg/l. Intravenous urography shows bilateral hydronephrosis B. Prostatic carcinoma
with drawing together of the ureters in the midline and CT/MRI reveals a peri- C. Renal calculi
aortic mass. Bone scan is clear. D. Clot retention
E. Retroperitoneal fibrosis
F. Benign prostatic hypertrophy
G. Lumbar vertebral metastases
H. Ureteric stricture

Your Answer:
C. Renal calculi
Correct Answer:
Your answer is correct!

All answers:
2. A 60-year-old woman with a history of recurrent UTIs presents with oliguria,
A. Retroperitoneal haematoma
haematuria and loin pain. Ultrasound shows a hydronephrosis of the right
B. Prostatic carcinoma
kidney. Diagnosis is confirmed on the plain abdominal film.
C. Renal calculi
D. Clot retention
E. Retroperitoneal fibrosis
F. Benign prostatic hypertrophy
G. Lumbar vertebral metastases
H. Ureteric stricture

Your Answer:
F. Benign prostatic hypertrophy
Correct Answer:
Your answer is correct!

All answers:
3. A 75-year-old man presents with urinary hesitancy and a poor stream. On A. Retroperitoneal haematoma
rectal examination he has a smoothly enlarged prostate. His PSA is 6ng/ml. B. Prostatic carcinoma
C. Renal calculi
D. Clot retention
E. Retroperitoneal fibrosis
F. Benign prostatic hypertrophy
G. Lumbar vertebral metastases
H. Ureteric stricture

Feedback

Q1 - E

Retroperitoneal fibrosis (RPF) or Ormond's disease involves proliferation of fibrous tissue in the retroperitoneum
typically over the anterior surface of L4-L5. This leads to entrapment of retroperitoneal structures, notably the ureters.
RPF is rare (incidence 1 per 200,000-500,000 population)
It is 2x as common in males & peak incidence is 40-60 years.
Aetiology
In most cases, the aetiology is unknown.
It is associated with

https://www.mcqbank.co.uk/testhistory.php?view=1487811 27/120
04/09/2023, 11:44 mcqbank.co.uk/testhistory.php?view=1487811
autoimmune diseases
ankylosing spondylitis, systemic lupus erythematosus, scleroderma, systemic vasculitis, Wegener's
granulomatosis, polyarteritis nodosa, Raynaud disease, rheumatoid arthritis, Riedel thyroiditis & immune
complex membranous glomerulonephritis
metastatic malignancy
drugs
beta-blockers, methysergide & methyldopa
Presentation
The symptoms are usually non-specific.
Duration of symptoms prior to diagnosis is approximately 6-12 months.
The most common symptom is dull, poorly localized, non colicky pain in the flank, back, scrotum, or lower
abdomen.
Other symptoms include fever, lower-extremity oedema, phlebitis, & deep venous thrombosis.
Approximately half of individuals with RPF have hypertension, which is probably due to renin release
secondary to obstructive uropathy.
Obstruction of the ureters with a varying degree of renal insufficiency is the earliest & most common organ
involvement.
Investigations
Blood tests show a raised ESR & CRP with anaemia & uraemia.
CT urogram shows bilateral hydronephrosis with drawing together of the ureters in the midline
"Maiden-Waist" Sign on excretory urogram:

CT/MRI reveals a peri-aortic mass


Biopsy of retroperitoneal tissue at time of operation allows confirmation of diagnosis & exclusion of other pathologies
such as malignancy.
Management
In the absence of severe urinary tract obstruction (which generally requires surgery with omental wrapping),
empirical treatment includes corticosteroids, tamoxifen, & azathioprine.

Q2 - C

She has ureteric obstruction secondary to Renal calculi. She had an impressive staghorn calculus demonstrable on the KUB film.

A renal stone or calculus is a solid aggregation formed in the kidneys from dietary minerals in the urine.

Demographics
Annual incidence is 1:1000 & the average lifetime risk is 10%
Male to female ratio is 3:1
The peak age is between 30 - 50.
Pathology
Consist of crystal aggregates that form in the collecting ducts which can then be deposited anywhere from the renal
pelvis to the urethra.
calcium oxalate (75%)
magnesium ammonium phosphate [struvite] (10%)

https://www.mcqbank.co.uk/testhistory.php?view=1487811 28/120
04/09/2023, 11:44 mcqbank.co.uk/testhistory.php?view=1487811
urate (5%)
hydroxyapatite (5%)
cystine (1%)
Risk factors
Anatomical anomalies in the kidneys &/or urinary tract,
e.g. horseshoe kidney, ureteral stricture
Family history of renal stones
Hypertension
Gout
Hyperparathyroidism
Immobilisation
Relative dehydration
commoner in hot climates
Metabolic disorders
which increase excretion of solutes
e.g. chronic metabolic acidosis, hypercalciuria, hyperuricosuria
Deficiency of citrate in the urine
Cystinuria (an autosomal recessive aminoaciduria)
Drugs, especially thiazide diuretics
Presentation
Many stones are asymptomatic and discovered during investigations for other conditions.
Classic symptom is sudden, severe pain.
Pain starts in the loin about the level of the costovertebral angle & moves to the groin, with tenderness of the
loin or renal angle, sometimes with haematuria.
If the stone is high & distends the renal capsule then pain will be in the flank but as it moves down pain will
move anteriorly and down towards the groin.
Other symptoms which may be present include:
Rigors and fever
Dysuria
Haematuria
Urinary retention
Nausea and vomiting
Investigations
Urinalysis
The stone often causes some bleeding into the renal tract and this may produce a positive result for blood on
stick testing
a negative test does not exclude the diagnosis
Mid-stream specimen of urine for microscopy, culture & sensitivities.
Bloods
FBC, U&E, calcium & phosphate and urate.
Blood cultures & parathyroid hormone if indicated.
Encourage the patient to try to catch the stone for analysis.
This may mean urinating through a tea strainer, filter paper such as a coffee filter or a gauze.
Helical CT or CT KUB is the gold standard for the investigation of urinary stones (99% of stones visible) & has
superseded intravenous urography (IVU).
Plain X-rays (80% of stones are visible on a KUB plain film).
Renal ultrasound may also detect stones & reveal any evidence of obstruction (hydronephrosis or hydroureter)
Management
Most kidney stones do not require surgery and will pass on their own.
Surgery is necessary when the pain is persistent & severe, in renal failure and when there is a kidney
infection.
It may also be advisable if the stone fails to pass or move after 30 days.
Finding a significant stone before it passes into the ureter allows physicians to fragment it surgically before it causes
any severe problems.
In most of these cases, non-invasive extracorporeal shock wave lithotripsy (ESWL) will be used.
Otherwise some form of invasive procedure is required.
With approaches including
Ureteroscopic fragmentation (or simple basket extraction if feasible) using laser
Ultrasonic or mechanical (pneumatic, shock-wave) forms of energy to fragment the larger stones.
Percutaneous nephrolithotomy (PCNL) or rarely open surgery may ultimately be necessary for large or complicated
stones or stones which fail other less invasive attempts at treatment.
Prevention
A normal calcium intake is now recommended as low calcium diets increase oxalate excretion.

https://www.mcqbank.co.uk/testhistory.php?view=1487811 29/120
04/09/2023, 11:44 mcqbank.co.uk/testhistory.php?view=1487811
Advise to consume less oxalate by reducing tea, chocolate, nuts, strawberries, rhubarb, spinach, beans & beetroot &
to drink plenty of fluids especially in the summer.
Complications
Complete blockage of the urinary flow from a kidney decreases glomerular filtration rate (GFR) and, if it persists
for >48 hours, may cause irreversible renal damage.
If ureteric stones cause symptoms after 4 weeks, there is a 20% risk of complications, including deterioration of
renal function, sepsis, & ureteric stricture.
Infection can be life-threatening.
Persisting obstruction predisposes to pyelonephritis.

Q3 - F

Benign prostatic hyperplasia (BPH) is increase in size of the prostate.

It is characterized by hyperplasia of prostatic stromal & epithelial cells, resulting in the formation of large discrete
nodules in the periurethral region of the prostate.
An estimated 50% of men have histological evidence of BPH by 50 years & 75% by 75 years.
Presentation
When sufficiently large, the nodules compress the urethral canal leading to symptoms of
urinary hesitancy
frequency
dysuria
↑ risk of urinary tract infections
retention
The digital rectal examination can assess prostate size & contour.
The median sulcus is not obliterated in in BPH unlike prostatic carcinoma .
Symptoms are not related to prostate size.
A more precise volumetric determination can be made using transrectal ultrasonography (TRUS).
Management
Investigation of benign prostatic hypertrophy begins with Urinalysis and sending a MSU for MC&S.
Routine blood tests include a FBC, U&E & LFT (to exclude possible prostate malignancy that has
metastasised) & a Prostate specific antigen (PSA).
Medical treatment includes
α1-adrenergic receptor antagonists (e.g. terazosin, tamsulosin)
relaxes smooth muscle in the prostate & bladder neck thereby decreasing the degree of blockage
of urine flow.
5α-reductase inhibitors (e.g. finasteride)
inhibits 5a-reductase, which reduces production of dihydrotestosterone, a hormone responsible
for enlarging the prostate.
When used together with alpha blockers a reduction of BPH progression to acute urinary retention & surgery
has been noted in patients with larger prostates.
Minimally invasive surgical options include:
Transurethral microwave thermotherapy (TUMT)
Transurethral Needle Ablation (TUNA)
which deliver enough energy into the prostate to create sufficient heat to cause cell death & eventual
shrinkage of the prostate.
Surgical options
Transurethral resection of prostate (TURP) which is considered the gold standard in surgical therapy & involves
removing part of the prostate through the urethra.

Send us your feedback about this MCQ | Add question feedback to My Revision Notes | Rate this MCQ:

Single Best Answer Question

Maximum Mark: 1
Mark Awarded: 0
Time taken to answer: 0 minutes 52 seconds.
Main Question

Which ONE of the following conditions give rise to a decreased specific gravity on urine dipstick analysis?
Answers

Your answer:
B. SIADH
https://www.mcqbank.co.uk/testhistory.php?view=1487811 30/120
04/09/2023, 11:44 mcqbank.co.uk/testhistory.php?view=1487811

Correct Answer:
E. Excessive fluid intake

All Answers:
A. Dehydration
B. SIADH
C. Cardiac failure
D. Renal artery stenosis
E. Excessive fluid intake

Feedback

Specific gravity (SG) <1.008 is dilute and >1.020 is concentrated.

Increased SG is seen in
conditions causing dehydration
glycosuria
renal artery stenosis
heart failure (secondary to decreased blood flow to the kidneys)
inappropriate antidiuretic hormone secretion
proteinuria
Falsely high readings may occur in the presence of dextran solutions and IV radiopaque dyes.

Decreased SG is seen in
excessive fluid intake
renal failure
pyelonephritis
central and nephrogenic diabetes insipidus
Falsely low readings are associated with alkaline urine (e.g. a high-citrate diet).

Send us your feedback about this MCQ | Add question feedback to My Revision Notes | Rate this MCQ:

Single Best Answer Question

Maximum Mark: 1
Mark Awarded: 0
Time taken to answer: 1 minutes 11 seconds.
Main Question

Which one of the following blood gas values is MOST likely to belong to a patient with chronic renal failure (eating a normal
protein diet)?

pH HCO3‑ (mmol/l) PCO2 (kPa)


A 7.65 48 6
B 7.50 15 2.5
C 7.40 24 5.3
D 7.32 30 8
E 7.31 16 4.4

Answers

Your answer:
D.

Correct Answer:
E.

All Answers:
A.
B.
C.
D.
E.

https://www.mcqbank.co.uk/testhistory.php?view=1487811 31/120
04/09/2023, 11:44 mcqbank.co.uk/testhistory.php?view=1487811
Feedback

In chronic renal failure with ingestion of normal protein, fixed acids (lactic acid, phosphoric acid, sulphuric acid, acetoacetic
acid, & beta-hydroxybutyric acid) will be produced from protein catabolism. Because the failing kidney does not produce enough
NH4+ to excrete all the of the fixed acid, metabolic acidosis (with respiratory compensation) results.

Send us your feedback about this MCQ | Add question feedback to My Revision Notes | Rate this MCQ:

Extended Matching Question

Maximum Mark: 3
Mark Awarded: 3
Time taken to answer: 4 minutes 18 seconds.
Main Question

For each patient below with urinary symptoms, choose the most likely cause.
Select ONE option only from the list.
Each option may be selected once, more than once or not at all.
Answers

Your Answer:
A. Normal pressure hydrocephalus
Correct Answer:
Your answer is correct!

1. A 75-year-old man presents with problems walking described by his


daughter as a “shuffling wide-based” gait which has developed over the
All answers:
previous 2 years. He also reports urinary frequency, occasional urge
A. Normal pressure hydrocephalus
incontinence, and some memory loss. On examination, his legs have reduced
B. Renal colic
power symmetrically, but tone is normal with no cogwheeling. He has normal
C. Alzheimer’s disease
facial expressiveness. Digital rectal exam reveals a smooth, rubbery and mildly
D. Parkinson's disease
enlarged prostate.
E. Urinary tract infection
F. Benign prostatic enlargement
G. Acute prostatitis
H. Prostate cancer

Your Answer:
F. Benign prostatic enlargement
Correct Answer:
Your answer is correct!

2. A 60-year-old man presents to his GP with a 3-month history of increasing


urinary frequency without burning and nocturia 3 times each evening. He has All answers:
limited his fluid consumption and caffeine intake in the evening without much A. Normal pressure hydrocephalus
benefit. Examination demonstrates no suprapubic mass or tenderness. A rectal B. Renal colic
examination reveals normal rectal tone and a moderately enlarged prostate C. Alzheimer’s disease
without nodules or tenderness. D. Parkinson's disease
E. Urinary tract infection
F. Benign prostatic enlargement
G. Acute prostatitis
H. Prostate cancer

Your Answer:
E. Urinary tract infection
Correct Answer:
Your answer is correct!

3. A 70-year-old man who has been an inpatient for 4 days with an


All answers:
exacerbation of congestive heart failure, presents with acute confusion. He has
A. Normal pressure hydrocephalus
had an indwelling urinary catheter to strictly monitor urine output since
B. Renal colic
admission. He has no previous history of dementia. Examination confirms
C. Alzheimer’s disease
fever and suprapubic tenderness.
D. Parkinson's disease
E. Urinary tract infection
F. Benign prostatic enlargement
G. Acute prostatitis
H. Prostate cancer

https://www.mcqbank.co.uk/testhistory.php?view=1487811 32/120
04/09/2023, 11:44 mcqbank.co.uk/testhistory.php?view=1487811
Feedback

Q1 - A

The triad of urinary incontinence, dementia & abnormal gait is suggestive of normal pressure hydrocephalus. Most, if not all,
men aged 75 will have at least a mildly enlarged prostate. C.f. normal pressure hydrocephalus, Parkinson’s disease is more
likely to exhibit asymmetric signs and of course a predominant tremor usually affecting the arms and loss of facial expression.
Cognitive impairment generally precedes any gait impairment in Alzheimer’s.

Q2 – F

The lower urinary tract symptoms in combination with this man’s age and examination findings suggest benign prostatic
enlargement. Prostate nodules or asymmetry would be more consistent with prostate cancer. A warm tender prostate with
signs of infection would be suspicious for prostatitis.

Q3 – E

UTI is a common cause of confusion in the elderly. Risk factors in this man include being catheterised. Renal colic typically has
a more dramatic presentation with intense colicky abdominal pain.

Normal pressure hydrocephalus


Classically characterised by the triad of urinary incontinence, dementia & abnormal gait – which can be remembered
with the unkind mnemonic Wet, Wacky, & Wobbly.
As the name suggests, mean CSF opening pressure is within the normal range (<18 cmH2O or 13 mmHg).
The gait is described as a broad-based gait pattern with diminished height of the steps which some describe as being
“magnetic” where the patients' feet appear to be magnetically attached to the floor.

Classic NPH Gait Pre-Shunt


Surgery
HSML

01:16

Majority of cases are idiopathic. Incidence is much higher in elderly populations.


CT & MRI show enlarged lateral & third ventricles.

Differential diagnosis includes parkinsonism and other common dementia disorders.


If treated early enough it responds to ventriculoperitoneal shunting but selection of patients for surgery is challenging
as many are considered a high surgical risk.

Renal colic
Renal or ureteric colic is a common condition generally describing acute and severe loin pain caused when a urinary
stone moves from the kidney or obstructs the flow of urine.
Risk factors include chronic dehydration, family history, Crohn's disease & medications such as protease inhibitors.
The main symptom is severe unilateral abdominal pain starting in the loin or flank and radiating to the labia in
women or the groin or testicle in men. Pain is often accompanied by nausea, vomiting, & haematuria.
Complications include urinary tract obstruction (hesitancy of micturition or an intermittent urinary stream) or
coexisting urinary tract infection (fever & sweats)
Immediate admission should be arranged if the person is:

https://www.mcqbank.co.uk/testhistory.php?view=1487811 33/120
04/09/2023, 11:44 mcqbank.co.uk/testhistory.php?view=1487811
In shock or has signs of systemic infection (e.g. fever & sweats).
At ↑ risk of acute kidney injury e.g. pre-existing chronic kidney disease, solitary or transplanted kidney, or if
bilateral obstructing stones are suspected.
Dehydrated and cannot take oral fluids due to nausea &/or vomiting.
For all others with suspected renal colic:
Urgent (within 24h of presentation) imaging should be offered (low-dose non-contrast CT for most adults;
ultrasound for pregnant women, children, & young people).
NSAID by any route should be offered for pain relief. If NSAIDs are contraindicated or ineffective, intravenous
paracetamol should be offered. If both NSAIDs & IV paracetamol are contraindicated or ineffective, or IV paracetamol
is unavailable, an opioid analgesic should be considered.
Management of urinary stones depends on factors such as size of stone, severity of symptoms, location of stone (renal or
ureteric), and patient’s age. Options include watchful waiting, medical expulsive therapy, & surgical treatment.

Alzheimer’s disease
Most common cause of dementia, responsible for ~70% of all dementias.
Prevalence is strongly linked to age, with >1% of 60-64 y.o being diagnosed with the condition, compared to ~30%
of those over 85-90y.
Results from accumulation of cerebral amyloid-β.
In classical/typical Alzheimer's disease, the patient will present initially with antegrade episodic memory deficits. Over
time (often years), the disease progresses, with eventual involvement of attentional & executive processes, semantic
memory, praxis, and visuoperceptual abilities.
Neuropsychiatric symptoms are also common and eventually affect almost all patients. These include apathy,
depression, anxiety, aggression/agitation, and psychosis (delusions & hallucinations)
~65% will go on to suffer from incontinence.
Only definitive diagnostic test is brain biopsy which in practice is rarely obtained. As such, the combination of clinical
features & neuroimaging are usually considered sufficient.
No cure - some drugs have been developed trying to improve symptoms or, at least, temporarily slow down their
progression such as Cholinesterase inhibitors e.g. donepezil or partial NMDA receptor antagonists. Medication is also
often prescribed for behavioural symptoms.

Parkinson's disease
Characterised by a resting tremor, rigidity, bradykinesia, and postural instability.
Parkinsonian gait has many features, including:
Short steps, reduced arm swing, stooped posture and festination (hasty but short steps attempting to
compensate for displaced centre of gravity).

Gait impairments in Parkinsons disease

HSML

02:27

Patients may also demonstrate non-motor symptoms:


constipation
anosmia (often an early symptom predating the cardinal features)
REM sleep behavioural disorder
Autonomic dysfunction such as postural hypotension
Psychosis especially visual hallucinations,
Bradyphrenia (slowness of thought) & dementia (generally a late feature)
Fatigue & somnolence.
The most common urinary symptoms in Parkinson's disease are reported to be nocturia, followed by frequency and urinary
incontinence.
Drug options broadly include
Monoamine-oxidase-B inhibitors (MAO-BIs) - selegiline, rasagiline.
Dopamine agonists - Pramipexole, ropinirole & rotigotine.
Amantadine (weak antagonist of NMDA-type glutamate receptor, increases dopamine release & blocks dopamine
reuptake).
Anticholinergics.

https://www.mcqbank.co.uk/testhistory.php?view=1487811 34/120
04/09/2023, 11:44 mcqbank.co.uk/testhistory.php?view=1487811

Urinary tract infection


Urinary tract infection (UTI) is infection of any part of the urinary tract, usually by bacteria.
Commonest uropathogen is Escherichia coli in ~80% of adult cases.
UTI in men is generally uncommon, but incidence rates are higher in elderly men and those with risk factors, such as BPH,
catheterisation, previous urinary tract surgery or immunocompromised state.
A lower UTI should be suspected if the man has:
Dysuria, frequency, urgency, nocturia, & suprapubic pain.
Suprapubic tenderness, odorous or cloudy urine, haematuria.
UTI can present with atypical symptoms in men who are frail and elderly, in institutional care, or who have an indwelling
urinary catheter.
Diagnosis of a UTI should be confirmed by obtaining a urine sample for culture & sensitivity before starting antibiotic
treatment. Urine dipstick tests or microscopy should not be relied on to confirm the diagnosis.
Management of suspected lower UTI in men involves:
Arranging hospital admission if symptoms are severe (e.g. nausea & vomiting, confusion, tachypnoea, tachycardia, or
hypotension).
Starting empirical antibiotic treatment with trimethoprim or nitrofurantoin for 7 days for men who are not
catheterised.
Starting empirical antibiotic treatment with trimethoprim, nitrofurantoin or amoxicillin for 7 days for men who have
an indwelling urinary catheter, or pivmecillinam if first-line antibiotics are unsuitable.
Referral for urological assessment should be offered to men if they:
Have ongoing symptoms despite appropriate antibiotic treatment.
May have an underlying cause or risk factor for the UTI.
Have recurrent episodes of UTI (e.g. ≥2 episodes in a 6-month period).
People suspected of having urological cancer should be referred urgently using a suspected cancer pathway for an
appointment within 2 weeks.

Benign prostatic hyperplasia & Benign prostatic enlargement


Benign prostatic hyperplasia (BPH) is where there are hyperplastic changes in the prostate. Benign prostatic enlargement
(BPE) describes enlargement of the prostate gland by BPH.
~25–50% with BPH will have lower urinary tract symptoms (LUTS).
LUTS is predominantly due to 2 components: a static component related to an ↑ in benign prostatic tissue
narrowing the urethral lumen and a dynamic component related to an ↑ in prostatic smooth muscle tone mediated
by alpha-adrenergic receptors. Symptoms may also be contributed by bladder over-activity.
Patients present with storage symptoms (frequency, urgency, nocturia, & incontinence) &/or voiding symptoms (weak
stream, dribbling, dysuria, straining).
Evaluation includes history and examination including an abdominal examination for a palpable bladder and digital rectal
exam (DRE).
DRE may demonstrate enlarged prostate volume.
Prostate nodules or asymmetry is more consistent with prostate cancer.
Tenderness is suspicious for prostatitis.
Consider urinalysis, prostate-specific antigen (PSA) level, and International Prostate Symptom Scoring (IPSS) in
the appropriate patient groups.
If symptoms or disease severity warrant, therapy can be initiated with an alpha-blocker, 5-alpha-reductase inhibitors,
combination therapy, or other agents depending on symptom profile.
Common complications are disease progression & urinary retention, which may require invasive therapy.
Failure of medical management or renal complications are indications for surgical intervention.

Acute prostatitis
This is a severe, potentially life-threatening bacterial infection of the prostate.
Urinary infection with pathogens may be caused by urethral instrumentation, trauma, bladder outflow obstruction, or
dissemination of infection from outside the urinary tract.
Should be suspected in a man who presents with signs & symptoms of:
UTI — dysuria, frequency, urgency.
Prostatitis — perineal, penile, or rectal pain; acute urinary retention, obstructive voiding symptoms; low back pain,
pain on ejaculation; tender, swollen, warm prostate (on gentle rectal examination).
Bacteraemia — rigors, arthralgia, or myalgia; fever, tachycardia.
Treatment involves a 2 week course of antibiotics typically ciprofloxacin or ofloxacin.
Urgent referral to a genito-urinary medicine (GUM) clinic should be arranged if an STI is identified.
Most will recover completely within 2 weeks.
Following recovery, NICE CKS advises referral for investigation to exclude structural abnormality of the urinary tract.

Prostate cancer
Almost all cancers of the prostate (95%) are adenocarcinomas.
Most common cancer in men & 2nd most common cause of cancer death in men in the UK (after lung cancer),
accounting for 13% of all cancer deaths.
Risk factors include:

https://www.mcqbank.co.uk/testhistory.php?view=1487811 35/120
04/09/2023, 11:44 mcqbank.co.uk/testhistory.php?view=1487811
Increasing age.
Black ethnicity.
Family history of prostate cancer.
Obesity or being overweight.
Most are indolent & grow slowly — a minority are aggressive & invade local structures or metastasize to remote tissues.
Prostate cancer should be suspected in men who have any of the following unexplained symptoms:
Lower back or bone pain.
Lethargy.
Erectile dysfunction.
Haematuria.
Anorexia/weight loss.
Lower urinary tract symptoms.
Assessment should include a digital rectal examination (DRE) and a prostate-specific antigen (PSA) test.
Urgent referral to a urological cancer specialist should be arranged if prostate cancer is suspected either because the
prostate is hard and nodular on DRE, or because the PSA level ≥3.0 nanogram/mL.
In secondary care, men may be offered prostate biopsy ± imaging.
Treatment options include:
Watchful waiting.
Active surveillance.
Radical treatments (such as prostatectomy, or radiotherapy).
Adjunctive & palliative treatments (such as hormonal treatment, or chemotherapy).

References
BMJ Best Practice - Normal pressure hydrocephalus
Radiopaedia - Normal pressure hydrocephalus
NICE Clinical Knowledge Summaries - Renal or ureteric colic - acute
Radiopaedia - Alzheimer disease
BMJ Best Practice - Parkinson's disease
NICE Clinical Knowledge Summaries - Urinary tract infection (lower) – men
BMJ Best Practice - Benign prostatic hyperplasia
NICE Clinical Knowledge Summaries - Prostatitis - acute
NICE Clinical Knowledge Summaries - Prostate cancer

Send us your feedback about this MCQ | Add question feedback to My Revision Notes | Rate this MCQ:

Single Best Answer Question

Maximum Mark: 1
Mark Awarded: 1
Time taken to answer: 0 minutes 14 seconds.
Main Question

Which SINGLE ONE of the following is the MOST common cause of a lower urinary tract infection (UTI) in women?
Answers

Your answer:
C. Escherichia coli

Correct Answer:
Your answer is correct!

All Answers:
A. Pseudomonas aeruginosa
B. Proteus mirabilis
C. Escherichia coli
D. Klebsiella pneumoniae
E. Staphylococcus saprophyticus

Feedback

Urinary tract infection (UTI) results from pathogenic organisms gaining access to the urinary tract and not being effectively
eliminated. Entry of bacteria into the urinary tract may be:
Direct, for example, from insertion of a catheter into the bladder, instrumentation, or surgery.
Via the blood stream (more likely in immunocompromised people).
Retrograde, ascending through the urethra into the bladder.

https://www.mcqbank.co.uk/testhistory.php?view=1487811 36/120
04/09/2023, 11:44 mcqbank.co.uk/testhistory.php?view=1487811
UTI is usually caused by bacteria originating from the gastrointestinal tract.
The spectrum of micro-organisms which cause UTI is similar in men and women. The most common causative micro-
organisms are:
Escherichia coli (80%)
Staphylococcus saprophyticus (4%)
Klebsiella pneumoniae (4%)
Proteus mirabilis (4%)
Less common micro-organisms causing UTI include:
Enterobacter species, Enterococcus species, Serratia marcescens, Pseudomonas species, and Staphylococcus
aureus.
Candida albicans — rare in the community, but may be seen in people with risk factors such as indwelling catheters,
or men who are immunocompromised.

[Reference: NICE Clinical Knowledge Summaries – Urinary tract infection (lower)]

Urinary tract infection (lower)

Background
Lower urinary tract infection (UTI) = infection of the bladder.
Cystitis is often used as a synonym for lower UTI (particularly for women), although technically it means 'bladder
inflammation' and there may be non-infectious causes such as radiation & chemical-induced cystitis.
Upper UTI is infection of the upper urinary tract i.e. ureters & kidneys (pyelonephritis).
UTI is very common in women & accounts ~2% of all GP consultations.
50% of women will be treated for a symptomatic UTI during their lifetime.
UTI is not common in men, but incidence is higher in elderly men (likely to have additional risk factors) or with
indwelling urinary catheters.
Usually caused by gut bacteria from GI tract.
Escherichia coli accounts for 80% & Staphylococcus saprophyticus 10% of UTIs. Proteus mirabilis is more common
in men or associated with renal tract abnormalities, particularly calculi.

Definitions

Significant bacteriuria = 105 colony-forming units per ml (CFU/mL).


Asymptomatic bacteriuria = significant bacteriuria without clinical infection.
NICE Antenatal Care guidelines state that asymptomatic bacteriuria should be screened for & treated in
pregnancy as associated with premature delivery & low birthweight.
Do not treat asymptomatic bacteriuria in patients with indwelling catheters.
Uncomplicated UTI = typical pathogens with normal urinary tract & kidney function, and no predisposing co-morbidities.
Complicated UTI = increased likelihood of complications such as persistent infection, treatment failure & recurrent
infection. Assoc with ≥1 risk factors:
Abnormal urinary tract (e.g. calculus, vesicoureteric reflux, neurogenic bladder, indwelling catheter, urinary
obstruction, recent instrumentation).
Virulent organism (e.g. Staphylococcus aureus).
Immunosuppression (e.g. poorly controlled diabetes).
Impaired renal function.
Recurrent UTI = Defined as ≥2 UTI in 6mths or ≥3 UTI in 12mths. Can be:
Relapse - same organism
Reinfection - different organism

Diagnosis
Typical features of UTI = dysuria, frequency, urgency, changes in urine appearance or consistency, nocturia,
suprapubic discomfort/tenderness.
Note typical features may be absent in the elderly with underlying cognitive impairment — can present with
delirium, lethargy, ↓ ability to carry out activities of daily living & anorexia.
Women
If uncomplicated UTI in <65y.o, urine dipstick can be used as a diagnostic aid — dipstick is unreliable in women
aged >65y & those who are catheterised.
If dipstick +ve for nitrite or leukocyte & RBC ⟹ UTI is likely.
Send urine for culture if previous antibiotic treatment has failed or possibility of antibiotic resistance.
If dipstick -ve for nitrite & +ve for leukocyte ⟹ UTI is possible.
Send urine for culture to confirm diagnosis.
If dipstick -ve for nitrite, leukocyte & RBC ⟹ UTI is unlikely.
No need to send urine culture — consider other diagnoses.
Send urine for culture if:
Pregnant.

https://www.mcqbank.co.uk/testhistory.php?view=1487811 37/120
04/09/2023, 11:44 mcqbank.co.uk/testhistory.php?view=1487811
>65y.
Persistent symptoms that do not resolve with antibiotics.
Recurrent UTI.
Urinary catheter in situ or recently catheterised.
Risk factors for resistance or complicated UTI such as GU tract abnormalities, renal impairment,
residence in a long-term care facility, hospitalisation for >7d in last 6m, recent travel to a country with ↑
resistance or previous resistant UTI.
Atypical symptoms.
Visible or non-visible haematuria.
Men
Confirm diagnosis with urine culture before starting empirical drug treatment.
If catheterised - only send urine sample if features of systemic infection.
Do not use urine dipstick or microscopy to diagnose UTI in men:
Who are not catheterized — dipsticks are poor at ruling out infection.
But, may be helpful in some situations, e.g. UTI is unlikely if mild or non-specific symptoms AND -
ve urine dipstick test (i.e. both nitrite & leukocytes -ve).
Note, presence of these markers does not rule in UTI, although +ve nitrite makes UTI more
likely (PPV 96%).
With an indwelling catheter — working diagnosis should be based on clinical judgement.
Aged >65y — dipsticks become more unreliable with increasing age >65y.

Differential diagnosis
Pyelonephritis - urinary symptoms associated with fever &/or loin pain.
Drug-induced cystitis - cyclophosphamide, allopurinol, danazol, or tiaprofenic acid.
Sexually transmitted Infections e.g. chlamydia trachomatis (pyuria without bacteriuria).
Dermatological conditions e.g. psoriasis, dermatitis, lichen sclerosis or lichen planus.
Spondyloarthropathies e.g. reactive arthritis or Bechet’s syndrome.
Malignancy.
Ovarian cancer may present with persistent urinary urgency &/or frequency.
Bladder, or renal cancer may present with haematuria (visible or non-visible).
In women – consider
Urethral syndrome (aka painful bladder syndrome, interstitial cystitis & trigonitis = symptoms of cystitis in
absence of UTI - symptoms relieved by voiding & aggravated by drinking alcohol or caffeinated drinks).
Atrophic vaginitis/urethritis (menopausal woman with vaginal discharge or itch, & pain during sexual
intercourse).
In men – consider
Acute prostatitis (fever, irritative urinary symptoms, perineal/suprapubic pain, pain on ejaculation or with bowel
movements, & exquisitely tender prostate on PR).
Epididymitis (scrotal pain & epididymis is oedematous & tender).

Empirical Treatment
Women
If mild symptoms with normal immunity, renal function & renal tract, treatment can be delayed if she wishes
to see if symptoms resolve, especially if -ve urine dip (nitrites & leucocytes) indicating low probability of UTI.
For all other women especially if pregnant prescribe treatment without delay.
1st Choice
Nitrofurantoin for 3d.
If eGFR ≥45.
c/i in G6PD deficiency & acute porphyria.
Trimethoprim for 3d.
BNF advises caution if predisposed to folate deficiency.
Renal impairment:
If eGFR 15–30 - use half normal dose after 3d.
If eGFR <15 - use half normal dose.
c/i in blood dyscrasias.
2nd Choice (if no improvement in symptoms when 1st-choice antibiotic is taken for at least 48h or if first-
choice is unsuitable).
Nitrofurantoin for 3d (if not used as 1st choice).
Pivmecillinam for 3d.
Fosfomycin single dose sachet.
Pregnant
Nitrofurantoin (avoid at term due to risk of neonatal haemolysis) for 7d.
2nd choice
Amoxicillin (only if culture results available & susceptible) for 7d.
Cefalexin for 7d.

https://www.mcqbank.co.uk/testhistory.php?view=1487811 38/120
04/09/2023, 11:44 mcqbank.co.uk/testhistory.php?view=1487811
BNF states that manufacturer advises avoiding trimethoprim during pregnancy. There is a
teratogenic risk in first trimester (folate antagonist).
Men
Start empirical antibiotic drug treatment with trimethoprim or nitrofurantoin for 7d.

Duration
3d course for women with:
Uncomplicated UTI.
UTI with haematuria.
Recurrent UTI.
5–10d course for women who have:
Abnormal urinary tract.
Immunosuppression.
Impaired renal function.
7d course if:
Pregnant.
Catheterised.
Change catheter before starting antibiotics if catheter has been insitu for >7d.
Consider withholding antibiotics until culture results available to guide treatment if mild symptoms with normal
immunity & renal function.
Male.

References
NICE Clinical Knowledge Summaries - Urinary tract infection (lower) – women
NICE Clinical Knowledge Summaries - Urinary tract infection (lower) – men
British National Formulary

Send us your feedback about this MCQ | Add question feedback to My Revision Notes | Rate this MCQ:

Single Best Answer Question

Maximum Mark: 1
Mark Awarded: 0
Time taken to answer: 7 minutes 2 seconds.
Main Question

Which of the following is NOT a treatment for erectile dysfunction in men?


Answers

Your answer:
A. Sildenafil

Correct Answer:
D. Surgical bone graft

All Answers:
A. Sildenafil
B. Penile prosthesis
C. Vacuum device
D. Surgical bone graft
E. Intraurethral alprostadil

Feedback

Erectile dysfunction (ED) is the inability to attain and maintain an erection sufficient for satisfactory sexual
performance.

ED shares risk factors with cardiovascular disease.


History gathering
The following features suggest psychogenic causes:
Sudden onset
Early collapse of erection
Self-stimulated or waking erections
Premature ejaculation or inability to ejaculate
Problems or changes in relationship
Major life events

https://www.mcqbank.co.uk/testhistory.php?view=1487811 39/120
04/09/2023, 11:44 mcqbank.co.uk/testhistory.php?view=1487811
Psychological problems
The following features suggest organic causes:
Gradual onset
Normal ejaculation
Normal libido (except hypogonadal men)
Risk factor in medical history (cardiovascular, endocrine or neurological)
Operations, radiotherapy, or trauma to pelvis or scrotum
Current drug recognised as associated with erectile dysfunction
Smoking, high alcohol consumption, use of recreational or body-building drugs
Management
First-line therapy for erectile dysfunction
Oral agents
Phosphodiesterase inhibitors (sildenafil, tadalafil, & vardenafil) improve the relaxation of smooth
muscle.
Efficacy of the drug is dependent on release of nitric oxide from the nerve terminals of the
cavernosal nerve.
Contra-indications
Patients receiving nitrates
Previous history of non-arteritic anterior ischaemic optic neuropathy
Hypotension
avoid if systolic blood pressure <90 mmHg
Recent stroke or myocardial infarction
Unstable angina
Patients in whom vasodilation or sexual activity are inadvisable
Side-effects
GI s/e
Dyspepsia
Nausea & vomiting
Headaches (including migraine)
Visual disturbances
blue discolouration
non-arteritic anterior ischaemic optic neuropathy has been reported—stop drug if
sudden visual impairment occurs)
Nasal congestion
Flushing
Vacuum devices
External cylinder fitted over the penis to allow air to be pumped out, resulting in engorgement of penis
with blood.
They work best when there is a motivated, interested & understanding partner.
They may be the treatment of choice in well-informed older patients and in those with
comorbidities precluding use of drugs or invasive methods.
Adverse events include pain, petechiae, bruising & numbness.
Second-line therapy for erectile dysfunction
Intraurethral alprostadil (prostaglandin E1) is inserted as a pellet into the urethral meatus & produces an
erection after about 15 minutes.
Intracavernosal alprostadil is given into the corpora cavernosa to produce an erection lasting <1 hour.
Penile pain is usually mild but a significant number of men stop using this method because of penile pain.
Third-line therapy for erectile dysfunction
Penile prosthesis
Semi-rigid, malleable or inflatable devices surgically inserted to produce an erect state.
Prostheses should be considered in patients whose impotence has an organic cause and who are
unwilling to consider, fail to respond to, or are unable to continue with medical treatment or external
devices.

Private or NHS prescription?


In Aug 2014, generic sildenafil (& apomorphine) was taken off the SLS list following a significant reduction in the price of
sildenafil after the expiry of patent protection for Viagra®, however restrictions continue to apply to branded Viagra®.
Other phosphodiesterase inhibitors (e.g. tadalafil, avanafil) & alprostadil can still only be prescribed by GPs on the NHS for
men who:
1. Have diabetes, MS, Parkinson's disease, poliomyelitis, prostate cancer, severe pelvic injury, single-gene
neurological disease, spina bifida, or spinal cord injury.
2. Are receiving dialysis for renal failure.
3. Have had radical pelvic surgery, prostatectomy, or a kidney transplant.
4. Were receiving Caverject®, Erecnos®, MUSE®, Viagra®, or Viridal® for erectile dysfunction, at the expense of the
NHS, on 14 September 1998.

https://www.mcqbank.co.uk/testhistory.php?view=1487811 40/120
04/09/2023, 11:44 mcqbank.co.uk/testhistory.php?view=1487811
5. Treatment should also be available from specialist services (under local agreement) when the condition is causing
severe distress:
Significant disruption to normal social and occupational activities.
A marked effect on mood, behaviour, social and environmental awareness.
A marked effect on interpersonal relationships.
If the patient does not fit these criteria, a private prescription can be issued.

Send us your feedback about this MCQ | Add question feedback to My Revision Notes | Rate this MCQ:

Single Best Answer Question

Maximum Mark: 1
Mark Awarded: 1
Time taken to answer: 1 minutes 37 seconds.
Main Question

Which one of the following statements is TRUE with respect to bladder carcinoma?
Answers

Your answer:
B. Associated with cigarette smoking.

Correct Answer:
Your answer is correct!

All Answers:
A. Most common pathology is squamous cell carcinoma.
B. Associated with cigarette smoking.
C. Associated with alcohol abuse.
D. Classically presents as painful haematuria.
E. Commoner in young females.

Feedback

The most common type of bladder cancer in the UK is urothelial carcinoma, formerly known as transitional cell
carcinoma (TCC).

The urothelium in the entire urinary tract may be involved, including the renal pelvis, ureter, bladder, & urethra.
Risk factors
1. About half of bladder cancers are caused by smoking.
2. Other risk factors include industrial exposure to aromatic amines in dyes, paints, solvents, leather dust, inks,
combustion products, rubber & textiles.
3. Environmental pollution, e.g. arsenic-contaminated wells, has been identified as a factor.
4. There may be a genetic predisposition.
5. Radiation to the pelvis and cyclophosphamide are risks.
6. There is a tenuous link between chronic infection and bladder cancer, e.g. HIV & herpes simplex
7. Coffee may increase the risk of bladder cancer by 20% but some studies have suggested otherwise.
8. Suggestions of risk from artificial sweeteners have not been substantiated & there is no increased risk in those who
dye their hair.
Presentation
Usually affects males in their 60’s.
90% present with painless visible haematuria, which is the classic presentation.
Consider all patients with visible haematuria to have bladder cancer until proven otherwise.
30% present irritative bladder symptoms such as dysuria, urgency, or frequency of urination.
Management
Diagnosis of bladder cancer is transurethral cystoscopy & urine cytology.
Urine cytology can be obtained in voided urine or at the time of the cystoscopy ("bladder washout").
Treatment varies depending on whether or not the tumour has invaded the muscle.
May involve intravesical immuno (BCG) or chemotherapy, transurethral resection of bladder tumour
(TURBT), radical cystectomy, external beam radiotherapy or adjuvant & neoadjuvant chemotherapy.

Send us your feedback about this MCQ | Add question feedback to My Revision Notes | Rate this MCQ:

https://www.mcqbank.co.uk/testhistory.php?view=1487811 41/120
04/09/2023, 11:44 mcqbank.co.uk/testhistory.php?view=1487811

Extended Matching Question

Maximum Mark: 3
Mark Awarded: 3
Time taken to answer: 2 minutes 35 seconds.
Main Question

For each patient below, choose the most likely tubulointerstitial disease.
Select ONE option only from the list.
Each option may be selected once, more than once or not at all.
Answers

Your Answer:
B. Alport syndrome
Correct Answer:
Your answer is correct!

All answers:
1. A 6-year-old boy who was deaf at birth presents with haematuria, lethargy,
A. Urate nephropathy
malaise and ankle swelling. Bloods reveal a urea of 25mmol/l and a creatinine
B. Alport syndrome
of 500μmol/l.
C. Immunoglobulin A nephropathy
D. Balkan nephropathy
E. Thin GBM disease
F. Analgesic nephropathy
G. Diabetic nephropathy
H. Radiation nephritis

Your Answer:
F. Analgesic nephropathy
Correct Answer:
Your answer is correct!

2. A 65-year-old woman who takes regular NSAIDs for arthritis of the knees All answers:
presents with symptoms & signs of chronic renal failure. Urinalysis shows a A. Urate nephropathy
sterile pyuria and blood tests reveal a urea of 30mmol/l and a creatinine of B. Alport syndrome
700μmol/l. C. Immunoglobulin A nephropathy
D. Balkan nephropathy
E. Thin GBM disease
F. Analgesic nephropathy
G. Diabetic nephropathy
H. Radiation nephritis

Your Answer:
D. Balkan nephropathy
Correct Answer:
Your answer is correct!

All answers:
3. A 40-year-old male immigrant from Eastern Europe who used to live along
A. Urate nephropathy
the River Danube presents with progressive renal impairment. He has an
B. Alport syndrome
unusual yellow pigmentation on his palms & soles.
C. Immunoglobulin A nephropathy
D. Balkan nephropathy
E. Thin GBM disease
F. Analgesic nephropathy
G. Diabetic nephropathy
H. Radiation nephritis

Feedback

Q1 - B

Alport syndrome is a X-linked or autosomal recessive inherited rare defect in the genes encoding the chains of type IV collagen
of the basement membranes.
Presents as sensorineural deafness, pyelonephritis, haematuria, renal failure.
Treatment is dialysis or renal transplant for end stage renal failure.

Q2 - F

https://www.mcqbank.co.uk/testhistory.php?view=1487811 42/120
04/09/2023, 11:44 mcqbank.co.uk/testhistory.php?view=1487811
Analgesic nephropathy is more common in elderly people because of the higher incidence of arthritic disorders.
It begins after several months of NSAID, paracetamol or phenacetin exposure.
A unique feature of allergic interstitial nephritis caused by NSAIDs is that patients may present with nephrotic syndrome.
In such patients, massive proteinuria with hypoalbuminaemia and oedema are present in addition to the typical features
of acute interstitial nephritis.
Sudden flank pain should prompt an U/S to exclude obstruction from a sloughed papilla.
There is also an increased risk of urothelial tumours.

Q3 - D

Balkan nephropathy was first identified among several small, discrete communities along the Danube River.
The symptoms are not typical for the common chronic nephritis:
Incidence only in adults (no children affected)
lack of high blood pressure
xanthochromia (presence of bilirubin) of palms and soles (Tanchev's sign)

early anaemia of hypochromous type


lack of proteinuria
slowly progressive renal failure
Molecular epidemiological studies provide a strong case that chronic dietary exposure to aristolochic acid, a principal
component of Aristolochia clematitis which grows as a weed in the wheat fields of the endemic regions is the cause.

Send us your feedback about this MCQ | Add question feedback to My Revision Notes | Rate this MCQ:

Extended Matching Question

Maximum Mark: 3
Mark Awarded: 2
Time taken to answer: 0 minutes 46 seconds.
Main Question

For each patient below with a scrotal lump, choose the most likely cause.
Select ONE option only from the list.
Each option may be selected once, more than once or not at all.
Answers

Your Answer:
E. Inguinal hernia
Correct Answer:
Your answer is correct!

All answers:
1. A 35-year-old man complains of a swelling in the right groin that descends
A. Ectopic testis
into the scrotum which can be reduced and controlled by pressure over the
B. Hydrocoele
deep inguinal ring.
C. Inguinal lymph node
D. Femoral hernia
E. Inguinal hernia
F. Testicular cancer
G. Epididymal cyst
H. Spermatocele

https://www.mcqbank.co.uk/testhistory.php?view=1487811 43/120
04/09/2023, 11:44 mcqbank.co.uk/testhistory.php?view=1487811

Your Answer:
B. Hydrocoele
Correct Answer:
H. Spermatocele

All answers:
2. A 40-year-old man has a smooth fluctuant swelling within the scrotum
A. Ectopic testis
separate to the testes which when aspirated revealed the presence of milky
B. Hydrocoele
fluid.
C. Inguinal lymph node
D. Femoral hernia
E. Inguinal hernia
F. Testicular cancer
G. Epididymal cyst
H. Spermatocele

Your Answer:
A. Ectopic testis
Correct Answer:
Your answer is correct!

All answers:
3. A 10-month-old infant is brought in to see his GP by his mum as she is
A. Ectopic testis
worried by a lump in his right inguinal region. The GP cannot palpate the
B. Hydrocoele
child’s right testicle.
C. Inguinal lymph node
D. Femoral hernia
E. Inguinal hernia
F. Testicular cancer
G. Epididymal cyst
H. Spermatocele

Feedback

Q1 - E

The 3 most likely differentials to consider here are inguinal or femoral hernia and an inguinal lymph node. An inguinal hernia
protrudes through a dilated deep inguinal ring or attenuated inguinal floor in the inguinal canal and an inguinal hernia bulge lies
more cephalad, in the line of the inguinal ligament between the anterior iliac spine and pubis compared with a femoral hernia
which is located below the inguinal ligament, lateral and inferior to the pubic tubercle. Femoral hernias are also more common in
slender, often older females (and are more prone to strangulation). An enlarged lymph node may be associated with a history of
trauma, infection or malignancy. It is firm, tender, and non-reducible, most often mimicking a femoral hernia.

An inguinal hernia is a protuberance of intra-abdominal tissue through the abdominal wall where it is weakened by
the presence of the inguinal canal.
It can be classified as indirect (originate lateral to the inferior epigastric artery) or direct (originate medial to the inferior
epigastric artery). Although, differentiating between indirect and direct inguinal hernias are common examination
questions, there is no clinical merit in distinguishing the two.

8x commoner in males and affects 1-3% of young children.


Risk factors
Infants: prematurity, male sex.
Adults: male sex, obesity, constipation, chronic cough, heavy lifting.
Presentation
Presents as swelling in the groin, dragging sensation ± pain.
Management
Ultrasound if there is doubt.

https://www.mcqbank.co.uk/testhistory.php?view=1487811 44/120
04/09/2023, 11:44 mcqbank.co.uk/testhistory.php?view=1487811
Inguinal hernias should be repaired unless there are specific contraindications.
Complications of incarceration, obstruction & strangulation are greater than those of operation.
In infants & children, only the hernial sac needs to be excised - herniotomy.
For other groups, the posterior inguinal wall must be repaired also - herniorrhaphy.

Q2 - H

The differentials here are hydrocoele, testicular cancer, epididymal cyst or spermatocele. Testicular cancer most commonly
presents as a hard, painless nodule on one testis. A hydrocele is a collection of fluid between the layers of the membrane (tunica
vaginalis) that surrounds the testis or along the spermatic cord. The main symptom is a painless, swollen scrotum on one or both
sides, which feels like a water-filled balloon. Epididymal cysts and spermatoceles on the other hand are soft, firm cystic structures
that can be palpated separately from the testis. If aspirated, spermatoceles yield a milky fluid.

A spermatocele is a cyst that contains spermatozoa.


They occur in the head of the epididymis or less commonly, in the spermatic cord.

Presentation
Clinically, they resemble epididymal cysts but contain milky, rather than clear, fluid.
There is no way clinically to differentiate between an epididymal cyst and a spermatocele but can be
differentiated as sperm are present in the milky fluid aspirate of a spermatocele.
Most are asymptomatic.
Management
Surgical excision may be required if the patient is troubled by the lesion but operation may cause obstruction to the
passage of sperm, and so, may be contraindicated if the patient wishes to remain fertile and has an abnormality on
the other side.

Q3 - A

An ectopic testis is one that has deviated from the normal path of descent after it has emerged through the superficial inguinal
ring.

The term cryptorchidism, from the Greek kryptos (hidden) & orchis (testicle), is used to describe an undescended testis or a
testis that is absent from the scrotum.

Cryptorchidism is the most common congenital anomaly of the male genitalia


Prevalence in the region of 8 per 1000 live births.
There is a higher incidence in premature babies (up to 25%).
Embryology
By the 7th month of gestation, the right & left testes should reach their respective inguinal canals & by 9 months,
each testis has usually reached the scrotum.

https://www.mcqbank.co.uk/testhistory.php?view=1487811 45/120
04/09/2023, 11:44 mcqbank.co.uk/testhistory.php?view=1487811

Absence may be due to:


1. Testicular agenesis (anorchia)
uncommon
2. Retractile testis
from an exaggerated cremasteric reflex
3. Ascending testis syndrome
a previously normal or a retractile testis can become high with a shortened spermatic cord that
prevents the testis from staying in the scrotum
4. Testicular maldescent
descent can be
Arrested - where descent is along the normal path but incomplete.
Ectopic - where descent deviates from the normal path and is most often found in the superficial
inguinal pouch.

Clinical features
Around 70% of all undescended testes are palpable.
It can be difficult to distinguish undescended testes from retractile testis.
Unilateral cryptorchidism is 4x more likely than bilateral and the right testis is more likely to be affected than the
left.
Diagnosis
Imaging or ultrasound does not add any benefit to differentiating between palpable & non-palpable testes.
Diagnostic laparoscopy is the only way to confirm or rule out an intra-abdominal, inguinal or absent/vanishing
testis (non-palpable testis).
However, an examination under anaesthetic should be carried out before laparoscopy as a previously non-
palpable testis may become palpable.
Management
If, by the age of one year, descent has not occurred, spontaneous descent is unlikely. Treatment should be initiated,
as there is also potential for histological deterioration and loss of testicular quality (may affect future fertility).
If there is unilateral undescended testis still present at 3 months then the child should be referred to an appropriate
paediatric surgeon, ideally before the age of 6 months.
The ideal management of cryptorchidism is a highly debated topic within the field of paediatric surgery.
Medical
Testicular descent is hormonally dependent so treatment with human chorionic gonadotrophin (hCG) or
gonadotrophin-releasing hormone (GnRH) has been used in some centres but NICE has found that
hormone therapy is ineffective and therefore not recommended.

https://www.mcqbank.co.uk/testhistory.php?view=1487811 46/120
04/09/2023, 11:44 mcqbank.co.uk/testhistory.php?view=1487811
Surgical
Orchiopexy before 10-11 years may protect against the ↑ risk of testicular cancer associated with
cryptorchidism.
Orchiopexy should not be performed before 6 months of age, as testes may descend spontaneously
during the first few months of life.
Treatment should be completed by 12-18 months of age.
Complications
↑ risk of testicular torsion (if intraabdominal testis will present as acute abdomen)
Impaired fertility
20x increased risk of developing testicular malignancy

Reference
Patient.co.uk Professional Reference - Undescended and Maldescended Testes

Send us your feedback about this MCQ | Add question feedback to My Revision Notes | Rate this MCQ:

Single Best Answer Question

Maximum Mark: 1
Mark Awarded: 1
Time taken to answer: 0 minutes 19 seconds.
Main Question

Which ONE of the following is a risk factor for developing a testicular tumour?
Answers

Your answer:
C. Undescended testis

Correct Answer:
Your answer is correct!

All Answers:
A. Tight trousers
B. Binge drinking
C. Undescended testis
D. Gynaecomastia
E. Trauma

Feedback

Testicular tumours are the most common malignancy in young men.

Incidence is 7.5:100,000 men in the UK per year.


The peak incidence for teratomas is 25 years and seminomas is 35 years.
Classification
Testicular cancer is classified into 3 main types
1. Germ cell tumours
95% of testicular tumours are germ cell tumours of which 45% are seminomas & 50% are non-
seminomas (i.e. teratoma).
2. Non–germ cell tumours
e.g. Leydig's cell tumours, Sertoli's cell tumours & sarcomas
3. Extragonadal tumours
Risk factors
cryptorchidism
malignancy in the contralateral testis
Klinefelter's syndrome
family history
male infertility
low birth weight
young maternal age
young paternal age
multiparity
breech delivery
infantile hernia
testicular microlithiasis - small intratesticular calcifications seen on ultrasound

https://www.mcqbank.co.uk/testhistory.php?view=1487811 47/120
04/09/2023, 11:44 mcqbank.co.uk/testhistory.php?view=1487811
Presentation
Usually presents as a painless lump but is painful in 30%.
Some may also report a dragging sensation.
In 10% there is a recent history of trauma but it is probably the trauma that leads the man to examine himself
and find the tumour rather than being the cause of malignant change.
Other presentations include
hydrocoele
gynaecomastia from βhCG production
effects of distant metastases:
Seminomas metastasise to para-aortic nodes and produce back pain
Teratomas undergo blood-borne spread to liver, lung, bone & brain.
Diagnosis
Diagnosis is usually confirmed by ultrasound.
Alpha-fetoprotein (αFP) is produced by yolk sac elements and is raised in teratomas.
Beta-human chorionic gonadotrophin (βhCG) is produced by trophoblastic elements and is raised in both
teratomas & seminomas.
Management
Management is dependent on type of tumour & stage
Removal of primary tumour by inguinal orchidectomy +/- radiotherapy (seminomas are radiosensitive, teratomas
are not radiosensitive) +/- chemo for advanced disease.
~90% of patients classified as having a good prognosis achieve a durable complete remission with treatment.
Even metastatic disease should be seen as potentially curable.

Send us your feedback about this MCQ | Add question feedback to My Revision Notes | Rate this MCQ:

Single Best Answer Question

Maximum Mark: 1
Mark Awarded: 1
Time taken to answer: 4 minutes 12 seconds.
Main Question

A 35-year-old football player is struck in the groin by a football. A few days later he examines himself in the shower and finds a
painless lump in his scrotum. When you examine him you find that there is a hard painless lump that is not separable from the
left testicle and is not transilluminable nor does it disappear when the patient is lying supine. Which of the following is the MOST
likely diagnosis?
Answers

Your answer:
D. Testicular tumour

Correct Answer:
Your answer is correct!

All Answers:
A. Haematoma
B. Testicular necrosis
C. Epididymal cyst
D. Testicular tumour
E. Post-traumatic testicular microlithiasis

Feedback

Scrotal haematomas are typically associated with pain & swelling.

Testicular necrosis is usually the result of torsion of the spermatic cord.

Epididymal cysts are extratesticular, fluctuant, cystic swellings which transilluminate and are readily palpable separate from the
body of the testis.

Testicular microlithiasis are small (1-2mm) intratesticular calcifications which are clinically asymptomatic and are usually seen
incidentally on ultrasound. There seems to be an increased association with germ cell tumours particularly seminomas.

Testicular tumours are the most common malignancy in young men.

https://www.mcqbank.co.uk/testhistory.php?view=1487811 48/120
04/09/2023, 11:44 mcqbank.co.uk/testhistory.php?view=1487811
Incidence is 7.5:100,000 men in the UK per year.
The peak incidence for teratomas is 25 years and seminomas is 35 years.
Classification
Testicular cancer is classified into 3 main types
1. Germ cell tumours
95% of testicular tumours are germ cell tumours of which 45% are seminomas & 50% are non-
seminomas (i.e. teratoma).
2. Non–germ cell tumours
e.g. Leydig's cell tumours, Sertoli's cell tumours & sarcomas
3. Extragonadal tumours
Risk factors
cryptorchidism
malignancy in the contralateral testis
Klinefelter's syndrome
family history
male infertility
low birth weight
young maternal age
young paternal age
multiparity
breech delivery
infantile hernia
testicular microlithiasis - small intratesticular calcifications seen on ultrasound
Presentation
Usually presents as a painless lump but is painful in 30%.
Some may also report a dragging sensation.
In 10% there is a recent history of trauma but it is probably the trauma that leads the man to examine himself
and find the tumour rather than being the cause of malignant change.
Other presentations include
hydrocoele
gynaecomastia from βhCG production
effects of distant metastases:
Seminomas metastasise to para-aortic nodes and produce back pain
Teratomas undergo blood-borne spread to liver, lung, bone & brain.
Diagnosis
Diagnosis is usually confirmed by ultrasound.
Alpha-fetoprotein (αFP) is produced by yolk sac elements and is raised in teratomas.
Beta-human chorionic gonadotrophin (βhCG) is produced by trophoblastic elements and is raised in
both teratomas & seminomas.
Management
Management is dependent on type of tumour & stage
Removal of primary tumour by inguinal orchidectomy +/- radiotherapy (seminomas are radiosensitive, teratomas
are not radiosensitive) +/- chemo for advanced disease.
~90% of patients classified as having a good prognosis achieve a durable complete remission with treatment.
Even metastatic disease should be seen as potentially curable.

Send us your feedback about this MCQ | Add question feedback to My Revision Notes | Rate this MCQ:

Single Best Answer Question

Maximum Mark: 1
Mark Awarded: 1
Time taken to answer: 0 minutes 20 seconds.
Main Question

Which one of the following is NOT a cause of a raised Prostate Specific Antigen (PSA) level?
Answers

Your answer:
E. Priapism

https://www.mcqbank.co.uk/testhistory.php?view=1487811 49/120
04/09/2023, 11:44 mcqbank.co.uk/testhistory.php?view=1487811
Correct Answer:
Your answer is correct!

All Answers:
A. Old age
B. Acute urinary retention
C. Urinary catheterisation
D. Prostatitis
E. Priapism

Feedback

Prostate-specific antigen (PSA) is an enzyme produced by normal prostate cells that participates in the dissolution of the
seminal fluid coagulum and plays an important role in fertility.

PSA evaluation was never intended to serve as a diagnostic test for prostate cancer but is useful in helping to identify men
in whom a prostate biopsy would be appropriate.
The PSA level is a good marker for prostate volume.
Causes of a high PSA concentration
old age
acute urinary retention
urinary catheterisation
prostatitis
prostate carcinoma
transurethral resection of the prostate
benign prostatic hyperplasia
There is guidance as to the level of PSA test that indicates further investigation is required (NHS Cancer screening
programmes).
This reference range takes into account the patient’s age:
50-59 yrs ≥3.0ng/ml
60-69 yrs ≥4.0ng/ml
≥70 yrs >5.0ng/ml
Before undergoing a PSA test, men should
not have an active urinary infection.
not have ejaculated in the past 48 hours.
not have exercised vigorously in the past 48 hours.
not have had a prostate biopsy in the past 6 weeks.
The PSA test should be carried out before a digital rectal examination if possible. If this is not possible then some
advocate delaying the PSA test by one week after the digital rectal examination (controversial).

Send us your feedback about this MCQ | Add question feedback to My Revision Notes | Rate this MCQ:

Single Best Answer Question

Maximum Mark: 1
Mark Awarded: 1
Time taken to answer: 14 minutes 17 seconds.
Main Question

Which one of the following is NOT a risk factor for the progression of chronic kidney disease (CKD)?
Answers

Your answer:
B. Caucasian race

Correct Answer:
Your answer is correct!

All Answers:
A. Hypertension
B. Caucasian race
C. Chronic NSAID use
D. Diabetes
E. Cardiovascular disease

Feedback

https://www.mcqbank.co.uk/testhistory.php?view=1487811 50/120
04/09/2023, 11:44 mcqbank.co.uk/testhistory.php?view=1487811

Chronic kidney disease (CKD)

Defined as abnormalities of kidney function or structure present for >3 months, with implications for health.

This includes all people with markers of kidney damage and those with a GFR<60 ml/min/1.73m2 on ≥2
occasions separated by a period of ≥90 days.
Common causes of chronic kidney disease
diabetic nephropathy
chronic glomerulonephritis
chronic pyelonephritis
hypertension
adult polycystic kidney disease
Revised classification of CKD
NICE in 2014 advised that CKD should be classified according to estimated GFR (eGFR) & albumin:creatinine
ratio (ACR), using 'G' to denote the GFR category (G1–G5, which have the same GFR thresholds as the previous
CKD stages 1–5) and 'A' for the ACR category (A1–A3).

A person with an eGFR of 25ml/min/1.73m2 and an ACR of 15mg/mmol has CKD G4A2.

A person with an eGFR of 50ml/min/1.73m2 and an ACR of 35mg/mmol has CKD G3aA3.

GFR eGFR
Terms
category (ml/min/1.73m2)
G1 >90 Normal or high
Mildly decreased (relative to young
G2 60–89
adult level)
G3a 45–59 Mildly to moderately decreased
G3b 30–44 Moderately to severely decreased
G4 15–29 Severely decreased
G5 <15 Kidney failure
ACR
ACR (mg/mmol) Terms
category
A1 <3 Normal to mildly increased
Moderately increased (relative to
A2 3–30
young adult level)
Severely increased (Including
A3 >30 nephrotic syndrome where ACR usually
>220mg/mmol)
The following are risk factors for the progression of CKD:
Hypertension
Cardiovascular disease
Smoking
Chronic use of NSAIDs
Proteinuria
Diabetes
Black/Asian ethnicity
Urinary outflow obstruction
General Management
Avoidance of nephrotoxins, e.g. IV radiocontrast agents, NSAIDs, aminoglycosides.
Cardiovascular prophylaxis
For patients with 10-year risk of cardiovascular disease >20%, consider aspirin treatment (if blood pressure
<150/90 mm Hg) and lipid-lowering drug therapy.
Blood pressure monitoring: blood pressure should be measured at least annually.
Control of hypertension
Hypertension should be tightly controlled
The majority of patients with CKD will require more than two drugs to treat hypertension.
ACE inhibitors are first line and are particularly helpful in proteinuric renal disease (e.g. diabetic
nephropathy).
As these drugs tend to reduce filtration pressure a small fall in GFR and rise in creatinine
can be expected.
Furosemide is useful as an anti-hypertensive in patients with CKD, particularly when the GFR falls
to below 30-45 ml/min as it has the added benefit of lowering serum potassium.
T2DM
Add SGLT2 inhibitor (e.g. dapagliflozin) for T2DM patients who have albuminuria (ACR >30mg/mmol) despite
taking max doses of ARB or ACE inhibitor.

Reference
NICE Clinical Knowledge Summaries - Chronic kidney disease

https://www.mcqbank.co.uk/testhistory.php?view=1487811 51/120
04/09/2023, 11:44 mcqbank.co.uk/testhistory.php?view=1487811

Send us your feedback about this MCQ | Add question feedback to My Revision Notes | Rate this MCQ:

Single Best Answer Question

Maximum Mark: 1
Mark Awarded: 1
Time taken to answer: 0 minutes 31 seconds.
Main Question

A 30-year-old man presents to his GP with a first presentation of suspected acute pyelonephritis. According to national guidance,
which SINGLE ONE of the following is the most appropriate FIRST-LINE treatment option in the community?
Answers

Your answer:
A. Cefalexin

Correct Answer:
Your answer is correct!

All Answers:
A. Cefalexin
B. Metronidazole
C. Clindamycin
D. Doxycycline
E. Nitrofurantoin

Feedback

Patients with acute pyelonephritis should be treated in hospital if they seem unwell as they may have severe pyelonephritis, a
urinary obstruction, or complications of pyelonephritis. Patients with acute pyelonephritis that is less severe can be treated in the
in the community with oral antibiotics. Consider also the patient’s ability to cope at home when deciding to treat in the
community.

According to NICE guidance, for non-pregnant women, men, & those with indwelling catheters, the following are first line:
Cefalexin for 7-10 days
Co-amoxiclav (only if appropriate in line with culture and sensitivity results) for 7-10 days
Trimethoprim (only if appropriate in line with culture and sensitivity results) for 14 days
Ciprofloxacin (consider safety issues) for 7 days

When urine culture results are available, the empirical antibiotic should be reviewed and changed according to susceptibility
results if the bacteria are resistant. A narrow-spectrum antibiotic should be used where possible. If symptoms worsen, or do not
start to improve within 48 hours of starting the antibiotic, other possible diagnoses should be considered and any signs or
symptoms suggesting a more serious condition, such as sepsis should be assessed.

[Reference – BMJ Best Practice]

Acute Pyelonephritis (Upper Urinary Tract Infection)

Background
= infection of renal pelvis ± infection of parenchyma.
Usually caused by bacteria ascending from lower urinary tract. Most common causative pathogens are gram-negative
bacteria:
Escherichia coli (70%)
Klebsiella species (20%)
Proteus mirablis
Pseudomonas species
Enterobacter species

Clinical features
Flank/renal angle pain &/or tenderness.
Myalgia.
Flu-like symptoms.
Rigors or ↑ temperature of ≥37.9°C (or <36°C in people aged >65y).
Most people have fever, although may be absent early in people with early or mild cases, frail, older people, or in
immunocompromised.

https://www.mcqbank.co.uk/testhistory.php?view=1487811 52/120
04/09/2023, 11:44 mcqbank.co.uk/testhistory.php?view=1487811
Nausea/vomiting.

Diagnosis
In all patients suspected of acute pyelonephritis, arrange collection of urine sample to determine infecting micro-
organism.
Dipstick testing of urine for nitrites & leukocytes is not necessary, however may be useful adjunct to guide diagnosis. It
should not be used in people:
With indwelling catheter — make working diagnosis based on clinical judgement.
Aged >65 years — dipsticks become more unreliable with increasing age.
There are no clinical features or routine investigations that conclusively distinguish acute pyelonephritis from
lower urinary tract infection.
However, the triad of flank pain (typically unilateral), fever, and nausea & vomiting occurs much more
often with pyelonephritis than with lower UTI.
Definitive diagnosis is made in those with loin pain &/or fever if BOTH of the following:
UTI confirmed on culture.
Other causes excluded e.g. pelvic inflammatory disease, appendicitis or renal calculi, etc.

Management in primary care


Admit to hospital if severe acute pyelonephritis or more serious condition (e.g. sepsis).
Consider same-day referral or seeking specialist advice:
Significantly dehydrated or unable to take oral fluids/medicines.
Pregnant.
Higher risk of developing complications — e.g. abnormality of genitourinary tract or underlying disease
(e.g. diabetes or immunosuppression).
Recurrent episodes of UTI (i.e. ≥2 episodes in 6-month period).
Offer an antibiotic for those who do not require admission i.e. low risk of complications.
Obtain urine sample for culture before starting empirical antibiotics & adjust when sensitivities are back
For non-pregnant women, men, & those with indwelling catheters, the following are first line:
Cefalexin for 7-10d.
Co-amoxiclav (if in line with sensitivity results) for 7-10d.
Trimethoprim (if in line with sensitivity results) for 14d.
Ciprofloxacin (consider safety issues) for 7d.
For pregnant women who do not require admission:
Cefalexin for 7-10d.
Indwelling urinary catheters:
Check catheter is draining correctly.
Consider removing or changing catheter ASAP if in situ for >7d.
Do not allow catheter removal or change to delay antibiotic treatment.
Do not give antibiotic prophylaxis for catheter changes unless history of symptomatic UTIs due to catheter
change.
Advise
Seek medical help if symptoms worsen, do not start to improve within 48h of taking antibiotic, or if becomes very
unwell.
Use paracetamol or ibuprofen for pain if required.
Drink sufficient fluids to avoid dehydration.
Review culture & sensitivity results when available, and change antibiotic if indicated.
Consider referral for investigation of underlying renal tract abnormality:
Men, following single episode without obvious cause.
Women with recurrent pyelonephritis.

Complications
Renal impairment/failure - due to scarring of renal parenchyma.
Septicaemia
Preterm labour

Reference
NICE Clinical Knowledge Summaries – Acute Pyelonephritis

Send us your feedback about this MCQ | Add question feedback to My Revision Notes | Rate this MCQ:

https://www.mcqbank.co.uk/testhistory.php?view=1487811 53/120
04/09/2023, 11:44 mcqbank.co.uk/testhistory.php?view=1487811

Single Best Answer Question

Maximum Mark: 1
Mark Awarded: 0
Time taken to answer: 2 minutes 25 seconds.
Main Question

A 30-year-old man taking lithium for bipolar disorder was made NBM for an emergency operation to repair a wrist laceration.
Whilst waiting, he began to show signs of altered mental status which the nursing staff attributed to his bipolar disorder. However,
he then started having seizures at which point the on call medical team was summoned immediately. Urgent blood tests revealed
a serum sodium of 175mmol/l (normal 135-145) and a urine osmolality of 160mOsm/kg (normal 300-900). His seizures were
stabilised, and he was fluid resuscitated with 5% glucose solution. Which of the following would be the NEXT most appropriate
treatment?
Answers

Your answer:
C. Hypertonic saline fluid

Correct Answer:
D. Thiazide diuretic

All Answers:
A. Restart his lithium
B. Oral rehydration
C. Hypertonic saline fluid
D. Thiazide diuretic
E. Demeclocycline

Feedback

The patient is hypernatremic with a low urine osmolality (<300mOsm/kg) (and if measured a high serum osmolarity) which is
typical for a patient with Nephrogenic Diabetes Insipidus. There are congenital and acquired causes (hypokalaemia,
hypercalcaemia, CKD and drugs such as lithium & demeclocycline). This patient’s nephrogenic DI secondary to his lithium
treatment for his bipolar condition was unmasked by his lack of access to drinking water.

Management Principles:
1. Stop any drugs that may be causing the problem.
2. Mild cases need no special treatment apart from correcting deficits in extracellular fluid volume by maintaining adequate
water intake (orally).
3. More severe cases may need treatment with
Large doses desmopressin (DDAVP) - ADH analogue.
Thiazide diuretics - inhibits reabsorption of NaCl in the distal renal tubule.
NSAIDs (e.g. indomethacin) may be effective in reducing the volume of urine produced.
Sodium restriction

Diabetes insipidus (DI) is a condition characterized by excessive thirst & excretion of large amounts of severely diluted
urine.

The most common type is neurogenic DI, caused by a deficiency of antidiuretic hormone (ADH).
The second common type of DI is nephrogenic DI, which is caused by an insensitivity of the kidneys to ADH.
Polyuria, polydipsia, & nocturia (from 3-18 L) are the predominant symptoms.
A urine specific gravity of 1.005 or less and a urine osmolality less than 200 mOsm/kg is the hallmark of
diabetes insipidus. Random plasma osmolality is generally >287 mOsm/kg.
The water deprivation test can differentiate between polydipsia & DI and also differentiate cranial and nephrogenic DI:

https://www.mcqbank.co.uk/testhistory.php?view=1487811 54/120
04/09/2023, 11:44 mcqbank.co.uk/testhistory.php?view=1487811

Send us your feedback about this MCQ | Add question feedback to My Revision Notes | Rate this MCQ:

Single Best Answer Question

Maximum Mark: 1
Mark Awarded: 0
Time taken to answer: 0 minutes 35 seconds.
Main Question

Which one of the following is a cause of hyperkalaemia?


Answers

Your answer:
D. Alkalosis

Correct Answer:
B. Exercise

All Answers:
A. Decreased serum osmolarity
B. Exercise
C. β-agonists
D. Alkalosis
E. Insulin injection

Feedback

Exercise causes a shift of K+ from cells into blood resulting in hyperkalaemia.

Hypo-osmolarity, insulin, β-agonists & alkalosis cause a shift in K+ from blood into cells resulting in hypokalaemia.

Send us your feedback about this MCQ | Add question feedback to My Revision Notes | Rate this MCQ:

Extended Matching Question

Maximum Mark: 3
Mark Awarded: 1
Time taken to answer: 2 minutes 6 seconds.
Main Question

For each patient below with a scrotal swelling, choose the most likely cause.
Select ONE option only from the list.

https://www.mcqbank.co.uk/testhistory.php?view=1487811 55/120
04/09/2023, 11:44 mcqbank.co.uk/testhistory.php?view=1487811
Each option may be selected once, more than once or not at all.
Answers

Your Answer:
H. Hydrocele
Correct Answer:
E. Epididymal cyst

All answers:
1. A 40-year-old man presents with a transilluminable lump in the scrotum A. Testicular torsion
distinct from the testes. B. Testicular tumour
C. Varicocoele
D. Inguinal hernia
E. Epididymal cyst
F. Epididymal TB
G. Epididymitis
H. Hydrocele

Your Answer:
E. Epididymal cyst
Correct Answer:
H. Hydrocele

All answers:
2. A 30-year-old man presents with a firm lump in the scrotum. The testis A. Testicular torsion
cannot be felt but the lump transilluminates very well. B. Testicular tumour
C. Varicocoele
D. Inguinal hernia
E. Epididymal cyst
F. Epididymal TB
G. Epididymitis
H. Hydrocele

Your Answer:
C. Varicocoele
Correct Answer:
Your answer is correct!

All answers:
3. A 25-year-old man presents with a scrotal swelling that feels like a “bag of A. Testicular torsion
worms” when he is standing. B. Testicular tumour
C. Varicocoele
D. Inguinal hernia
E. Epididymal cyst
F. Epididymal TB
G. Epididymitis
H. Hydrocele

Feedback

Q1 - E

Epididymal cysts are smooth, extra-testicular, spherical cysts in the head of the epididymis that are not uncommon in
adult men.
Presentation
They usually develop around the age of 40.
Often they are multiple and may be bilateral.
As they are cystic and fluid filled they are well defined, fluctuant & will transilluminate.
As they arise in the epididymis, the testis is palpable quite separately from the cyst.

https://www.mcqbank.co.uk/testhistory.php?view=1487811 56/120
04/09/2023, 11:44 mcqbank.co.uk/testhistory.php?view=1487811

Unlike a hydrocele where the testis is palpable within the fluid filled swelling.
Management
Treatment is not usually necessary and explanation with advice is all that is usually required.
Patients should be advised to seek medical advice if epididymal cysts become painful or suddenly start
increasing in size.
They can be treated sclerotherapy or by surgical excision.

Q2 - H

Hydrocele is a fluid collection within the tunica vaginalis of the scrotum or along the spermatic cord.

Prevalence is 1% of adult men.


Pathology
Three types:
Communicating or congenital hydroceles have a patent processus vaginalis permitting flow of peritoneal
fluid into the scrotum.
Non-communicating hydroceles have a patent processus vaginalis but no communication with the
peritoneal cavity occurs.
Hydrocele of the cord occurs when closure of the tunica vaginalis is defective.
>80% of newborn boys have a patent processus vaginalis, but most close spontaneously within 18 months of age.
Aetiology
Most paediatric hydroceles are congenital, but rarer causes include malignancy & infection.
Hydroceles in adults may be secondary to
orchitis
epididymitis
tuberculosis
torsion
testicular tumours
trauma
following renal transplantation
post-radiation therapy
Presentation
2 peak ages of presentation (1-2year & >40years).
The usual presentation is a painless enlarged scrotum with a sensation of heaviness or dragging.
Hydroceles are located superior & anterior to the testis, in contrast to spermatoceles, which lie
superior & posterior to the testis.

https://www.mcqbank.co.uk/testhistory.php?view=1487811 57/120
04/09/2023, 11:44 mcqbank.co.uk/testhistory.php?view=1487811

Management
Investigations are usually not required but ultrasound can help to determine if any underlying pathology is present.
Many newborn hydroceles resolve before the age of 1 year and so observation is usually appropriate.
Once underlying pathology has been excluded, adults with non-communicating hydroceles can be managed
conservatively with a scrotal support.
Surgery may be indicated if the hydrocele is large & uncomfortable, becomes infected or repeatedly
reaccumulates after aspiration.

Q3 - C

A varicocoele is an abnormal dilatation of the testicular veins in the pampiniform venous plexus caused by venous
reflux.
They are important because they are a well- recognised cause of reduced testicular function and are associated
with male infertility.
It is unusual in boys under age 10 years & incidence ↑ after puberty.
Presentation
They are usually asymptomatic & only rarely cause pain.
The scrotum is often described as feeling 'like a bag of worms'.

A varicocele cannot usually be palpated lying down.


Dilation & tortuosity of the veins increases with standing and usually decreases on lying down.
Most are in the left testicle (80-90%).
The scrotum on the side of a varicocoele hangs lower than on the normal side.
Management
Doppler studies is the diagnostic method of choice but is not indicated unless physical examination is inconclusive.
May require surgery if symptomatic.

Send us your feedback about this MCQ | Add question feedback to My Revision Notes | Rate this MCQ:

https://www.mcqbank.co.uk/testhistory.php?view=1487811 58/120
04/09/2023, 11:44 mcqbank.co.uk/testhistory.php?view=1487811

Single Best Answer Question

Maximum Mark: 1
Mark Awarded: 1
Time taken to answer: 0 minutes 21 seconds.
Main Question

What should you advise a patient prior to having their blood sampled for a Prostate Specific Antigen (PSA) test?
Answers

Your answer:
E. Not to ejaculate in the preceding 48 hours.

Correct Answer:
Your answer is correct!

All Answers:
A. Not to urinate an hour before the test.
B. Not to exercise vigorously in the preceding month.
C. Not to consume carbonated drinks on the day of the test.
D. Not to consume grapefruit juice in the preceding 24 hours.
E. Not to ejaculate in the preceding 48 hours.

Feedback

Prostate-specific antigen (PSA) is an enzyme produced by normal prostate cells that participates in the dissolution of the
seminal fluid coagulum and plays an important role in fertility.

PSA evaluation was never intended to serve as a diagnostic test for prostate cancer but is useful in helping to identify men
in whom a prostate biopsy would be appropriate.
The PSA level is a good marker for prostate volume.
Causes of a high PSA concentration
old age
acute urinary retention
urinary catheterisation
prostatitis
prostate carcinoma
transurethral resection of the prostate
benign prostatic hyperplasia
There is guidance as to the level of PSA test that indicates further investigation is required (NHS Cancer screening
programmes).
This reference range takes into account the patient’s age:
50-59 yrs ≥3.0ng/ml
60-69 yrs ≥4.0ng/ml
≥70 yrs >5.0ng/ml
Before undergoing a PSA test, men should
not have an active urinary infection.
not have ejaculated in the past 48 hours.
not have exercised vigorously in the past 48 hours.
not have had a prostate biopsy in the past 6 weeks.
The PSA test should be carried out before a digital rectal examination if possible. If this is not possible then some
advocate delaying the PSA test by one week after the digital rectal examination (controversial).

Send us your feedback about this MCQ | Add question feedback to My Revision Notes | Rate this MCQ:

Extended Matching Question

Maximum Mark: 3
Mark Awarded: 1
Time taken to answer: 3 minutes 33 seconds.
Main Question

For each common drug below, choose the most appropriate course of action in patients with chronic renal impairment.
Select ONE option only from the list.
Each option may be selected once, more than once or not at all.
Answers

https://www.mcqbank.co.uk/testhistory.php?view=1487811 59/120
04/09/2023, 11:44 mcqbank.co.uk/testhistory.php?view=1487811

Your Answer:
G. Monitor drug levels more often
Correct Answer:
A. No change required

All answers:
A. No change required
1. Phenytoin
B. Reduce dose size
C. Reduce dose frequency
D. Increase dose size
E. Increase dose frequency
F. Monitor drug levels
G. Monitor drug levels more often
H. Contraindicated

Your Answer:
D. Increase dose size
Correct Answer:
Your answer is correct!

All answers:
A. No change required
2. Furosemide
B. Reduce dose size
C. Reduce dose frequency
D. Increase dose size
E. Increase dose frequency
F. Monitor drug levels
G. Monitor drug levels more often
H. Contraindicated

Your Answer:
A. No change required
Correct Answer:
C. Reduce dose frequency

All answers:
A. No change required
3. Cefuroxime
B. Reduce dose size
C. Reduce dose frequency
D. Increase dose size
E. Increase dose frequency
F. Monitor drug levels
G. Monitor drug levels more often
H. Contraindicated

Feedback

Q1 - A

Phenytoin is metabolised in the liver and so is unaffected by renal disease.

Q2 - D

Furosemide acts on the Na-K-2Cl co-transporter on the ascending limb of the loop of Henle.

https://www.mcqbank.co.uk/testhistory.php?view=1487811 60/120
04/09/2023, 11:44 mcqbank.co.uk/testhistory.php?view=1487811

Hence higher doses will be needed if the number of functioning nephrons are reduced.
Some clinicians use high doses of loop diuretics to convert oliguric renal failure to non-oliguric renal failure to
facilitate fluid & electrolyte management and to reduce the need for dialysis.
However, be aware of ototoxicity at higher doses.

Q3 - C

Cephalosporins are excreted unchanged by the kidney and will accumulate in renal impairment so reduce dose frequency in
severe renal failure.

Send us your feedback about this MCQ | Add question feedback to My Revision Notes | Rate this MCQ:

Extended Matching Question

Maximum Mark: 2
Mark Awarded: 1
Time taken to answer: 2 minutes 32 seconds.
Main Question

For each patient below, choose the most likely diagnosis.


Select ONE option only from the list.
Each option may be selected once, more than once or not at all.
Answers

Your Answer:
A. Renal stone
Correct Answer:
Your answer is correct!

All answers:
1. A 40-year-old man complains of excruciating right flank pain. On A. Renal stone
examination he is writhing in agony and urinalysis reveals blood +++. B. Benign prostate hypertrophy
C. Prostate cancer
D. Urinary tract infection
E. Polycystic kidney disease
F. Pelvic inflammatory disease
G. Renal cell carcinoma
H. Bladder carcinoma

2. A 75-year-old man presents with acute urinary retention. Prostate exam Your Answer:
reveals an asymmetrically enlarged stony hard prostate. B. Benign prostate hypertrophy
Correct Answer:
C. Prostate cancer

All answers:

https://www.mcqbank.co.uk/testhistory.php?view=1487811 61/120
04/09/2023, 11:44 mcqbank.co.uk/testhistory.php?view=1487811
A. Renal stone
B. Benign prostate hypertrophy
C. Prostate cancer
D. Urinary tract infection
E. Polycystic kidney disease
F. Pelvic inflammatory disease
G. Renal cell carcinoma
H. Bladder carcinoma

Feedback

Q1 - A

A renal stone or calculus is a solid aggregation formed in the kidneys from dietary minerals in the urine.

Demographics
Annual incidence is 1:1000 & the average lifetime risk is 10%
Male to female ratio is 3:1
The peak age is between 30 - 50.
Pathology
Consist of crystal aggregates that form in the collecting ducts which can then be deposited anywhere from the renal
pelvis to the urethra.
calcium oxalate (75%)
magnesium ammonium phosphate [struvite] (10%)
urate (5%)
hydroxyapatite (5%)
cystine (1%)
Risk factors
Anatomical anomalies in the kidneys &/or urinary tract,
e.g. horseshoe kidney, ureteral stricture
Family history of renal stones
Hypertension
Gout
Hyperparathyroidism
Immobilisation
Relative dehydration
commoner in hot climates
Metabolic disorders
which increase excretion of solutes
e.g. chronic metabolic acidosis, hypercalciuria, hyperuricosuria
Deficiency of citrate in the urine
Cystinuria (an autosomal recessive aminoaciduria)
Drugs, especially thiazide diuretics
Presentation
Many stones are asymptomatic and discovered during investigations for other conditions.
Classic symptom is sudden, severe pain.
Pain starts in the loin about the level of the costovertebral angle & moves to the groin, with tenderness of the
loin or renal angle, sometimes with haematuria.
If the stone is high & distends the renal capsule then pain will be in the flank but as it moves down pain will
move anteriorly and down towards the groin.
Other symptoms which may be present include:
Rigors and fever
Dysuria
Haematuria
Urinary retention
Nausea and vomiting
Investigations
Urinalysis
The stone often causes some bleeding into the renal tract and this may produce a positive result for blood on
stick testing
a negative test does not exclude the diagnosis
Mid-stream specimen of urine for microscopy, culture & sensitivities.
Bloods
FBC, U&E, calcium & phosphate and urate.
Blood cultures & parathyroid hormone if indicated.
Encourage the patient to try to catch the stone for analysis.

https://www.mcqbank.co.uk/testhistory.php?view=1487811 62/120
04/09/2023, 11:44 mcqbank.co.uk/testhistory.php?view=1487811
This may mean urinating through a tea strainer, filter paper such as a coffee filter or a gauze.
Helical CT or CT KUB is the gold standard for the investigation of urinary stones (99% of stones visible) & has
superseded intravenous urography (IVU).
Plain X-rays (80% of stones are visible on a KUB plain film).
Renal ultrasound may also detect stones & reveal any evidence of obstruction (hydronephrosis or hydroureter)
Management
Most kidney stones do not require surgery and will pass on their own.
Surgery is necessary when the pain is persistent & severe, in renal failure and when there is a kidney
infection.
It may also be advisable if the stone fails to pass or move after 30 days.
Finding a significant stone before it passes into the ureter allows physicians to fragment it surgically before it causes
any severe problems.
In most of these cases, non-invasive extracorporeal shock wave lithotripsy (ESWL) will be used.
Otherwise some form of invasive procedure is required.
With approaches including
Ureteroscopic fragmentation (or simple basket extraction if feasible) using laser
Ultrasonic or mechanical (pneumatic, shock-wave) forms of energy to fragment the larger stones.
Percutaneous nephrolithotomy (PCNL) or rarely open surgery may ultimately be necessary for large or complicated
stones or stones which fail other less invasive attempts at treatment.
Prevention
A normal calcium intake is now recommended as low calcium diets increase oxalate excretion.
Advise to consume less oxalate by reducing tea, chocolate, nuts, strawberries, rhubarb, spinach, beans & beetroot &
to drink plenty of fluids especially in the summer.
Complications
Complete blockage of the urinary flow from a kidney decreases glomerular filtration rate (GFR) and, if it persists
for >48 hours, may cause irreversible renal damage.
If ureteric stones cause symptoms after 4 weeks, there is a 20% risk of complications, including deterioration of
renal function, sepsis, & ureteric stricture.
Infection can be life-threatening.
Persisting obstruction predisposes to pyelonephritis.

Q2 - C

Prostate cancer is the second leading cause of cancer-related death in the UK among men (after lung cancer).
The majority are well differentiated adenocarcinomas.
The tumour arises peripherally in the glandular tissue and grows slowly and commonly, remains asymptomatic.
Even when symptomatic, the disease pursues a long course - often, the patient dies from other causes.
Presentation
The most common age of presentation is between 65-85 years.
It is rare before 50 years.
Presentation may include
bladder outlet obstruction presenting as hesitancy or acute retention
haematuria
pain (back, perineal or testicular)
constipation
ureteric obstruction (anuria, uraemia)
features of metastatic disease
On rectal examination, the prostate may show asymmetrical, nodular enlargement and feel stony hard
and irregular with obliteration of the median sulcus.
Gleason grading system is most widely used.
Patients with a Gleason score of ≤4 do well clinically, whereas patients with a score of 8-9 do poorly.
Management
Traditionally, prostate cancer grades were described according to the Gleason Score, a system named for the
pathologist who developed it in the 1960s.
The cells are graded on a scale of 1 to 5 with grade 1 cells resembling normal prostate tissue and grade 5 cells
being highly differentiated.
The pathologist looking at the biopsy sample will assign one Gleason grade to the most predominant pattern in
the biopsy and a second Gleason grade to the second most predominant pattern.
Therefore, Gleason scores range from 2-10 - a Gleason score of 6 is low grade, 7 is intermediate grade,
and a score of 8 to 10 is high grade cancer.
Treatment for prostate cancer may involve
active surveillance
surgery
radiotherapy including brachytherapy (prostate brachytherapy) and external beam radiation therapy
High-Intensity Focused Ultrasound (HIFU)
chemotherapy
https://www.mcqbank.co.uk/testhistory.php?view=1487811 63/120
04/09/2023, 11:44 mcqbank.co.uk/testhistory.php?view=1487811
cryosurgery
hormonal therapy
Which option is best will depend on the stage of the disease, the Gleason score, and the PSA level.

Send us your feedback about this MCQ | Add question feedback to My Revision Notes | Rate this MCQ:

Extended Matching Question

Maximum Mark: 3
Mark Awarded: 3
Time taken to answer: 3 minutes 41 seconds.
Main Question

For each patient below presenting with renal symptoms, choose the most likely glomerular cause.
Select ONE option only from the list.
Each option may be selected once, more than once or not at all.
Answers

Your Answer:
B. Berger’s disease (IgA Nephropathy)
Correct Answer:
Your answer is correct!

All answers:
1. An otherwise healthy 22-year-old man presents with visible haematuria
A. Wegener's granulomatosis
accompanied by flank pain. He has a 2-day history of sore throat, fever, chills,
(granulomatosis with polyangiitis)
malaise, and headache. Physical examination reveals erythema and
B. Berger’s disease (IgA Nephropathy)
inflammation of the uvula and pharynx, enlarged tonsils with patchy greyish-
C. Churg-Strauss syndrome (eosinophilic
white exudates, and tender anterior cervical lymphadenopathy. The rest of the
granulomatosis with polyangiitis)
examination is normal. Urinalysis shows cola-coloured urine with haematuria
D. Henoch-Schönlein purpura (IgA
and 3+ protein.
vasculitis)
E. Systemic lupus erythematosus
F. Goodpasture's syndrome (anti-glomerular
basement membrane antibody disease)
G. Post-streptococcal glomerulonephritis
H. Alport's syndrome

Your Answer:
G. Post-streptococcal glomerulonephritis
Correct Answer:
Your answer is correct!

All answers:
A. Wegener's granulomatosis
2. A 10-year boy presents with swelling in the face, legs and scrotal area (granulomatosis with polyangiitis)
which developed 8 days after antibiotic treatment for tonsillitis. Urinalysis B. Berger’s disease (IgA Nephropathy)
reveals haematuria and proteinuria. Blood tests show elevated urea, C. Churg-Strauss syndrome (eosinophilic
creatinine, and antistreptolysin O (ASO) antibodies. granulomatosis with polyangiitis)
D. Henoch-Schönlein purpura (IgA
vasculitis)
E. Systemic lupus erythematosus
F. Goodpasture's syndrome (anti-glomerular
basement membrane antibody disease)
G. Post-streptococcal glomerulonephritis
H. Alport's syndrome

3. A previously healthy 61-year-old woman presents with a 3-month history of Your Answer:
sinusitis and nasal discharge. She has noted only marginal, temporary A. Wegener's granulomatosis
improvement despite multiple courses of antibiotics. The nasal discharge is (granulomatosis with polyangiitis)
purulent and frequently bloody. She also has a 2-week history of migratory Correct Answer:
joint pain, mainly affecting wrists, knees, and ankles. She does not describe Your answer is correct!
joint swelling. She reports having less energy and has lost 5kg in weight over
the past 2 months. She has no respiratory, urinary, neurological, or other
symptoms. Urinalysis reveals 3+ microscopic haematuria and 2+ proteinuria. All answers:
A. Wegener's granulomatosis
(granulomatosis with polyangiitis)
B. Berger’s disease (IgA Nephropathy)

https://www.mcqbank.co.uk/testhistory.php?view=1487811 64/120
04/09/2023, 11:44 mcqbank.co.uk/testhistory.php?view=1487811
C. Churg-Strauss syndrome (eosinophilic
granulomatosis with polyangiitis)
D. Henoch-Schönlein purpura (IgA
vasculitis)
E. Systemic lupus erythematosus
F. Goodpasture's syndrome (anti-glomerular
basement membrane antibody disease)
G. Post-streptococcal glomerulonephritis
H. Alport's syndrome

Feedback

Q1 - B

This man has IgA nephropathy (or Berger’s disease) which is the most common type of glomerulonephritis in adults
worldwide. It typically presents as nephritic syndrome (visible haematuria) 24-48 hours after an upper respiratory tract infection.
High-dose prednisolone can reduce proteinuria and delay renal impairment, and in patients with deteriorating renal function,
immunosuppressive drugs are often used. Prognosis is variable, with 20-30% of patients progressing to end-stage renal failure.
IgA nephropathy may be differentiated from a post-streptococcal glomerulonephritis where visible haematuria occurs 1 to 3
weeks after the onset of streptococcal infection. Note that Henoch-Schönlein purpura [HSP] also known as IgA vasculitis has
similar histopathological findings to IgA nephropathy but can be differentiated by the fact it is predominantly a disease of
childhood and by the presence of systemic symptoms, including purpuric rash (90% of cases); abdominal pain (85%); joint pain
(70%); and GI bleeding.

Q2 - G

This boy has a poststreptococcal (or postinfectious) glomerulonephritis which is an immunologically mediated glomerular
injury triggered by an infection, most commonly streptococcal infections. The disease typically presents approximately 2 weeks
after infection with nephritic syndrome (i.e. gross haematuria, oliguria, oedema). Management is largely supportive, with careful
monitoring of fluid balance. It is usually a self-limited disease, especially in children, but long-term follow-up studies indicate
persistent low-grade renal abnormalities in a significant proportion of patients.

Q3 - B

This woman has Wegener's granulomatosis (granulomatosis with polyangiitis [GPA]) which is a systemic vasculitis that
typically involves small and medium vessels. Although any organ may be targeted, the classic triad consists of upper and lower
respiratory tract involvement and pauci-immune glomerulonephritis. Involvement of cutaneous, ocular, musculoskeletal, and
peripheral nervous system tissue is also common. Differentials include Churg-Strauss syndrome or eosinophilic granulomatosis
with polyangiitis (EGPA) whose main features include asthma, gastrointestinal and cardiac involvement with significant (>10% of
peripheral WBC count) peripheral eosinophilia or tissue eosinophilia. Goodpasture's syndrome (anti-glomerular basement
membrane antibody disease) is another differential where the presentation of pulmonary haemorrhage and glomerulonephritis is
clinically indistinguishable from Wegener's granulomatosis. But, the presence of involvement in other sites (e.g. upper respiratory
tract) argues against a diagnosis of Goodpasture's syndrome. SLE and other connective tissue diseases (CTDs) are also another
possibility. SLE is characterised by photosensitive rashes, alopecia, serositis of pleura, pericardium, or peritoneum with a high
prevalence of ocular and oral sicca symptoms related to reduced tear and saliva production, and Raynaud phenomenon.

Glomerular diseases

Background
Renal glomeruli excrete urinary substances & excess water as an ultrafiltrate into the urine by selectively filtering the blood.
Any damage to the glomeruli disrupts the filtration process and results in blood components (proteins & red blood cells)
appearing in the urine.
Glomerular damage is commonly caused by immune-mediated processes, which often lead to glomerulonephritis.
Glomerulonephritis is an important cause of renal impairment accounting for 10–15% of end stage renal
disease (ESRD), following only diabetes & hypertension in importance.
Non-inflammatory causes, such as metabolic disease (e.g. diabetes, amyloidosis), can also result in significant damage to
the glomeruli.
Glomerular diseases are usually categorized by the syndrome they cause, which is either nephritic, nephrotic or mixed:
Nephritic syndrome (low-level proteinuria, haematuria, oliguria, & hypertension)
Nephrotic syndrome (high-level proteinuria & generalized oedema).
Mixed nephritic-nephrotic syndrome - some diseases that manifest with nephritic syndrome can simultaneously
cause nephrotic-range proteinuria (>3.5 g/day).
All glomerular diseases can progress to acute or chronic renal failure. Thus, quick diagnosis & initiation of therapy are
required to prevent irreversible kidney damage.

Nephritic Syndrome
Poststreptococcal glomerulonephritis
Poststreptococcal (or postinfectious) glomerulonephritis refers to acute glomerular inflammation that results from
a preceding streptococci infection.
Most commonly seen in children following group A streptococcal tonsillopharyngitis, but may also be triggered
by skin infections such as impetigo.

https://www.mcqbank.co.uk/testhistory.php?view=1487811 65/120
04/09/2023, 11:44 mcqbank.co.uk/testhistory.php?view=1487811
Deposition of immune complexes containing streptococcal antigen damages the glomerular basement membrane.
Typically presents 2w after infection as a nephritic syndrome with visible haematuria, mild proteinuria,
oedema, & hypertension. ↑ antistreptolysin O titers (ASO), ↓ complement levels, & ↑ creatinine support the
diagnosis.
While most children recover fully, the prognosis in adults is typically less favourable.
IgA nephropathy (Berger disease)
Most common primary glomerulonephritis in adults worldwide.
Most frequently affects men in 2nd-3rd decades of life.
Clinical manifestations are usually triggered 24-48h after an upper respiratory tract or GI infection and include
visible haematuria & flank pain.
Even with appropriate treatment, ~50% of patients progress to end-stage renal disease within 20–25 years.
Granulomatosis with polyangiitis (Wegener's granulomatosis)
Systemic vasculitis that typically involves small & medium vessels.
Classic triad = upper & lower respiratory tract involvement & glomerulonephritis.
Involvement of cutaneous, ocular, musculoskeletal, & peripheral nervous system tissue is also common.
Microscopic polyangiitis
Ill-defined autoimmune disease characterized by small-vessel vasculitis.
Eosinophilic granulomatosis with polyangiitis (Churg-Strauss syndrome)
Vasculitis is associated with asthma & eosinophilia.
May have perinuclear anti-neutrophil cytoplasmic antibodies (p-ANCA).
Cardiac involvement causes significant morbidity and mortality.
Rapidly progressive glomerulonephritis (RPGN)
Rapid destruction of renal glomeruli that often leads to end-stage renal disease.
3 different pathophysiological mechanisms that can result in RPGN:
Anti-glomerular basement membrane antibody disease (Goodpasture syndrome = combination of
glomerulonephritis & alveolar haemorrhage).
Immune complex glomerulonephritis (e.g. lupus nephritis).
Glomerulonephritis associated with vasculitis (e.g. Wegener's)
May present with mild symptoms such as flank pain, oedema, & ↓ urine output. Lab tests show sudden ↑ in urea
& creatinine as well as RBC casts & dysmorphic RBC on urinalysis.
Alport syndrome (hereditary nephritis)
Genetic disorder that is characterized by glomerulonephritis, often in combination with sensorineural hearing loss
& sometimes eye abnormalities.
Caused by a genetic defect of type IV collagen, usually inherited via X-linked pattern & therefore mostly affects
boys.
Typically, presents with intermittent visible haematuria during infancy. In adolescence, patients start to develop
more serious signs of chronic kidney disease (e.g. proteinuria), and may experience hearing loss or, rarely, vision
problems. In milder forms, patients remain asymptomatic & only require monitoring.
Classic form usually leads to end-stage renal disease between 2nd- 3rd decade of life - definitive treatment is
kidney transplant.
Thin basement membrane disease
Usually transmitted in autosomal dominant fashion with some families having a mutation in the type IV collagen
gene.
Most patients are asymptomatic & are incidentally noted to have microscopic haematuria on routine
urinalysis, although mild proteinuria & visible haematuria are occasionally present. Renal function is typically
normal, but some develop progressive renal failure for unknown reasons. Recurrent flank pain, similar to that in IgA
nephropathy, is a rare manifestation.

Nephritic-nephrotic syndrome
Membranoproliferative glomerulonephritis
Characterised by glomerular basement membrane (GBM) thickening.
Not to be confused with membranous glomerulonephritis where the basement membrane is thickened, but the
mesangium is not.
Associated with hepatitis C & several autoimmune conditions including SLE.
Diffuse proliferative glomerulonephritis
↑ cellular proliferation & involvement of >50% of glomeruli,
Most serious & most common form (~50%) of renal lesion in SLE.
Most have haematuria with moderate to severe proteinuria, hypertension, & renal insufficiency.

Nephrotic Syndrome
Minimal change disease
Most common cause of nephrotic syndrome in children.
Often idiopathic.
Secondary causes (rare):
Immune stimulus (e.g. infection, immunization)
Tumours (e.g. Hodgkin lymphoma)

https://www.mcqbank.co.uk/testhistory.php?view=1487811 66/120
04/09/2023, 11:44 mcqbank.co.uk/testhistory.php?view=1487811
Certain drugs (e.g. NSAIDs)
Responds well to prednisone.
Good prognosis.
Focal segmental glomerulosclerosis
Most common cause of nephrotic syndrome in adults.
Caused by injury to podocytes in the renal glomeruli.
Manifests initially with proteinuria, which progresses to nephrotic syndrome & ultimately to end-stage renal failure.
May be primary (idiopathic) due to unknown cause or secondary to underlying conditions, such as HIV, obesity,
medications.
Prognosis is extremely variable.
Membranous nephropathy
Common cause of nephrotic syndrome in adults.
Treatment in low-risk patients is largely conservative, including low-sodium & low-protein diet and statins for
hyperlipidaemia. Hypertension is treated with ACEi or ARB medication, & oedema managed with diuretics.
Medium/high risk patients may be treated with combination of corticosteroids with either cytotoxic agents or
ciclosporin.
Usually leads to ESRD if left untreated.
Diabetic nephropathy
Major cause of ESRD.
Seen in patients with diabetes for >10 years
Chronic hyperglycaemia → glycosylation of basement membrane → ↑ permeability & thickening of basement
membrane → hyperfiltration (↑ GFR) → ↑ intraglomerular pressure → progressive glomerular hypertrophy, ↑ renal size
& glomerular scarring (glomerulosclerosis) → worsening of filtration capacity.
Clinical features
Often asymptomatic - patients may complain of foamy urine.
Initially, moderately ↑ albuminuria (microalbuminuria),
Eventually, significantly ↑ albuminuria (macroproteinuria) & nephrotic syndrome may develop.
Prevention & management
Stringent glycaemic control.
Antihypertensive treatment - ACEi or ARB are first-line antihypertensives in patients with diabetes. 2nd line
agents to be added to ACEi or ARB to further control hypertension include diuretics or calcium channel
blockers.
Dietary modification - daily salt intake <5–6g/day; phosphorus & potassium intake restriction in advanced
nephropathy; protein restriction.
Amyloid light-chain (AL) amyloidosis, light chain deposition disease
Kidneys are the organs most commonly affected by primary amyloidosis.
Usually presents with unexplained weight loss, fatigue, & oedema resistant to diuretics.
Treatment includes appropriate management of resulting clinical syndromes, such as nephrotic syndrome,
neuropathy, cardiomyopathy, & conduction disorders.
Definitive treatment of primary systemic amyloidosis (AL amyloidosis) includes myeloablative high-dose
chemotherapy or stem cell reconstitution in selected patients.

References
Amboss – Glomerular diseases
BMJ Best Practice

Send us your feedback about this MCQ | Add question feedback to My Revision Notes | Rate this MCQ:

Single Best Answer Question

Maximum Mark: 1
Mark Awarded: 1
Time taken to answer: 0 minutes 46 seconds.
Main Question

A 16-year-old girl attends her GP surgery complaining of suprapubic pain, frequency and dysuria. Urinalysis shows the presence
of nitrites and leucocytes. Which one of the following is the MOST likely diagnosis?
Answers

Your answer:
B. Gram-negative urinary tract infection

Correct Answer:
Your answer is correct!

https://www.mcqbank.co.uk/testhistory.php?view=1487811 67/120
04/09/2023, 11:44 mcqbank.co.uk/testhistory.php?view=1487811
All Answers:
A. Gram-positive urinary tract infection
B. Gram-negative urinary tract infection
C. Renal stones
D. Urethral syndrome
E. Chlamydial urethritis

Feedback

This patient has a Gram-negative UTI.

Nitrites are the result of the breakdown of nitrates which are normally excreted in the urine, however gram-negative
bacteria such as E.coli contain the enzyme nitrate reductase which converts the nitrate into nitrite therefore the
presence of nitrite in the sample suggests a UTI.
Gram-positive bacteria however do not have this enzyme and therefore a sample containing such bacteria will show
negative, e.g. staphlococcus.
Leucocytes in the urine suggest pyuria but not all patients with a UTI have pyuria, especially the very young &
neutropenic patients.
Leucocytes from local sources (vagina, foreskin) may also contaminate urine & leucocytes may also appear in the
urine in many other febrile illnesses e.g. URTI, pneumonia, etc. so the specificity is low.
Leucocyte esterase can only be detected with relatively high WBC counts in urine, so the test has low sensitivity.

Send us your feedback about this MCQ | Add question feedback to My Revision Notes | Rate this MCQ:

Single Best Answer Question

Maximum Mark: 1
Mark Awarded: 1
Time taken to answer: 0 minutes 33 seconds.
Main Question

Which one of the following is NOT a cause of a sterile pyuria?


Answers

Your answer:
A. Acute pyelonephritis

Correct Answer:
Your answer is correct!

All Answers:
A. Acute pyelonephritis
B. Within 2 weeks of an inadequately treated UTI
C. Appendicitis
D. Prostatitis
E. Tubulointerstitial nephritis

Feedback

Sterile pyuria is the presence of elevated numbers of white cells (>10/mm3) in urine which appears sterile using standard
culture techniques. The causes of sterile pyuria include:

a treated UTI within 2 weeks of treatment/inadequately treated UTI


UTI with fastidious culture requirement
renal stones
prostatitis
chlamydia urethritis
renal papillary necrosis (e.g. from analgesic excess)
tubulointerstitial nephritis
genitourinary tuberculosis (always consider 3 early morning urine samples)
interstitial cystitis
urinary tract neoplasm
polycystic kidney
appendicitis

https://www.mcqbank.co.uk/testhistory.php?view=1487811 68/120
04/09/2023, 11:44 mcqbank.co.uk/testhistory.php?view=1487811

Send us your feedback about this MCQ | Add question feedback to My Revision Notes | Rate this MCQ:

Single Best Answer Question

Maximum Mark: 1
Mark Awarded: 1
Time taken to answer: 0 minutes 23 seconds.
Main Question

Which one of the following is characteristic of stress incontinence in female patients?


Answers

Your answer:
A. Leaking small amounts of urine on coughing.

Correct Answer:
Your answer is correct!

All Answers:
A. Leaking small amounts of urine on coughing.
B. Having to rush to get to the toilet on time.
C. Nocturia occurring 3 times nightly.
D. Dribbling after passing water.
E. Loss of continence when anxious.

Feedback

Stress and urge incontinence require different treatments and are often confused. Stress incontinence is typically associated
with leaking urine on coughing, sneezing or laughter, leaking on playing sport or sudden movement. Urge incontinence or
detrusor instability is more likely with a history of frequency of ≥6 per day & ≥3 times at night, leaking at night and having to
rush to the toilet. Dribbling is a symptom of overflow incontinence.

Urinary incontinence is the involuntary leakage of urine. There are several different types:

Functional incontinence occurs when patients have mental or physical disabilities (e.g. Parkinson's disease) that keep
them from urinating normally, although the urinary system itself is intact.
Stress incontinence is the involuntary loss of urine through an intact urethra secondary to an increase in intra-abdominal
pressure and in the absence of detrusor activity.
It is associated with a defect of the urethral sphincter.
Urine loss occurs when the intra-vesical pressure exceeds the maximum urethral pressure.
Presentation
Most patients complain of involuntary loss of urine on coughing or straining.
Often, this accompanied by symptoms of frequency & urgency.
Usually result of
childbirth - pregnancy, labour & delivery may all cause mechanical & denervation injury to the normal
supports of the bladder neck & proximal urethra
menopausal changes - decreased intraurethral pressure due to lack of oestrogen
Investigations
The urinary stress test is a simple test for stress incontinence.
The patient is examined in the lithotomy position with a full bladder.
The examiner asks the patient to cough whilst observing the urethral meatus.
Stress incontinence is indicated by short spurts of urinary loss simultaneous with each
cough.
Loss of large volumes of urine or delayed leakage suggests uninhibited bladder contractions.
The test should be repeated in the standing position if the initial result is negative.
Urodynamic studies include urinary flow rate, cystometry, urethral pressure profile & electromyography.
Management
Conservative management
lifestyle interventions such as caffeine reduction, reducing fluid intake or weight loss.
pelvic floor muscle training.
Anticholinergics such as oxybutynin.
Surgical treatment is reserved for those in whom conservative measures have failed.
Urge incontinence or detrusor instability is characterised by uncontrolled contraction of the bladder wall (detrusor
muscle) producing urgency & sometimes leakage, often with frequency & nocturia.
Detrusor overactivity is the second commonest cause of female urinary incontinence behind stress incontinence.

https://www.mcqbank.co.uk/testhistory.php?view=1487811 69/120
04/09/2023, 11:44 mcqbank.co.uk/testhistory.php?view=1487811
Risk factors include multiple sclerosis & stroke but most cases have no specific cause.
In men, urge incontinence may be due to neurological disease or an enlarged prostate.
Presentation
Involuntary urine leakage accompanied by, or immediately preceded by, urgency of micturition. This
means a sudden & compelling desire to urinate that cannot be deferred.
Urinary frequency is also common.
Investigations
Investigations include mid-stream urine MC&S to exclude urinary tract infection.
Urodynamic studies show involuntary contraction of bladder during filling.
Management
Management of incontinence in adults arising from detrusor instability is by combining:
Conservative management
Bladder training - the goals are to increase the amount of time between emptying the bladder
and the amount of fluids the bladder can hold.
Anticholinergics, e.g. oxybutynin have a direct relaxant effect on urinary smooth muscle. They
reduce involuntary detrusor contractions and increase bladder capacity.
Invasive options
Botulinum toxin A
Injection of the bladder wall with botulinum toxin A is the first-line invasive option.
Sacral nerve stimulation is used in patients who do not respond to other treatments.
Surgery is only indicated for intractable & severe detrusor overactivity.
Laparoscopic augmentation cystoplasty.
Mixed incontinence is involuntary leakage of urine associated with both urgency & exertion, effort, sneezing or coughing.
Overactive bladder syndrome (OAB) is urgency that occurs with or without urge incontinence, and usually with
frequency & nocturia.
It may be called 'OAB wet' or 'OAB dry', depending on whether or not the urgency is associated with incontinence.
The usual cause of this problem is detrusor overactivity.
Overflow incontinence is incomplete bladder emptying secondary to impaired detrusor contractility or bladder outlet
obstruction.
Factors involved in the development of overflow incontinence are
physical obstruction, such as pelvic organ prolapse & enlarged prostate
neurological abnormalities, such as spinal cord injuries
It is also commonly associated with bladder neuropathy as occurs in diabetes mellitus
Patients often complain of continuous small-volume leakage associated with weak urinary stream, dribbling,
hesitancy, frequency, & nocturia.
True incontinence may be due to a fistulous tract between the vagina & either the ureter, bladder, or urethra.
There is continuous leakage of urine.

Reference
Patient.co.uk - Urinary Incontinence

Send us your feedback about this MCQ | Add question feedback to My Revision Notes | Rate this MCQ:

Extended Matching Question

Maximum Mark: 3
Mark Awarded: 2
Time taken to answer: 1 minutes 37 seconds.
Main Question

For each patient below with urinary incontinence, choose the most likely cause.
Select ONE option only from the list.
Each option may be selected once, more than once or not at all.
Answers

1. A 55-year-old woman complains of urgency, frequency and nocturia. She is Your Answer:
unable to hold urine as soon as the urge to pass urine is felt and has soiled her D. Urge incontinence
underwear on a few occasions. Correct Answer:
Your answer is correct!

All answers:
A. Retention with overflow
B. True incontinence

https://www.mcqbank.co.uk/testhistory.php?view=1487811 70/120
04/09/2023, 11:44 mcqbank.co.uk/testhistory.php?view=1487811
C. Spinal cord compression
D. Urge incontinence
E. Functional incontinence
F. Mixed incontinence
G. Stress incontinence
H. None of the above

Your Answer:
G. Stress incontinence
Correct Answer:
Your answer is correct!

All answers:
2. A 55-year-old multiparous woman complains of leakage of small amounts of A. Retention with overflow
urine when coughing or sneezing. B. True incontinence
C. Spinal cord compression
D. Urge incontinence
E. Functional incontinence
F. Mixed incontinence
G. Stress incontinence
H. None of the above

Your Answer:
B. True incontinence
Correct Answer:
A. Retention with overflow

All answers:
3. An 85-year-old man with a background of benign prostatic hyperplasia A. Retention with overflow
complains of continuous small-volume leakage. B. True incontinence
C. Spinal cord compression
D. Urge incontinence
E. Functional incontinence
F. Mixed incontinence
G. Stress incontinence
H. None of the above

Feedback

Q1 - D

Q2 - G

Q3 - A

Urinary incontinence is the involuntary leakage of urine. There are several different types:

Functional incontinence occurs when patients have mental or physical disabilities (e.g. Parkinson's disease) that keep
them from urinating normally, although the urinary system itself is intact.
Stress incontinence is the involuntary loss of urine through an intact urethra secondary to an increase in intra-abdominal
pressure and in the absence of detrusor activity.
It is associated with a defect of the urethral sphincter.
Urine loss occurs when the intra-vesical pressure exceeds the maximum urethral pressure.
Presentation
Most patients complain of involuntary loss of urine on coughing or straining.
Often, this accompanied by symptoms of frequency & urgency.
Usually result of
childbirth - pregnancy, labour & delivery may all cause mechanical & denervation injury to the normal
supports of the bladder neck & proximal urethra
menopausal changes - decreased intraurethral pressure due to lack of oestrogen
Investigations
The urinary stress test is a simple test for stress incontinence.
The patient is examined in the lithotomy position with a full bladder.
The examiner asks the patient to cough whilst observing the urethral meatus.
Stress incontinence is indicated by short spurts of urinary loss simultaneous with each
cough.

https://www.mcqbank.co.uk/testhistory.php?view=1487811 71/120
04/09/2023, 11:44 mcqbank.co.uk/testhistory.php?view=1487811
Loss of large volumes of urine or delayed leakage suggests uninhibited bladder contractions.
The test should be repeated in the standing position if the initial result is negative.
Urodynamic studies include urinary flow rate, cystometry, urethral pressure profile & electromyography.
Management
Conservative management
lifestyle interventions such as caffeine reduction, reducing fluid intake or weight loss.
pelvic floor muscle training.
Anticholinergics such as oxybutynin.
Surgical treatment is reserved for those in whom conservative measures have failed.
Urge incontinence or detrusor instability is characterised by uncontrolled contraction of the bladder wall (detrusor
muscle) producing urgency & sometimes leakage, often with frequency & nocturia.
Detrusor overactivity is the second commonest cause of female urinary incontinence behind stress incontinence.
Risk factors include multiple sclerosis & stroke but most cases have no specific cause.
In men, urge incontinence may be due to neurological disease or an enlarged prostate.
Presentation
Involuntary urine leakage accompanied by, or immediately preceded by, urgency of micturition. This
means a sudden & compelling desire to urinate that cannot be deferred.
Urinary frequency is also common.
Investigations
Investigations include mid-stream urine MC&S to exclude urinary tract infection.
Urodynamic studies show involuntary contraction of bladder during filling.
Management
Management of incontinence in adults arising from detrusor instability is by combining:
Conservative management
Bladder training - the goals are to increase the amount of time between emptying the bladder
and the amount of fluids the bladder can hold.
Anticholinergics, e.g. oxybutynin have a direct relaxant effect on urinary smooth muscle. They
reduce involuntary detrusor contractions and increase bladder capacity.
Invasive options
Botulinum toxin A
Injection of the bladder wall with botulinum toxin A is the first-line invasive option.
Sacral nerve stimulation is used in patients who do not respond to other treatments.
Surgery is only indicated for intractable & severe detrusor overactivity.
Laparoscopic augmentation cystoplasty.
Mixed incontinence is involuntary leakage of urine associated with both urgency & exertion, effort, sneezing or coughing.
Overactive bladder syndrome (OAB) is urgency that occurs with or without urge incontinence, and usually with
frequency & nocturia.
It may be called 'OAB wet' or 'OAB dry', depending on whether or not the urgency is associated with incontinence.
The usual cause of this problem is detrusor overactivity.
Overflow incontinence is incomplete bladder emptying secondary to impaired detrusor contractility or bladder outlet
obstruction.
Factors involved in the development of overflow incontinence are
physical obstruction, such as pelvic organ prolapse & enlarged prostate
neurological abnormalities, such as spinal cord injuries
It is also commonly associated with bladder neuropathy as occurs in diabetes mellitus
Patients often complain of continuous small-volume leakage associated with weak urinary stream, dribbling,
hesitancy, frequency, & nocturia.
True incontinence may be due to a fistulous tract between the vagina & either the ureter, bladder, or urethra.
There is continuous leakage of urine.

Reference
Patient.co.uk - Urinary Incontinence

Send us your feedback about this MCQ | Add question feedback to My Revision Notes | Rate this MCQ:

Single Best Answer Question

Maximum Mark: 1
Mark Awarded: 1
Time taken to answer: 0 minutes 48 seconds.
Main Question

https://www.mcqbank.co.uk/testhistory.php?view=1487811 72/120
04/09/2023, 11:44 mcqbank.co.uk/testhistory.php?view=1487811

A 30-year-old woman with a history of Crohn's disease complains of passing foul-smelling, greenish-brown urine. She has also
noticed bubbles in her urinary stream. Which of the following tests would MOST likely reveal the diagnosis?
Answers

Your answer:
A. CT abdomen & pelvis

Correct Answer:
Your answer is correct!

All Answers:
A. CT abdomen & pelvis
B. Ultrasound pelvis
C. Colonoscopy
D. KUB x-ray
E. Barium enema

Feedback

This patient has pneumaturia.

Pneumaturia is the passage of gas mixed with urine, usually as a result of a vesico-colic fistula.
Causes include diverticulitis (50-70%), malignancy (20%) & Crohn’s disease (10%).
May occasionally be due to gas-producing bacterial infections of the urinary tract.
The patient presents with dysuria and frequency, and may describe bubbles in the urine.
There may also be faecaluria.
Urinalysis usually shows a full field of WBCs, bacteria, and debris.
Urine culture findings are most commonly interpreted as mixed flora.
The predominant offending organism is Escherichia coli.
A variant of the Bourne test using orally administered charcoal is also helpful.
Charcoal in the urine is detected either visually or microscopically in the centrifuged urine of patients.
CT scanning of the abdomen and pelvis is the most sensitive test for detecting a colovesical fistula, and CT scanning
should be included as part of the initial evaluation of patients with suspected colovesical fistulae.
CT scanning can demonstrate small amounts of air or contrast material in the bladder, localized thickening of the
bladder wall, or an extraluminal gas-containing mass adjacent to the bladder.
Cystoscopy is an essential component of the diagnostic evaluation.
The findings of this procedure can suggest the presence of a fistula, and cystoscopy can be used for evaluation for
possible malignancy.
Barium enema rarely reveals a fistula but is useful in differentiating diverticular disease from cancer.
Cystography may demonstrate contrast outside the bladder but is less likely to demonstrate a fistula.
Ultrasonography is rarely used for primary imaging of fistulae.
Magnetic resonance imaging may be useful in identifying deep perineal fistulae but is not generally used in the routine
workup of colovesical fistulae.
Colonoscopy is not particularly valuable in detecting a fistula, but it is helpful in determining the nature of the bowel
disease that caused the fistula and is typically part of the evaluation.

Reference
GP Notebook - Pneumaturia

Send us your feedback about this MCQ | Add question feedback to My Revision Notes | Rate this MCQ:

Single Best Answer Question

Maximum Mark: 1
Mark Awarded: 0
Time taken to answer: 26 minutes 16 seconds.
Main Question

A 25-year-old man presents with frank haematuria and flank pain. His father had kidney problems and died from an intracerebral
bleed. On examination, he is hypertensive and enlarged kidneys can be balloted. Which of the following would be the MOST
appropriate initial investigation?
Answers

Your answer:
B. CT Abdomen

https://www.mcqbank.co.uk/testhistory.php?view=1487811 73/120
04/09/2023, 11:44 mcqbank.co.uk/testhistory.php?view=1487811
Correct Answer:
E. Ultrasound abdomen

All Answers:
A. KUB film
B. CT Abdomen
C. Cystoscopy
D. Aldosterone/renin ratio (ARR)
E. Ultrasound abdomen

Feedback

Autosomal dominant polycystic kidney disease (ADPKD) is the most frequent genetic cause of renal failure in adults.

Approximately 90% have an abnormality on chromosome 16.


ADPKD is a multisystemic and progressive disorder with formation of cysts in the kidney and other organs (e.g. liver,
pancreas, spleen)
Hepatic cysts may occur in 75% of patients >60y.
They appear to be affected by female steroid hormones, and, although they may occur in both sexes, massive
cysts occur almost exclusively in women.
Non-cystic manifestations include an increased incidence of mitral valve prolapse, intracranial berry aneurysms (in up
to 40% of cases) and an increased frequency of colonic diverticula & renal cell carcinoma.
Presentation
Clinical features usually begin in the 3rd - 4th decade of life, but cysts may be detectable in childhood.
Patients usually present with
haematuria - from cyst rupture
flank pain - from cyst enlargement
hypertension
stroke - from subarachnoid haemorrhage from intracranial aneurysms
renal failure
palpable masses in the flanks
increased incidence of UTIs
Investigations
Urinalysis - check for infection, protein (microalbuminuria occurs in about a third but heavy proteinuria is rare)
and haematuria.
Bloods - FBC (polycystic kidneys can produce excess erythropoietin and hence raise Hb), U&E
Ultrasound can detect renal cysts from 1-1.5 cm in diameter.
Sensitivity is 100% over the age of 20 but false negatives can occur below this age.
It is also possible to scan other organs like the liver or pancreas for cysts.
CT is more sensitive in that it can detect cysts down to 0.5 cm diameter but the dose of radiation is quite high.
MRI is an equally sensitive alternative.
Gene testing can be done by linkage or sequence analysis:
Management
Diagnosis of ADPKD has implications both for the patient & their family.
Treatment is currently supportive only.
No treatments have, as yet, been shown to halt or even slow disease progression.

Send us your feedback about this MCQ | Add question feedback to My Revision Notes | Rate this MCQ:

Single Best Answer Question

Maximum Mark: 1
Mark Awarded: 1
Time taken to answer: 1 minutes 6 seconds.
Main Question

A 30-year-old woman is referred by her GP to the urogynaecologist as she has a long history of experiencing sudden, urges to
urinate without incontinence since the age of 14 years and with concomitant pelvic pain from the age of 22 years. She is currently
symptomatic and has brought in her 3-day voiding diary which demonstrates around 20 voids per 24 hours and 6 voids after
bedtime. Fluid intake was reasonable at 2 litres. On bi-manual examination, there is pain with palpation of the urethra, bladder
neck, and levator ani (pelvic diaphragm). Which ONE of the following is the MOST likely diagnosis?
Answers

Your answer:
B. Interstitial cystitis (bladder pain syndrome)

https://www.mcqbank.co.uk/testhistory.php?view=1487811 74/120
04/09/2023, 11:44 mcqbank.co.uk/testhistory.php?view=1487811

Correct Answer:
Your answer is correct!

All Answers:
A. Vaginitis
B. Interstitial cystitis (bladder pain syndrome)
C. Endometriosis
D. Overactive bladder syndrome
E. Urinary tract infection

Feedback

Although interstitial cystitis (bladder pain syndrome) is a diagnosis of exclusion, features in this vignette such as the
extended duration of symptoms, pelvic pain, urinary symptoms, the examination findings and lack of gynaecological symptoms is
most suggestive of this diagnosis.

Vaginitis is inflammation of the vagina due to changes in the composition of the vaginal micro-environment from infection (e.g.
bacterial vaginosis, trichomoniasis, and candidiasis), irritants, or from hormonal deficiency (e.g. atrophic vaginitis). Symptoms
may include vaginal itching, burning, discharge or vaginal odour.

Endometriosis is defined as the presence of endometrial glands and stroma outside the endometrial cavity and uterine
musculature. Symptoms include urethral pain and haematuria, dysfunctional menstrual bleeding with pain increasing during the
menstrual cycle. Internal examination reveals high abdominal and cervical tenderness. Absence of anterior vaginal wall
tenderness over urethra or bladder neck on bi-manual examination differentiates this from interstitial cystitis. Laparoscopy
demonstrates sites of ectopic endometrial implantation.

Overactive bladder (OAB) syndrome is characterised by urgency, often with frequency and nocturia and sometimes leakage
(urge incontinence). It is often but not always associated with detrusor muscle overactivity. There is an absence of pain with
palpation of the urethra, bladder neck, and levator ani (pelvic diaphragm) on bi-manual examination differentiating this
from interstitial cystitis. Urodynamic studies may show involuntary contraction of the bladder during filling. If cystoscopy is
performed there are no glomerulations and/or Hunner's ulcers with hydro-distension which may be present with interstitial
cystitis.

A urinary tract infection (UTI) is an infection of the kidneys, bladder, or urethra. Infectious cystitis is the most common type of
UTI, which is caused by a bacterial infection of the bladder. Symptoms include fever and haematuria. Urine cultures are positive
and responds to antibiotic therapy. Note UTI may co-exist with interstitial cystitis (bladder pain syndrome).

Interstitial cystitis (bladder pain syndrome)

Background
Interstitial cystitis/bladder pain syndrome (IC/BPS) is a chronic, often debilitating, clinical syndrome of urinary
frequency, urgency, & pelvic pain.
IC/BPS is placed within the overall scope of chronic pelvic pain syndromes (CPPS), which also includes prostate,
gynaecological, & bowel pain syndromes.

Clinical features
The condition is typified by exacerbations, remissions, & varying degrees of symptom severity, although most patients
present with several months or years of symptoms.
Pain generally varies with bladder filling, and may be referred to the suprapubic area, urethra, or pelvis.
The resulting discomfort may range from abdominal suprapubic tenderness to intense pain of the pelvic floor
& dyspareunia.
Bi-manual pelvic examination
Reveals pain in the anterior vaginal wall overlying the urethra & bladder neck; severe pubic pain when the body
of the bladder is examined; and pain on palpation of the vaginal pelvic side walls onto the muscles of the pelvic
diaphragm, including the pubococcygeus & iliococcygeus, located at the 5 & 7 o'clock positions, superficially & deep
inside the vagina.

Investigations
IC/BPS is a diagnosis of exclusion, so other common disease processes must first be excluded before effective treatment
can be initiated.
Cystoscopy
May reveal a diffusely erythematous bladder epithelium & ulcerative patches (Hunner's ulcers) surrounded by
mucosal congestion.
Maximum bladder capacity under anaesthesia may be severely limited (<300mL) indicating an end-stage fibrotic
bladder.
After over-distension, many patients demonstrate glomerulations (discrete, tiny, raspberry-like lesions appearing
as minuscule mucosal tears & haemorrhages).
It should be noted that a negative cystoscopic examination does not rule out the diagnosis.

Management
Commonly, patients are treated for recurrent bacterial cystitis despite negative cultures.
On average, a patient experiences symptoms for 5 years prior to diagnosis.

https://www.mcqbank.co.uk/testhistory.php?view=1487811 75/120
04/09/2023, 11:44 mcqbank.co.uk/testhistory.php?view=1487811
Treatment approach is aimed at symptom relief & pain management.
Minimally invasive options (dietary changes, stress management, & behavioural modification e.g. bladder training
techniques & avoiding activities that exacerbate the condition, anticholinergic medication) tried before progressing to
more complex, invasive therapies (e.g. pelvic-floor trigger point injections, pelvic floor botox injections, surgery -
bladder augmentation or cystectomy with urinary diversion).

Reference
BMJ Best Practice - Interstitial cystitis (bladder pain syndrome)

Send us your feedback about this MCQ | Add question feedback to My Revision Notes | Rate this MCQ:

Single Best Answer Question

Maximum Mark: 1
Mark Awarded: 0
Time taken to answer: 1 minutes 45 seconds.
Main Question

A 65-year-old man undergoes a transurethral resection of the prostate (TURP) without complication. However, the nurses have
called you a couple of hours after he has arrived back on the ward as he has not passed any urine into his catheter bag. His
observations are unremarkable and the nurses suspect his catheter may be blocked. Which one of the following actions is MOST
appropriate?
Answers

Your answer:
B. Flush catheter with 1% hydrogen peroxide

Correct Answer:
C. Bladder washout with normal saline

All Answers:
A. Fluid challenge with a litre of normal saline
B. Flush catheter with 1% hydrogen peroxide
C. Bladder washout with normal saline
D. Recatheterise
E. Remove the catheter

Feedback

There is no suggestion in the question that the patient is dry and therefore a large volume fluid challenge would be
inappropriate here. His catheter bag is empty suggesting a blockage which is not uncommon following urinary tract surgery.
Bladder washout (see below) is a quick procedure to perform but if this does not relieve the blockage the catheter may need to
be changed. It would be unwise to leave the patient without a catheter - the urethra will be swollen following surgery so a
catheter will be required for the bladder to empty and also the catheter is typically attached to irrigation fluid to continuously
wash out the bladder.

Bladder washout is indicated for:


1. To clear an obstructed catheter.
2. To remove potential sources of obstruction e.g. blood clots or sediment from infection.
3. To prevent the formation & retention of blood clots e.g. following prostatic surgery.
4. On rare occasions to remove heavily contaminated material from a diseased urinary bladder.

A bladder washout is performed by flushing about 50ml of fluid into the end of the catheter end and then withdrawing a similar
amount of urine/flush. This can be repeated several times.
0.9% saline is used as it is isotonic, water should not be used as it is readily absorbed by osmosis.
Other solutions used include 0.02% chlorhexidine to prevent bacterial growth, 1% mandelic acid to prevent growth of
urease-producing bacteria, or 6% citric acid to dissolve persistent crystallisation.

Send us your feedback about this MCQ | Add question feedback to My Revision Notes | Rate this MCQ:

https://www.mcqbank.co.uk/testhistory.php?view=1487811 76/120
04/09/2023, 11:44 mcqbank.co.uk/testhistory.php?view=1487811

Single Best Answer Question

Maximum Mark: 1
Mark Awarded: 0
Time taken to answer: 1 minutes 24 seconds.
Main Question

Which one of the following is NOT characteristic of urethral trauma?


Answers

Your answer:
B. High-riding prostate on rectal exam

Correct Answer:
D. Blood in the rectal lumen

All Answers:
A. Absence of palpable prostate on rectal exam
B. High-riding prostate on rectal exam
C. Blood at the external urethral meatus
D. Blood in the rectal lumen
E. Scrotal haematoma

Feedback

Urethral injuries can be classified into 2 categories based on the anatomical site of the trauma.

Posterior urethral injuries are located in the membranous & prostatic urethra.
These injuries are most commonly related to major blunt trauma such as RTA & falls and are commonly associated
with pelvic fractures.
Anterior urethral injuries are located distal to the membranous urethra.
Most anterior urethral injuries come from blunt trauma to the perineum (straddle injuries), and many have delayed
manifestation, appearing years later as a stricture.
Presentation
Look for perineal bruising & blood at the external urethral meatus
And perform a rectal examination - abnormally high riding prostate or inability to palpate the prostate imply
urethral injury
Management
If you suspect urethral injury do not attempt urethral catheterisation but refer urgently to the Urologists.
Some Urologists perform a retrograde urethrogram.
Others prefer suprapubic catheterisation & subsequent imaging.

Send us your feedback about this MCQ | Add question feedback to My Revision Notes | Rate this MCQ:

Single Best Answer Question

Maximum Mark: 1
Mark Awarded: 1
Time taken to answer: 2 minutes 48 seconds.
Main Question

Which combination of the following parameters on urinalysis test strip (dipstick) is MOST useful for diagnosing urinary tract
infection (UTI) in adult women?

https://www.mcqbank.co.uk/testhistory.php?view=1487811 77/120
04/09/2023, 11:44 mcqbank.co.uk/testhistory.php?view=1487811

Answers

Your answer:
A. Leucocytes, nitrite, blood

Correct Answer:
Your answer is correct!

All Answers:
A. Leucocytes, nitrite, blood
B. Leucocytes, blood, glucose
C. Nitrite, protein, blood
D. Protein, specific gravity, glucose
E. Ketone, bilirubin, glucose

Feedback

According to NICE guidance, if the woman is under 65 years of age, and does not have risk factors for complicated UTI, urine
dipstick can be used as an aid to diagnosis. But, dipstick is unreliable in women aged older than 65 years and those who are
catheterised.

The nitrite urine dipstick test suggests the presence of bacteria and the leukocyte esterase dipstick test suggests the presence
of leukocytes. UTI is a common cause of haematuria.

The principle of the nitrite test is that most of the microorganisms causing UTI have the ability of converting urinary nitrate
to nitrite. The conversion occurs in urine which stays in the urinary bladder for a few hours. However, someone with more
frequent urination will have less stasis in the urinary bladder and thus may have a false negative result. Therefore, an early
morning sample is preferable. The nitrite test has high specificity but low sensitivity (i.e. a positive test provides strong
[but not conclusive] evidence that the urine is infected; while a negative test only provides weak evidence to exclude
infection).
Leukocyte esterase is an enzyme produced by leukocytes. The leukocyte esterase test has low specificity but high
sensitivity (i.e. a positive test only provides weak evidence that the urine is infected; but a negative test provides strong
[but not conclusive] evidence that the urine is not infected).

[Reference: NICE Clinical Knowledge Summaries - Urinary tract infection (lower)]

Urinary tract infection (lower)

Background
Lower urinary tract infection (UTI) = infection of the bladder.
Cystitis is often used as a synonym for lower UTI (particularly for women), although technically it means 'bladder
inflammation' and there may be non-infectious causes such as radiation & chemical-induced cystitis.
Upper UTI is infection of the upper urinary tract i.e. ureters & kidneys (pyelonephritis).
UTI is very common in women & accounts ~2% of all GP consultations.

https://www.mcqbank.co.uk/testhistory.php?view=1487811 78/120
04/09/2023, 11:44 mcqbank.co.uk/testhistory.php?view=1487811
50% of women will be treated for a symptomatic UTI during their lifetime.
UTI is not common in men, but incidence is higher in elderly men (likely to have additional risk factors) or
with indwelling urinary catheters.
Usually caused by gut bacteria from GI tract.
Escherichia coli accounts for 80% & Staphylococcus saprophyticus 10% of UTIs. Proteus mirabilis is more common
in men or associated with renal tract abnormalities, particularly calculi.

Definitions

Significant bacteriuria = 105 colony-forming units per ml (CFU/mL).


Asymptomatic bacteriuria = significant bacteriuria without clinical infection.
NICE Antenatal Care guidelines state that asymptomatic bacteriuria should be screened for & treated in
pregnancy as associated with premature delivery & low birthweight.
Do not treat asymptomatic bacteriuria in patients with indwelling catheters.
Uncomplicated UTI = typical pathogens with normal urinary tract & kidney function, and no predisposing co-morbidities.
Complicated UTI = increased likelihood of complications such as persistent infection, treatment failure & recurrent
infection. Assoc with ≥1 risk factors:
Abnormal urinary tract (e.g. calculus, vesicoureteric reflux, neurogenic bladder, indwelling catheter, urinary
obstruction, recent instrumentation).
Virulent organism (e.g. Staphylococcus aureus).
Immunosuppression (e.g. poorly controlled diabetes).
Impaired renal function.
Recurrent UTI = Defined as ≥2 UTI in 6mths or ≥3 UTI in 12mths. Can be:
Relapse - same organism
Reinfection - different organism

Diagnosis
Typical features of UTI = dysuria, frequency, urgency, changes in urine appearance or
consistency, nocturia, suprapubic discomfort/tenderness.
Note typical features may be absent in the elderly with underlying cognitive impairment — can present
with delirium, lethargy, ↓ ability to carry out activities of daily living & anorexia.
Women
If uncomplicated UTI in <65y.o, urine dipstick can be used as a diagnostic aid — dipstick is unreliable in women
aged >65y & those who are catheterised.
If dipstick +ve for nitrite or leukocyte & RBC ⟹ UTI is likely.
Send urine for culture if previous antibiotic treatment has failed or possibility of antibiotic resistance.
If dipstick -ve for nitrite & +ve for leukocyte ⟹ UTI is possible.
Send urine for culture to confirm diagnosis.
If dipstick -ve for nitrite, leukocyte & RBC ⟹ UTI is unlikely.
No need to send urine culture — consider other diagnoses.
Send urine for culture if:
Pregnant.
>65y.
Persistent symptoms that do not resolve with antibiotics.
Recurrent UTI.
Urinary catheter in situ or recently catheterised.
Risk factors for resistance or complicated UTI such as GU tract abnormalities, renal impairment,
residence in a long-term care facility, hospitalisation for >7d in last 6m, recent travel to a country with ↑
resistance or previous resistant UTI.
Atypical symptoms.
Visible or non-visible haematuria.
Men
Confirm diagnosis with urine culture before starting empirical drug treatment.
If catheterised - only send urine sample if features of systemic infection.
Do not use urine dipstick or microscopy to diagnose UTI in men:
Who are not catheterized — dipsticks are poor at ruling out infection.
But, may be helpful in some situations, e.g. UTI is unlikely if mild or non-specific symptoms AND -
ve urine dipstick test (i.e. both nitrite & leukocytes -ve).
Note, presence of these markers does not rule in UTI, although +ve nitrite makes UTI more
likely (PPV 96%).
With an indwelling catheter — working diagnosis should be based on clinical judgement.
Aged >65y — dipsticks become more unreliable with increasing age >65y.

Differential diagnosis
Pyelonephritis - urinary symptoms associated with fever &/or loin pain.
Drug-induced cystitis - cyclophosphamide, allopurinol, danazol, or tiaprofenic acid.
Sexually transmitted Infections e.g. chlamydia trachomatis (pyuria without bacteriuria).

https://www.mcqbank.co.uk/testhistory.php?view=1487811 79/120
04/09/2023, 11:44 mcqbank.co.uk/testhistory.php?view=1487811
Dermatological conditions e.g. psoriasis, dermatitis, lichen sclerosis or lichen planus.
Spondyloarthropathies e.g. reactive arthritis or Bechet’s syndrome.
Malignancy.
Ovarian cancer may present with persistent urinary urgency &/or frequency.
Bladder, or renal cancer may present with haematuria (visible or non-visible).
In women – consider
Urethral syndrome (aka painful bladder syndrome, interstitial cystitis & trigonitis = symptoms of cystitis in
absence of UTI - symptoms relieved by voiding & aggravated by drinking alcohol or caffeinated drinks).
Atrophic vaginitis/urethritis (menopausal woman with vaginal discharge or itch, & pain during sexual
intercourse).
In men – consider
Acute prostatitis (fever, irritative urinary symptoms, perineal/suprapubic pain, pain on ejaculation or with bowel
movements, & exquisitely tender prostate on PR).
Epididymitis (scrotal pain & epididymis is oedematous & tender).

Empirical Treatment
Women
If mild symptoms with normal immunity, renal function & renal tract, treatment can be delayed if she wishes
to see if symptoms resolve, especially if -ve urine dip (nitrites & leucocytes) indicating low probability of UTI.
For all other women especially if pregnant prescribe treatment without delay.
1st Choice
Nitrofurantoin for 3d.
If eGFR ≥45.
c/i in G6PD deficiency & acute porphyria.
Trimethoprim for 3d.
BNF advises caution if predisposed to folate deficiency.
Renal impairment:
If eGFR 15–30 - use half normal dose after 3d.
If eGFR <15 - use half normal dose.
c/i in blood dyscrasias.
2nd Choice (if no improvement in symptoms when 1st-choice antibiotic is taken for at least 48h or if first-
choice is unsuitable).
Nitrofurantoin for 3d (if not used as 1st choice).
Pivmecillinam for 3d.
Fosfomycin single dose sachet.
Pregnant
Nitrofurantoin (avoid at term due to risk of neonatal haemolysis) for 7d.
2nd choice
Amoxicillin (only if culture results available & susceptible) for 7d.
Cefalexin for 7d.
BNF states that manufacturer advises avoiding trimethoprim during pregnancy. There is a
teratogenic risk in first trimester (folate antagonist).
Men
Start empirical antibiotic drug treatment with trimethoprim or nitrofurantoin for 7d.

Duration
3d course for women with:
Uncomplicated UTI.
UTI with haematuria.
Recurrent UTI.
5–10d course for women who have:
Abnormal urinary tract.
Immunosuppression.
Impaired renal function.
7d course if:
Pregnant.
Catheterised.
Change catheter before starting antibiotics if catheter has been insitu for >7d.
Consider withholding antibiotics until culture results available to guide treatment if mild symptoms with normal
immunity & renal function.
Male.

References
NICE Clinical Knowledge Summaries - Urinary tract infection (lower) – women
NICE Clinical Knowledge Summaries - Urinary tract infection (lower) – men
British National Formulary

https://www.mcqbank.co.uk/testhistory.php?view=1487811 80/120
04/09/2023, 11:44 mcqbank.co.uk/testhistory.php?view=1487811

Send us your feedback about this MCQ | Add question feedback to My Revision Notes | Rate this MCQ:

Extended Matching Question

Maximum Mark: 3
Mark Awarded: 2
Time taken to answer: 1 minutes 43 seconds.
Main Question

For each patient below with a scrotal swelling, choose the most likely cause.
Select ONE option only from the list.
Each option may be selected once, more than once or not at all.
Answers

Your Answer:
G. Epididymal cyst
Correct Answer:
H. Hydrocele

All answers:
1. A 40-year-old man presents with a painless enlarged scrotum. On
A. Acute epididymo-orchitis
examination, there is a non-tender, soft, transilluminable swelling which lies
B. Indirect inguinal hernia
superior and anterior to the testis.
C. Direct inguinal hernia
D. Spermatocele
E. Varicocele
F. Testicular torsion
G. Epididymal cyst
H. Hydrocele

Your Answer:
F. Testicular torsion
Correct Answer:
Your answer is correct!

All answers:
2. A 13-year-old boy complains of sudden onset pain in his left testicle. On
A. Acute epididymo-orchitis
examination, the left testicle is painful to touch and is higher than the right.
B. Indirect inguinal hernia
Cremasteric reflex is lost on the left side.
C. Direct inguinal hernia
D. Spermatocele
E. Varicocele
F. Testicular torsion
G. Epididymal cyst
H. Hydrocele

Your Answer:
A. Acute epididymo-orchitis
Correct Answer:
Your answer is correct!

3. A 25-year-old man complains of scrotal pain and dysuria which started a All answers:
couple of days ago. He also admits to urethral discharge. On examination, his A. Acute epididymo-orchitis
scrotum is red, swollen and tender. Elevation of his scrotum seems to relieve B. Indirect inguinal hernia
the pain. C. Direct inguinal hernia
D. Spermatocele
E. Varicocele
F. Testicular torsion
G. Epididymal cyst
H. Hydrocele

Feedback

Q1 - H

Hydrocele is a fluid collection within the tunica vaginalis of the scrotum or along the spermatic cord.

https://www.mcqbank.co.uk/testhistory.php?view=1487811 81/120
04/09/2023, 11:44 mcqbank.co.uk/testhistory.php?view=1487811
Prevalence is 1% of adult men.
Pathology
Three types:
Communicating or congenital hydroceles have a patent processus vaginalis permitting flow of peritoneal
fluid into the scrotum.
Non-communicating hydroceles have a patent processus vaginalis but no communication with the
peritoneal cavity occurs.
Hydrocele of the cord occurs when closure of the tunica vaginalis is defective.
>80% of newborn boys have a patent processus vaginalis, but most close spontaneously within 18 months of age.
Aetiology
Most paediatric hydroceles are congenital, but rarer causes include malignancy & infection.
Hydroceles in adults may be secondary to
orchitis
epididymitis
tuberculosis
torsion
testicular tumours
trauma
following renal transplantation
post-radiation therapy
Presentation
2 peak ages of presentation (1-2year & >40years).
The usual presentation is a painless enlarged scrotum with a sensation of heaviness or dragging.
Hydroceles are located superior & anterior to the testis, in contrast to spermatoceles, which lie
superior & posterior to the testis.

Management
Investigations are usually not required but ultrasound can help to determine if any underlying pathology is present.
Many newborn hydroceles resolve before the age of 1 year and so observation is usually appropriate.
Once underlying pathology has been excluded, adults with non-communicating hydroceles can be managed
conservatively with a scrotal support.
Surgery may be indicated if the hydrocele is large & uncomfortable, becomes infected or repeatedly
reaccumulates after aspiration.

Q2 - F

Torsion of the testis causes occlusion of testicular blood vessels.

Peak incidence around 13yrs of age.


The left side is more commonly affected than the right with 2% of cases being bilateral.
Risk factors
high insertion of the tunica vaginalis (producing a "bell clapper testis" with a horizontal lie).

https://www.mcqbank.co.uk/testhistory.php?view=1487811 82/120
04/09/2023, 11:44 mcqbank.co.uk/testhistory.php?view=1487811

undescended testes.
Presentation
History includes a sudden onset of severe unilateral scrotal pain & scrotal swelling.
50% of patients have a history of prior intermittent testicular pain that has resolved spontaneously.
There may be associated nausea & vomiting, abdominal pain, fever and urinary frequency.
Examination reveals a tender testicle frequently elevated in position with a horizontal lie when compared with
the other side.
The scrotum is oedematous and there is ipsilateral loss of the cremasteric reflex.
Pain is not relieved upon elevation of the scrotum (elevation may improve the pain in epididymitis
- Prehn's sign).
Management
Testicular torsion is a clinical diagnosis.
Imaging studies usually are not necessary.
If the history and examination suggest testicular torsion, the patient should go directly to theatre.
The patient &/or his parents must be warned of the possible need to remove the testis if non-viable.
Ultrasound may be ordered if there is doubt and shows absence of arterial flow.
Urinalysis is usually normal but WBC can be observed in 30% - therefore, do not rely on urinary WBC presence
to exclude the diagnosis.

Q3 - A

Acute epididymo-orchitis is inflammation of the epididymis & adjacent testicle.

Incidence 1:1000 men.


Peaks in 15-30 years and the >60 year olds.
In men <35y.o, pathogens are most likely Chlamydia trachomatis & Neisseria gonorrhoeae.
Those >35y.o, pathogens are usually non-sexually transmitted gram -ve enteric organisms such as E.
coli & Pseudomonas.
If recent instrumentation or catheterisation has occurred then gram -ve enteric organisms are again more
commonly the cause
Presentation
Gradual in nature, with symptoms often peaking within 24 hours of onset.
Initially, the patient may note abdominal or flank pain which then localizes to the scrotum.
There may also be scrotal swelling, urinary
frequency, urgency, dysuria, nausea, fever, chills & urethral discharge.
On examination, the scrotum is erythematous & oedematous.
Early on, in cases without significant testicular involvement, tenderness may be clearly localized to the
epididymis.
The scrotum is often fixed to the underlying epididymis.
Classically, scrotal elevation decreases pain in epididymitis & not in torsion (Prehn's
sign).
Investigations
urinalysis & MSU
urethral swab
first void urine: for gonorrhoea & chlamydia
Management
May need to screen patient & contacts for STI and for other patients may need to investigate for anatomical
abnormalities of the urinary tract.
Treatment
Uncomplicated gonorrhoea
cefixime or ciprofloxacin
Uncomplicated genital chlamydial infection, non-gonococcal urethritis & non-specific genital infection
doxycycline or azithromycin
If urine dipstick positive and most likely enteric organisms

https://www.mcqbank.co.uk/testhistory.php?view=1487811 83/120
04/09/2023, 11:44 mcqbank.co.uk/testhistory.php?view=1487811
treat as for 'complicated UTI' - e.g. trimethoprim or ofloxacin for 14 days

Send us your feedback about this MCQ | Add question feedback to My Revision Notes | Rate this MCQ:

Single Best Answer Question

Maximum Mark: 1
Mark Awarded: 1
Time taken to answer: 0 minutes 17 seconds.
Main Question

A 40-year-old man is seen in A&E having been involved in a RTA. You note he has some blood at the external urethral meatus and
perineal bruising. Which one of the following should you NOT attempt?
Answers

Your answer:
B. Urethral catheterisation

Correct Answer:
Your answer is correct!

All Answers:
A. Pelvic x-ray
B. Urethral catheterisation
C. Rectal examination
D. Retrograde urethrogram
E. Suprapubic catheter

Feedback

Urethral injuries can be classified into 2 categories based on the anatomical site of the trauma.

Posterior urethral injuries are located in the membranous & prostatic urethra.
These injuries are most commonly related to major blunt trauma such as RTA & falls and are commonly associated
with pelvic fractures.
Anterior urethral injuries are located distal to the membranous urethra.
Most anterior urethral injuries come from blunt trauma to the perineum (straddle injuries), and many have delayed
manifestation, appearing years later as a stricture.
Presentation
Look for perineal bruising & blood at the external urethral meatus
And perform a rectal examination - abnormally high riding prostate or inability to palpate the prostate imply
urethral injury
Management
If you suspect urethral injury do not attempt urethral catheterisation but refer urgently to the Urologists.
Some Urologists perform a retrograde urethrogram.
Others prefer suprapubic catheterisation & subsequent imaging.

Send us your feedback about this MCQ | Add question feedback to My Revision Notes | Rate this MCQ:

https://www.mcqbank.co.uk/testhistory.php?view=1487811 84/120
04/09/2023, 11:44 mcqbank.co.uk/testhistory.php?view=1487811

Single Best Answer Question

Maximum Mark: 1
Mark Awarded: 1
Time taken to answer: 0 minutes 15 seconds.
Main Question

Which ONE of the following is a complication of an undescended testicle?


Answers

Your answer:
D. Impaired fertility

Correct Answer:
Your answer is correct!

All Answers:
A. Reduced risk of testicular torsion
B. Smaller volume testicle
C. Increased incidence of epididymo-orchitis
D. Impaired fertility
E. Reduced risk of developing testicular malignancy

Feedback

Fertility is impaired after both, unilateral or bilateral cryptorchidism. It has been quoted that around 90% of patients with
untreated bilateral cryptorchidism ultimately develop azoospermia as against the reported 0.5% incidence in the general
population.

[Reference - Goel P et al. Undescended testicle: An update on fertility in cryptorchid men. Indian J Med Res. 2015;141(2):163-
171]

The term cryptorchidism, from the Greek kryptos (hidden) & orchis (testicle), is used to describe an undescended testis or a
testis that is absent from the scrotum.

Cryptorchidism is the most common congenital anomaly of the male genitalia


Prevalence in the region of 8 per 1000 live births.
There is a higher incidence in premature babies (up to 25%).
Embryology
By the 7th month of gestation, the right & left testes should reach their respective inguinal canals & by 9 months,
each testis has usually reached the scrotum.

Absence may be due to:


1. Testicular agenesis (anorchia)
uncommon
2. Retractile testis
from an exaggerated cremasteric reflex
3. Ascending testis syndrome

https://www.mcqbank.co.uk/testhistory.php?view=1487811 85/120
04/09/2023, 11:44 mcqbank.co.uk/testhistory.php?view=1487811
a previously normal or a retractile testis can become high with a shortened spermatic cord that
prevents the testis from staying in the scrotum
4. Testicular maldescent
descent can be
Arrested - where descent is along the normal path but incomplete.
Ectopic - where descent deviates from the normal path and is most often found in the superficial
inguinal pouch.

Clinical features
Around 70% of all undescended testes are palpable.
It can be difficult to distinguish undescended testes from retractile testis.
Unilateral cryptorchidism is 4x more likely than bilateral and the right testis is more likely to be affected than the
left.
Diagnosis
Imaging or ultrasound does not add any benefit to differentiating between palpable & non-palpable testes.
Diagnostic laparoscopy is the only way to confirm or rule out an intra-abdominal, inguinal or absent/vanishing
testis (non-palpable testis).
However, an examination under anaesthetic should be carried out before laparoscopy as a previously non-
palpable testis may become palpable.
Management
If, by the age of one year, descent has not occurred, spontaneous descent is unlikely. Treatment should be initiated,
as there is also potential for histological deterioration and loss of testicular quality (may affect future fertility).
If there is unilateral undescended testis still present at 3 months then the child should be referred to an appropriate
paediatric surgeon, ideally before the age of 6 months.
The ideal management of cryptorchidism is a highly debated topic within the field of paediatric surgery.
Medical
Testicular descent is hormonally dependent - treatment with human chorionic gonadotrophin (hCG) or
gonadotrophin-releasing hormone (GnRH) has been used in some centres but NICE has found that
hormone therapy is ineffective and therefore not recommended.
Surgical
Orchiopexy before 10-11 years may protect against the ↑ risk of testicular cancer associated with
cryptorchidism.
Orchiopexy should not be performed before 6 months of age, as testes may descend spontaneously
during the first few months of life.
Treatment should be completed by 12-18 months of age.
Complications
↑ risk of testicular torsion (if intraabdominal testis will present as acute abdomen)
Impaired fertility
20x increased risk of developing testicular malignancy

Reference
Patient.co.uk Professional Reference - Undescended and Maldescended Testes

Send us your feedback about this MCQ | Add question feedback to My Revision Notes | Rate this MCQ:

Single Best Answer Question

Maximum Mark: 1
Mark Awarded: 0
Time taken to answer: 1 minutes 21 seconds.

https://www.mcqbank.co.uk/testhistory.php?view=1487811 86/120
04/09/2023, 11:44 mcqbank.co.uk/testhistory.php?view=1487811
Main Question

Which one of the following is the BEST general advice to give to patients to prevent renal stones?
Answers

Your answer:
A. Consume fewer dairy products.

Correct Answer:
E. Reduce chocolate intake.

All Answers:
A. Consume fewer dairy products.
B. Drink plenty of tea.
C. Eat a diet rich in nuts.
D. Ensure diet is rich in fruits such as rhubarb & strawberry.
E. Reduce chocolate intake.

Feedback

A renal stone or calculus is a solid aggregation formed in the kidneys from dietary minerals in the urine.

Demographics
Annual incidence is 1:1000 & the average lifetime risk is 10%
Male to female ratio is 3:1
The peak age is between 30 - 50.
Pathology
Consist of crystal aggregates that form in the collecting ducts which can then be deposited anywhere from the renal
pelvis to the urethra.
calcium oxalate (75%)
magnesium ammonium phosphate [struvite] (10%)
urate (5%)
hydroxyapatite (5%)
cystine (1%)
Risk factors
Anatomical anomalies in the kidneys &/or urinary tract,
e.g. horseshoe kidney, ureteral stricture
Family history of renal stones
Hypertension
Gout
Hyperparathyroidism
Immobilisation
Relative dehydration
commoner in hot climates
Metabolic disorders
which increase excretion of solutes
e.g. chronic metabolic acidosis, hypercalciuria, hyperuricosuria
Deficiency of citrate in the urine
Cystinuria (an autosomal recessive aminoaciduria)
Drugs, especially thiazide diuretics
Presentation
Many stones are asymptomatic and discovered during investigations for other conditions.
Classic symptom is sudden, severe pain.
Pain starts in the loin about the level of the costovertebral angle & moves to the groin, with tenderness of the
loin or renal angle, sometimes with haematuria.
If the stone is high & distends the renal capsule then pain will be in the flank but as it moves down pain will
move anteriorly and down towards the groin.
Other symptoms which may be present include:
Rigors and fever
Dysuria
Haematuria
Urinary retention
Nausea and vomiting
Investigations
Urinalysis
The stone often causes some bleeding into the renal tract and this may produce a positive result for blood on
stick testing
a negative test does not exclude the diagnosis
Mid-stream specimen of urine for microscopy, culture & sensitivities.

https://www.mcqbank.co.uk/testhistory.php?view=1487811 87/120
04/09/2023, 11:44 mcqbank.co.uk/testhistory.php?view=1487811
Bloods
FBC, U&E, calcium & phosphate and urate.
Blood cultures & parathyroid hormone if indicated.
Encourage the patient to try to catch the stone for analysis.
This may mean urinating through a tea strainer, filter paper such as a coffee filter or a gauze.
Helical CT or CT KUB is the gold standard for the investigation of urinary stones (99% of stones visible) & has
superseded intravenous urography (IVU).
Plain X-rays (80% of stones are visible on a KUB plain film).
Renal ultrasound may also detect stones & reveal any evidence of obstruction (hydronephrosis or hydroureter)
Management
Most kidney stones do not require surgery and will pass on their own.
Surgery is necessary when the pain is persistent & severe, in renal failure and when there is a kidney
infection.
It may also be advisable if the stone fails to pass or move after 30 days.
Finding a significant stone before it passes into the ureter allows physicians to fragment it surgically before it causes
any severe problems.
In most of these cases, non-invasive extracorporeal shock wave lithotripsy (ESWL) will be used.
Otherwise some form of invasive procedure is required.
With approaches including
Ureteroscopic fragmentation (or simple basket extraction if feasible) using laser
Ultrasonic or mechanical (pneumatic, shock-wave) forms of energy to fragment the larger stones.
Percutaneous nephrolithotomy (PCNL) or rarely open surgery may ultimately be necessary for large or complicated
stones or stones which fail other less invasive attempts at treatment.
Prevention
A normal calcium intake is now recommended as low calcium diets increase oxalate excretion.
Advise to consume less oxalate by reducing tea, chocolate, nuts, strawberries, rhubarb, spinach, beans & beetroot &
to drink plenty of fluids especially in the summer.
Complications
Complete blockage of the urinary flow from a kidney decreases glomerular filtration rate (GFR) and, if it persists
for >48 hours, may cause irreversible renal damage.
If ureteric stones cause symptoms after 4 weeks, there is a 20% risk of complications, including deterioration of
renal function, sepsis, & ureteric stricture.
Infection can be life-threatening.
Persisting obstruction predisposes to pyelonephritis.

Send us your feedback about this MCQ | Add question feedback to My Revision Notes | Rate this MCQ:

Single Best Answer Question

Maximum Mark: 1
Mark Awarded: 1
Time taken to answer: 0 minutes 22 seconds.
Main Question

A 10-year-old boy complains of severe pain in his left testicle after playing football with his friends. He denies trauma to his
scrotum during the game. On examination, his scrotum appears red and swollen. The left testicle is higher than the right.
Elevation of the testicle does not relieve the pain. Which one of the following is the MOST likely diagnosis?
Answers

Your answer:
D. Testicular torsion

Correct Answer:
Your answer is correct!

All Answers:
A. Left-sided cremasteric muscle spasm post-exercise
B. Trauma to a partially descended left testicle
C. Incarcerated scrotal hernia
D. Testicular torsion
E. Acute epididymo-orchitis

Feedback

Torsion of the testis causes occlusion of testicular blood vessels.

https://www.mcqbank.co.uk/testhistory.php?view=1487811 88/120
04/09/2023, 11:44 mcqbank.co.uk/testhistory.php?view=1487811

Peak incidence around 13yrs of age.


The left side is more commonly affected than the right with 2% of cases being bilateral.
Risk factors
high insertion of the tunica vaginalis (producing a "bell clapper testis" with a horizontal lie).

undescended testes.
Presentation
History includes a sudden onset of severe unilateral scrotal pain & scrotal swelling.
50% of patients have a history of prior intermittent testicular pain that has resolved spontaneously.
There may be associated nausea & vomiting, abdominal pain, fever and urinary frequency.
Examination reveals a tender testicle frequently elevated in position with a horizontal lie when compared with
the other side.
The scrotum is oedematous and there is ipsilateral loss of the cremasteric reflex.
Pain is not relieved upon elevation of the scrotum (elevation may improve the pain in epididymitis
- Prehn's sign).
Management
Testicular torsion is a clinical diagnosis.
Imaging studies usually are not necessary.
If the history and examination suggest testicular torsion, the patient should go directly to theatre.
The patient &/or his parents must be warned of the possible need to remove the testis if non-viable.
Ultrasound may be ordered if there is doubt and shows absence of arterial flow.
Urinalysis is usually normal but WBC can be observed in 30% - therefore, do not rely on urinary WBC presence
to exclude the diagnosis.

Send us your feedback about this MCQ | Add question feedback to My Revision Notes | Rate this MCQ:

Extended Matching Question

Maximum Mark: 3
Mark Awarded: 2
Time taken to answer: 0 minutes 53 seconds.
Main Question

For each patient below with haematuria, choose the most likely cause.
Select ONE option only from the list.
Each option may be selected once, more than once or not at all.
Answers

1. A 45-year-old man presents with visible haematuria for the last week but Your Answer:
this has now cleared. He also complains of flank pain and on examination he is C. Polycystic kidney disease
found to be hypertensive with ballotable masses in both his flanks. Correct Answer:
Your answer is correct!

All answers:
A. Acute pyelonephritis
B. Chronic pyelonephritis
C. Polycystic kidney disease
D. Bladder carcinoma
E. Renal cell carcinoma
F. Torsion of the testis
G. Ureteric calculus

https://www.mcqbank.co.uk/testhistory.php?view=1487811 89/120
04/09/2023, 11:44 mcqbank.co.uk/testhistory.php?view=1487811
H. Aortic dissection

Your Answer:
D. Bladder carcinoma
Correct Answer:
E. Renal cell carcinoma

All answers:
2. A 50-year-old obese man who is a smoker complains of blood in his urine,
A. Acute pyelonephritis
pain in his right flank and he has lost a stone in weight over the last month.
B. Chronic pyelonephritis
On examination, a tender mass can be palpated in his right flank.
C. Polycystic kidney disease
D. Bladder carcinoma
E. Renal cell carcinoma
F. Torsion of the testis
G. Ureteric calculus
H. Aortic dissection

Your Answer:
G. Ureteric calculus
Correct Answer:
Your answer is correct!

All answers:
3. A 30-year-old man presents with sudden, severe, crampy, right loin pain
A. Acute pyelonephritis
radiating to his groin associated with nausea and vomiting. Urinalysis reveals a
B. Chronic pyelonephritis
microscopic haematuria.
C. Polycystic kidney disease
D. Bladder carcinoma
E. Renal cell carcinoma
F. Torsion of the testis
G. Ureteric calculus
H. Aortic dissection

Feedback

Q1 - C

Autosomal dominant polycystic kidney disease (ADPKD) is the most frequent genetic cause of renal failure in adults.
Approximately 90% have an abnormality on chromosome 16.
ADPKD is a multisystemic and progressive disorder with formation of cysts in the kidney and other organs (e.g. liver,
pancreas, spleen)
Hepatic cysts may occur in 75% of patients >60y.
They appear to be affected by female steroid hormones, and, although they may occur in both sexes, massive
cysts occur almost exclusively in women.
Non-cystic manifestations include an increased incidence of mitral valve prolapse, intracranial berry aneurysms (in up
to 40% of cases) and an increased frequency of colonic diverticula & renal cell carcinoma.
Presentation
Clinical features usually begin in the 3rd - 4th decade of life, but cysts may be detectable in childhood.
Patients usually present with
haematuria - from cyst rupture
flank pain - from cyst enlargement
hypertension
stroke - from subarachnoid haemorrhage from intracranial aneurysms
renal failure
palpable masses in the flanks
increased incidence of UTIs
Investigations
Urinalysis - check for infection, protein (microalbuminuria occurs in about a third but heavy proteinuria is rare)
and haematuria.
Bloods - FBC (polycystic kidneys can produce excess erythropoietin and hence raise Hb), U&E
Ultrasound can detect renal cysts from 1-1.5 cm in diameter.
Sensitivity is 100% over the age of 20 but false negatives can occur below this age.
It is also possible to scan other organs like the liver or pancreas for cysts.
CT is more sensitive in that it can detect cysts down to 0.5 cm diameter but the dose of radiation is quite high.
MRI is an equally sensitive alternative.
Gene testing can be done by linkage or sequence analysis:
Management
Diagnosis of ADPKD has implications both for the patient & their family.

https://www.mcqbank.co.uk/testhistory.php?view=1487811 90/120
04/09/2023, 11:44 mcqbank.co.uk/testhistory.php?view=1487811
Treatment is currently supportive only.
No treatments have, as yet, been shown to halt or even slow disease progression.

Q2 - E

Renal cell carcinoma (RCC)


Most common tumour of the kidney in adults, accounting for >80% of renal neoplasms.
In children, Wilms' tumour is most common.
Benign tumours of the kidney are rare.
RCC originates from the proximal renal tubular epithelium.
Renal cancers account for ~2-3% of all malignancies, with highest incidence in Western countries.
Men are 1.5x more likely to be affected than women.
Incidence rises after 40y & is highest in 60-70 year olds.
Several risk factors have been identified, particularly smoking & obesity.
~2-3% of RCCs are hereditary & several autosomal dominant syndromes are described, the most common being
von Hippel Lindau disease.
The classic triad of haematuria, loin pain & loin mass is not often seen as the use of ultrasound & CT scans has
increased detection of asymptomatic RCC.
Other presentations include fatigue, weight loss, pyrexia of unknown origin (paraneoplastic
manifestation), hypertension, varicocele & leg oedema.
The left testicular vein drains first into the left renal vein & then to the inferior vena cava (IVC) while the right
testicular vein drains directly into the IVC just below the renal vein.

A left-sided renal cancer can block the renal vein due to external compression by a large tumour or internal
obstruction by advancing tumour cells. Further tumour extension into the IVC may also cause IVC obstruction.
~30% of patients present with metastatic disease – e.g. haemoptysis, bone pain or pathological fracture.
Lungs = most common site of metastases where the classical picture of cannon ball secondaries is almost
diagnostic. It is also one of the carcinomas to metastasise to bone where it produces osteolytic lesions.

Q3 - G

A renal stone or calculus is a solid aggregation formed in the kidneys from dietary minerals in the urine.

Demographics
Annual incidence is 1:1000 & the average lifetime risk is 10%
Male to female ratio is 3:1
The peak age is between 30 - 50.
Pathology
Consist of crystal aggregates that form in the collecting ducts which can then be deposited anywhere from the renal
pelvis to the urethra.
calcium oxalate (75%)
magnesium ammonium phosphate [struvite] (10%)
urate (5%)
hydroxyapatite (5%)
cystine (1%)
Risk factors
Anatomical anomalies in the kidneys &/or urinary tract,
e.g. horseshoe kidney, ureteral stricture
Family history of renal stones
Hypertension
Gout
Hyperparathyroidism
Immobilisation
Relative dehydration
commoner in hot climates

https://www.mcqbank.co.uk/testhistory.php?view=1487811 91/120
04/09/2023, 11:44 mcqbank.co.uk/testhistory.php?view=1487811
Metabolic disorders
which increase excretion of solutes
e.g. chronic metabolic acidosis, hypercalciuria, hyperuricosuria
Deficiency of citrate in the urine
Cystinuria (an autosomal recessive aminoaciduria)
Drugs, especially thiazide diuretics
Presentation
Many stones are asymptomatic and discovered during investigations for other conditions.
Classic symptom is sudden, severe pain.
Pain starts in the loin about the level of the costovertebral angle & moves to the groin, with tenderness of the
loin or renal angle, sometimes with haematuria.
If the stone is high & distends the renal capsule then pain will be in the flank but as it moves down pain will
move anteriorly and down towards the groin.
Other symptoms which may be present include:
Rigors and fever
Dysuria
Haematuria
Urinary retention
Nausea and vomiting
Investigations
Urinalysis
The stone often causes some bleeding into the renal tract and this may produce a positive result for blood on
stick testing
a negative test does not exclude the diagnosis
Mid-stream specimen of urine for microscopy, culture & sensitivities.
Bloods
FBC, U&E, calcium & phosphate and urate.
Blood cultures & parathyroid hormone if indicated.
Encourage the patient to try to catch the stone for analysis.
This may mean urinating through a tea strainer, filter paper such as a coffee filter or a gauze.
Helical CT or CT KUB is the gold standard for the investigation of urinary stones (99% of stones visible) & has
superseded intravenous urography (IVU).
Plain X-rays (80% of stones are visible on a KUB plain film).
Renal ultrasound may also detect stones & reveal any evidence of obstruction (hydronephrosis or hydroureter)
Management
Most kidney stones do not require surgery and will pass on their own.
Surgery is necessary when the pain is persistent & severe, in renal failure and when there is a kidney
infection.
It may also be advisable if the stone fails to pass or move after 30 days.
Finding a significant stone before it passes into the ureter allows physicians to fragment it surgically before it causes
any severe problems.
In most of these cases, non-invasive extracorporeal shock wave lithotripsy (ESWL) will be used.
Otherwise some form of invasive procedure is required.
With approaches including
Ureteroscopic fragmentation (or simple basket extraction if feasible) using laser
Ultrasonic or mechanical (pneumatic, shock-wave) forms of energy to fragment the larger stones.
Percutaneous nephrolithotomy (PCNL) or rarely open surgery may ultimately be necessary for large or complicated
stones or stones which fail other less invasive attempts at treatment.
Prevention
A normal calcium intake is now recommended as low calcium diets increase oxalate excretion.
Advise to consume less oxalate by reducing tea, chocolate, nuts, strawberries, rhubarb, spinach, beans & beetroot &
to drink plenty of fluids especially in the summer.
Complications
Complete blockage of the urinary flow from a kidney decreases glomerular filtration rate (GFR) and, if it persists
for >48 hours, may cause irreversible renal damage.
If ureteric stones cause symptoms after 4 weeks, there is a 20% risk of complications, including deterioration of
renal function, sepsis, & ureteric stricture.
Infection can be life-threatening.
Persisting obstruction predisposes to pyelonephritis.

Send us your feedback about this MCQ | Add question feedback to My Revision Notes | Rate this MCQ:

https://www.mcqbank.co.uk/testhistory.php?view=1487811 92/120
04/09/2023, 11:44 mcqbank.co.uk/testhistory.php?view=1487811

Single Best Answer Question

Maximum Mark: 1
Mark Awarded: 0
Time taken to answer: 0 minutes 20 seconds.
Main Question

Which ONE of the following Staphylococcus organisms is associated with UTIs in young fit adults, especially women?
Answers

Your answer:
B. S. aureus

Correct Answer:
A. S. saprophyticus

All Answers:
A. S. saprophyticus
B. S. aureus
C. S. epidermidis
D. S. haemolyticus
E. S. warneri

Feedback

In acute infection, the organisms are those normally colonising the perineum:
E.coli – 80% of all UTIs
Proteus
Klebsiella
Enterobacter
Candida
Enterococci
Staphylococci saprophyticus in sexually active young women

With abnormalities of the urinary tract, more unusual organisms are found - such as:
Pseudomonas aeruginosa
Staphylococcus epidermidis

Send us your feedback about this MCQ | Add question feedback to My Revision Notes | Rate this MCQ:

Extended Matching Question

Maximum Mark: 3
Mark Awarded: 1
Time taken to answer: 9 minutes 42 seconds.
Main Question

For each patient below with oliguria, choose the most likely cause.
Select ONE option only from the list.
Each option may be selected once, more than once or not at all.
Answers

1. You are called to see a 59-year-old man who is oliguric one day post Your Answer:
gastrectomy. Blood tests show a urea of 14mmol/l, creatinine 110mmol/l, A. Atonic bladder
potassium 5mmol/l and urinary sodium is 7mmol/l (normal 25-250mmol/l). Correct Answer:
H. Hypovolaemia

All answers:
A. Atonic bladder
B. Acute tubular necrosis
C. Clot retention
D. Injury to sacral nerve plexus
E. Urethral transection
F. Bladder perforation
G. Acute peritonitis

https://www.mcqbank.co.uk/testhistory.php?view=1487811 93/120
04/09/2023, 11:44 mcqbank.co.uk/testhistory.php?view=1487811
H. Hypovolaemia

Your Answer:
H. Hypovolaemia
Correct Answer:
B. Acute tubular necrosis

2. A 30-year-old man is stabbed in the leg. He has lost a lot of blood and he is
All answers:
severely shocked when he is brought to A&E. He is resuscitated with blood and
A. Atonic bladder
once stable it is noted that he is oliguric. His urea is 18mmol/l, creatinine
B. Acute tubular necrosis
180mmol/l, potassium 5.5mmol/l and urinary sodium is 70mmol/l (normal 25-
C. Clot retention
250mmol/l).
D. Injury to sacral nerve plexus
E. Urethral transection
F. Bladder perforation
G. Acute peritonitis
H. Hypovolaemia

Your Answer:
C. Clot retention
Correct Answer:
Your answer is correct!

All answers:
3. A 70-year-old man one day post TURP complains of colicky lower abdominal A. Atonic bladder
pain and he has not passed urine all day. He has a palpable bladder. B. Acute tubular necrosis
C. Clot retention
D. Injury to sacral nerve plexus
E. Urethral transection
F. Bladder perforation
G. Acute peritonitis
H. Hypovolaemia

Feedback

Q1 - H

The most likely cause of his oliguria is that he is volume depleted from a combination of being kept NBM pre-operatively, losses
incurred during surgery and inadequate post-operative fluid replacement. This is supported by the urinary sodium which in
pre-renal failure is usually <20mmol/l as a result of reabsorbing sodium.

Q2 - B

Most acute renal failure (or now known as acute kidney injury) is caused by acute tubular necrosis which is characterised by
acute loss of renal excretory function.

Urinary sodium is classically >30 mmol/l, urea <150 mmol/l and urine plasma osmolality ratio is <1:1.
Interpretation of the above values is invalidated if loop diuretics or mannitol have been given.
Recovery is expected over days or weeks.
It is not much affected by therapeutic intervention as the kidney probably recovers by cellular regeneration, which
management is unable to speed up - hence only supportive therapy.
Classification
Ischaemic
Mainly affects the distal convoluted tubules & loop of Henle.
Due to "shock" commonly hypovolaemic, cardiogenic, or septic.
Pigment-induced ATN is a special form associated with massive haemoglobinuria (massive haemolysis) or
myoglobinuria (rhabdomyolysis).
Nephrotoxic
Mainly affects the proximal convoluted tubules.
Many agents are implicated, including
antibiotics - gentamicin
anaesthetics - methoxyflurane
heavy metals - mercury, lead, arsenic
chemotherapeutic agents - cyclosporine
organic solvents - methyl alcohol, carbon tetrachloride
poison - pesticides
radiological contrast

https://www.mcqbank.co.uk/testhistory.php?view=1487811 94/120
04/09/2023, 11:44 mcqbank.co.uk/testhistory.php?view=1487811
Q3 - C

Clot retention occurs as a sequel to bleeding in the urological tract.


Large clots can become wedged in the urethra & cause acute urinary retention.
It can be seen under any circumstance where there is macroscopic haematuria, but patients are most susceptible post
operatively if bladder irrigation is not sufficient - they suffer from clot retention within the catheter.

Send us your feedback about this MCQ | Add question feedback to My Revision Notes | Rate this MCQ:

Extended Matching Question

Maximum Mark: 3
Mark Awarded: 2
Time taken to answer: 3 minutes 19 seconds.
Main Question

For each patient below with acute renal impairment, choose the most likely cause.
Select ONE option only from the list.
Each option may be selected once, more than once or not at all.
Answers

Your Answer:
D. Primary aldosteronism
Correct Answer:
E. Renal artery stenosis

1. A 32-year-old woman with no prior medical history presented to her GP with


All answers:
worsening headache and is found to have a blood pressure of 180/110 mmHg.
A. Phaeochromocytoma
Her BP does not respond to thiazide diuretics and calcium-channel blockers. An
B. Fibromyalgia
ACE-inhibitor is commenced, and monitoring blood tests show a potassium of
C. Influenza
7.2 mmol/l, urea 24 mmol/l and creatinine 557μmol/l.
D. Primary aldosteronism
E. Renal artery stenosis
F. Renal colic
G. Rhabdomyolysis
H. Polycystic kidney disease

Your Answer:
G. Rhabdomyolysis
Correct Answer:
Your answer is correct!

2. A 23-year-old man presents to his GP after experiencing worsening back


pain for 3 days. He had been training in an extreme work-out session for
All answers:
endurance enhancement and bodybuilding, and also admits to using muscle
A. Phaeochromocytoma
protein supplements for the previous 2 months. He has taken ibuprofen in the
B. Fibromyalgia
past 3 days with no improvement in pain. He has also developed generalised
C. Influenza
fatigue and malaise. He notes his urine to be tea-coloured and reports a
D. Primary aldosteronism
gradually decreasing urine output.
E. Renal artery stenosis
F. Renal colic
G. Rhabdomyolysis
H. Polycystic kidney disease

Your Answer:
F. Renal colic
Correct Answer:
Your answer is correct!

3. A 32-year-old woman presents to A&E complaining of severe pain in her left


flank. The pain started 6 hours before and varies in intensity, getting worse
All answers:
every 5 to 10 minutes. When the pain is at its worst, she is unable to get
A. Phaeochromocytoma
comfortable in any position. She is nauseous and has vomited twice. She is
B. Fibromyalgia
afebrile and has tenderness at the left costovertebral angle. She was born with
C. Influenza
a solitary kidney and her blood tests show acutely elevated urea and creatinine
D. Primary aldosteronism
levels.
E. Renal artery stenosis
F. Renal colic
G. Rhabdomyolysis
H. Polycystic kidney disease

https://www.mcqbank.co.uk/testhistory.php?view=1487811 95/120
04/09/2023, 11:44 mcqbank.co.uk/testhistory.php?view=1487811
Feedback

Q1 - E

Renal artery stenosis (RAS) due to fibromuscular dysplasia is more common in women <30 years. RAS in general often
presents as accelerated, resistant, or malignant hypertension. RAS may be associated with acute decline in kidney function after
initiation of renin-angiotensin blockade. It may be diagnosed in assessing an unexplained atrophic kidney, or discrepancy in
kidney size >1.5 cm, or sudden, unexplained, and/or recurrent pulmonary oedema.
Primary hyperaldosteronism may also present with resistant or accelerated hypertension but here the plasma potassium is
usually low (while urine potassium is high).
Phaeochromocytoma may also present with resistant or accelerated hypertension or possibly episodic hypertension. The clinical
picture may include headaches, diaphoresis, and palpitations in the setting of paroxysmal hypertension. Acute decline in kidney
function after initiation of renin-angiotensin blockade is not a feature, however.

Renal artery stenosis


RAS = narrowing of the renal artery lumen.
It is considered angiographically significant if >50% reduction in vessel diameter is present.
Typically, due to atherosclerotic disease (older individuals) or fibromuscular dysplasia.
As the renal artery lumen progressively narrows, renal blood flow ↓ & eventually compromises renal function.
Ischaemic nephropathy = chronic reduction in GFR that occurs from a narrowing in the renal artery.
Renovascular hypertension = hypertension mediated by high levels of renin & angiotensin II, produced by an
underperfused kidney supplied by a stenosed renal artery.
Presentation
Most cases of RAS are asymptomatic, and the main problem is high blood pressure that cannot be controlled
with medication.
Worsening kidney function, especially after initiating renin-angiotensin blockade (e.g. ACE-inhibitors), &
recurrent flash pulmonary oedema are common features.
Management
Definitive diagnosis is with imaging.
Many patients may already be taking multiple antihypertensive medications.
As majority have refractory or difficult-to-control hypertension, regardless of the aetiology (fibromuscular
dysplasia vs atherosclerotic).
Addition of aspirin & high-intensity statin should follow as part of secondary prevention measures.
Control of other CVD risk factors (such as tobacco cessation or glycaemic control) is also recommended.
Percutaneous intervention (angioplasty ± stenting) remains controversial.
May be considered in patients with
difficult-to-control hypertension despite aggressive medical therapy.
rapidly declining level of kidney function.
recurrent flash pulmonary oedema.
[Reference - BMJ Best Practice – Renal artery stenosis]

Q2 – G

Unlike traumatic presentations, medical presentations of rhabdomyolysis are often subtle and must be suspected given the
appropriate clinical setting. The patient may be without symptoms or may have vague complaints of increased fatigue or
generalised malaise. Rather than complaining of muscular discomfort, the patient with drug use or an infection may present with
dark-coloured urine or oedema of the limbs. In one general review, only 50% of patients complained of muscular symptoms, and
physical findings of rhabdomyolysis were present in only 4% on admission. Unless the diagnosis is suspected and specific
laboratory evidence is obtained, rhabdomyolysis may not be diagnosed until complications ensue.
Renal colic may also present with low back pain and classically is described as severe, acute flank pain that radiates to the
ipsilateral groin. The pain is said to be some of the most severe physical pain a person can experience. However, some patients
may have no radiation and some stones are asymptomatic.
Fibromyalgia features musculoskeletal pain but the pain is typically a widespread body pain with areas of focal tenderness at
designated points. In addition, creatine kinase is not elevated and urine is of normal colour.
Influenza is characterised by myalgia but is also accompanied by respiratory tract symptoms, fever, and headache. Creatine
kinase will not be elevated unless influenza viral myositis is present which is very rare and is more likely in children.

Rhabdomyolysis
May result from any traumatic or medical injury to the skeletal muscle cells.
The subsequent release of intracellular ions, myoglobin, creatine kinase (CK), and urates into the circulation results
in electrolyte disturbances, acidaemia, disseminated intravascular coagulation, renal failure, & multi-
organ failure.
Presentation
May have an obvious presentation, such as traumatic 'crush' injury, or may be insidious, requiring a high clinical
index of suspicion.
Muscular pain or discomfort is common - tense & tender muscle compartments suggest compartment
syndrome.
Note that rhabdomyolysis can have no symptoms or physical signs.
Acute renal failure develops in ~35% of patients.

https://www.mcqbank.co.uk/testhistory.php?view=1487811 96/120
04/09/2023, 11:44 mcqbank.co.uk/testhistory.php?view=1487811
Suggested mechanisms include precipitation of myoglobin & uric acid crystals within renal tubules, ↓
glomerular perfusion, & the nephrotoxic effect of ferrihemate (formed upon dissociation of myoglobin in the
acidic environment of the renal parenchyma).
Management
Investigations
↑K & ↓Ca feature (low calcium levels may be present initially due to binding of free calcium to damaged muscle
cells).
↑ transaminases.
Urinalysis may reveal "blood" in the absence of red blood cells on microscopy, as the reagent reacts
with myoglobin.
Most reliable test is creatine kinase (CK) levels in the blood.
This enzyme is released by damaged muscle, & levels >5x upper limit of normal (normal <200 iU/l)
indicate rhabdomyolysis.
Myoglobin has a short half-life, & is therefore less useful as a diagnostic test in the later stages.
Treatment
Involves fluid resuscitation to maintain a urine output of 2-3ml/kg/hr, correction of acidosis & electrolyte
imbalances, urine alkalinisation may be required, mannitol has been advocated, dialyse if renal failure.
Compartment syndrome requires urgent fasciotomy.
[Reference – BMJ Best Practice – Rhabdomyolysis]

Q3 – F

Many patients with nephrolithiasis are asymptomatic, as their stone may be in the kidney and non-obstructing. In these
patients, diagnosis may be made following imaging (CT scan, abdominal x-ray, renal ultrasound, etc.) for other reasons. On the
hand, some patients may present with gross haematuria, evidence of an obstructive uropathy, or sepsis with fever, tachycardia,
and hypotension. Note that renal stones in patients with a solitary kidney requires urgent urological review in case of obstruction
and irreversible renal damage.

Renal colic
Renal or ureteric colic is a common condition generally describing acute and severe loin pain caused when a urinary
stone moves from the kidney or obstructs the flow of urine.
Risk factors include chronic dehydration, family history, Crohn's disease & medications such as protease inhibitors.
The main symptom is severe unilateral abdominal pain starting in the loin or flank and radiating to the labia in
women or the groin or testicle in men. Pain is often accompanied by nausea, vomiting, & haematuria.
Complications include urinary tract obstruction (hesitancy of micturition or an intermittent urinary stream) or
coexisting urinary tract infection (fever & sweats)
Immediate admission should be arranged if the person is:
In shock or has signs of systemic infection (e.g. fever & sweats).
At ↑ risk of acute kidney injury e.g. pre-existing chronic kidney disease, solitary or transplanted kidney, or if
bilateral obstructing stones are suspected.
Dehydrated and cannot take oral fluids due to nausea &/or vomiting.
For all others with suspected renal colic:
Urgent (within 24h of presentation) imaging should be offered (low-dose non-contrast CT for most adults;
ultrasound for pregnant women, children, & young people).
NSAID by any route should be offered for pain relief. If NSAIDs are contraindicated or ineffective, intravenous
paracetamol should be offered. If both NSAIDs & IV paracetamol are contraindicated or ineffective, or IV paracetamol
is unavailable, an opioid analgesic should be considered.
Management of urinary stones depends on factors such as size of stone, severity of symptoms, location of stone (renal or
ureteric), and patient’s age. Options include watchful waiting, medical expulsive therapy, & surgical treatment.
[Reference – NICE Clinical Knowledge Summaries – Renal or ureteric colic – acute]

Other options:

Phaeochromocytoma
A tumour arising from catecholamine-producing chromaffin cells and is named after its characteristic 'dusky-coloured
tumour' appearance.
About 80-90% arise in the adrenal gland.
Remainder being extra-adrenal in origin and most commonly found in the head & neck.
Mostly develop sporadically & majority are benign.
But, up to 40% are a manifestation of a hereditary syndrome, such as multiple endocrine neoplasia type 2 or Von
Hippel-Lindau syndrome.
Presentation
Palpitations, diaphoresis, pallor & paroxysmal hypertension.
Symptoms are usually episodic & tend to progress as the tumour grows.
Management

https://www.mcqbank.co.uk/testhistory.php?view=1487811 97/120
04/09/2023, 11:44 mcqbank.co.uk/testhistory.php?view=1487811
Diagnosed by ↑ levels of urine & serum catecholamines, metanephrines, & normetanephrines.
Treatment includes
Medical therapy for hypertension (phenoxybenzamine, phentolamine, alpha-blockers).
Surgical excision of tumour (open or laparoscopic adrenalectomy).
Complications include hypertensive crisis, myocardial infarction, and hypotension.
[Reference – BMJ Best Practice – Phaeochromocytoma]

Fibromyalgia
Chronic pain syndrome diagnosed by the presence of widespread body pain.
Original American College of Rheumatology criteria for the classification of fibromyalgia required that an individual had
both:
Widespread pain (front & back, right & left, both sides of the diaphragm) for ≥3m.
Tenderness (digital palpation - approximate force of 4 kg) of ≥11 out of 18 designated tender point sites:

Latest criteria do not require a tender point count, and focus more on identifying widespread pain in combination
with fatigue, memory & sleep difficulties.
Presentation
Chronic, widespread body pain and almost always have accompanying comorbid symptoms such as fatigue,
memory, sleep & mood difficulties.
Physical examination is typically normal but there is often diffuse tenderness, which may be assessed by counting the
number of tender points.
Management
Routine blood testing can help to exclude other differential diagnoses: e.g. ESR, TFTs, antinuclear antibodies.
However, be careful not to over-investigate.
Many therapies have been shown to be beneficial.
Non-pharmacological therapies include patient education, exercise, & cognitive behavioural
approaches.
Pharmacological therapies include tricyclic antidepressants, gabapentinoids, & serotonin-
noradrenaline (norepinephrine) reuptake inhibitors.
Patients often fare better when several different types of treatment are used together.
Even though practitioners may have a problem recognising fibromyalgia as a discrete disorder, they should
understand the diagnostic & therapeutic importance of 'centralisation' of pain as exemplified by a typical
fibromyalgia patient.
When a patient with any chronic pain state develops evidence of centralisation of pain, it is likely that
treatments that work well for acute pain or pain primarily due to nociceptive input (NSAIDs, opioids,
anti-inflammatories, immunosuppressives, injections, surgical procedures) will be less effective.
[Reference – BMJ Best Practice – Fibromyalgia]

Influenza
Acute respiratory tract infection typically caused by seasonal influenza A or B virus.
Typically presents in winter season.
Can occur in local community outbreaks, epidemics, &, rarely, pandemics.
Virus is transmitted by inhalation of infected respiratory secretions that have been aerosolised through coughing, sneezing,
or talking.
Presentation
Influenza infection is characterised by upper & lower respiratory tract symptoms of rhinorrhoea, cough,
fever, chills, headache, & myalgia.
Management
General measures
In otherwise healthy individuals with uncomplicated illness, self-management is recommended, including
resting at home, increased fluid intake, analgesics & antipyretics.
Note - aspirin should be avoided in children <16y due to risk of Reye's syndrome.
Pharmacological
Antiviral drugs are usually not prescribed to otherwise healthy individuals.

https://www.mcqbank.co.uk/testhistory.php?view=1487811 98/120
04/09/2023, 11:44 mcqbank.co.uk/testhistory.php?view=1487811
Oseltamivir and zanamivir reduce replication of influenza A and B viruses by inhibiting viral neuraminidase.
They are most effective if started within a few hours of the onset of symptoms.
Annual vaccination for prevention is available for at risk groups.
[Reference - Patient.co.uk Professional Reference – Influenza]

Primary aldosteronism
In PA, aldosterone production exceeds the body's requirements and is relatively autonomous with regard to its
normal chronic regulator, the renin-angiotensin II system.
This results in excessive sodium reabsorption via amiloride-sensitive epithelial sodium channels within the distal
nephron, leading to hypertension & suppression of renin-angiotensin II.
Urinary loss of potassium & hydrogen ions, exchanged for sodium at the distal nephron, may result in
hypokalaemia & metabolic alkalosis if severe & prolonged.
Presentation
Classic features include:
Hypertension.
Hypokalaemia (note 70% of patients may be normokalaemic).
Sodium may be normal or at the high end of normal.
Metabolic alkalosis.
Management
Optimal detection involves screening all hypertensive patients using the plasma aldosterone/renin ratio, after
controlling for factors (including medicines) that may confound results.
In patients with repeatedly ↑ aldosterone/renin ratios, definitive confirmation or exclusion of diagnosis involves
careful suppression testing with measurement of aldosterone response to fludrocortisone or to salt loading.
Subtype differentiation for optimal treatment involves genetic testing for the hybrid gene causing familial
hyperaldosteronism type I (glucocorticoid-remediable aldosteronism). A negative genetic test should be followed by
adrenal CT and adrenal venous sampling to differentiate unilateral from bilateral forms.
Most common specifically treatable & potentially curable form of hypertension, accounting for at least 5%
of hypertensive patients, with most patients being normokalaemic.
30% have unilateral forms correctable by unilateral laparoscopic adrenalectomy
70% have bilateral forms in which hypertension responds well to aldosterone antagonists.
[Reference - BMJ Best Practice – Primary aldosteronism]

Polycystic kidney disease


PKD is characterised by renal cysts and numerous systemic & extrarenal manifestations. There are 2 types: the
commoner autosomal-dominant PKD (ADPKD) and autosomal-recessive PKD (ARPKD).
Presentation
Most individuals present with complications of the disease. However, increasing numbers are being detected by
screening individuals who have an affected relative.
Complications associated with renal disease
Impaired urine concentrating capacity is a common early presentation with problems associated with excessive
water & salt loss such as nocturia.
Loin pain is the most common symptom (60%) – can be caused by renal haemorrhage, stones & UTIs.
Hypertension is a common presenting feature (50% of adults). Hypertension is associated with left
ventricular hypertrophy.
Bilateral kidney enlargement - abdominal examination may reveal enlarged & palpable kidneys - if other
organs are cystic there may be palpable hepatomegaly & even splenomegaly.
Gross haematuria following trauma is a classic presenting feature of ADPKD (30-50%).
UTI & pyelonephritis may be presenting features.
Renal stones are 2x as common as in the general population. Uric acid stones are more common than
calcium oxalate stones.
Kidney failure - usually in the 4th to 6th decade of life.
Extra renal complications - extrarenal cysts, intracranial aneurysms & dolichoectasias (elongated and
distended arteries), aortic root dilation & aneurysms, mitral valve prolapse, and abdominal wall hernias.
Management
Treatment is currently supportive only. No treatments have, as yet, been shown to halt or even slow disease
progression.
Long-term complications include hypertension, ↑ cardiovascular morbidity & mortality, chronic renal failure,
ruptured intracranial aneurysm, and end-stage renal disease (ESRD).
[References – BMJ Best Practice - Polycystic kidney disease & Patient.co.uk Professional Reference - Autosomal Dominant
Polycystic Kidney Disease]

Send us your feedback about this MCQ | Add question feedback to My Revision Notes | Rate this MCQ:

https://www.mcqbank.co.uk/testhistory.php?view=1487811 99/120
04/09/2023, 11:44 mcqbank.co.uk/testhistory.php?view=1487811

Single Best Answer Question

Maximum Mark: 1
Mark Awarded: 1
Time taken to answer: 1 minutes 22 seconds.
Main Question

A 34-year-old man presents with a painful, bruised deformed penis. He gives a history of recent painful sexual intercourse. On
examination, his penis is swollen and bruised with the appearance of an “eggplant”. There is also blood present at the meatus.
Which of the following is the MOST likely diagnosis?
Answers

Your answer:
A. Penile fracture

Correct Answer:
Your answer is correct!

All Answers:
A. Penile fracture
B. Penile compartment syndrome
C. Priapism
D. Torn frenulum
E. Penile haematoma

Feedback

Penile fracture is the traumatic rupture of the corpus cavernosum and is considered a urologic emergency.

Uncommon (30% occur during sexual intercourse).


Sudden blunt trauma or lateral bending of the erect penis can break the thinned & stiff tunica albuginea.
One or both corpora may be involved & injury to the penile urethra may occur (more common when both corpora
cavernosa are injured).
Presentation
Patients describe a snapping sound with immediate detumescence.
Pain varies depending on the severity of injury.
On examination, the normal external penile appearance is completely obliterated because of significant penile
deformity, swelling, and ecchymosis (the so-called "eggplant" deformity).

If the urethra has also been injured, blood is present at the meatus and patients report haematuria,
dysuria or experience acute urinary retention.
Retention may be secondary to the urethral injury or periurethral hematoma that is causing a bladder
outlet obstruction.
Urinary extravasation may be a late complication of unrecognized urethral injury.
Management
Successful voiding does not exclude urethral injury - therefore, retrograde urethrography is required whenever
urethral injury is suspected.
Penile fracture can usually be diagnosed clinically, however, in equivocal cases, diagnostic cavernosography or MRI
should be performed.
Historically, conservative management was considered the treatment of choice for penile fractures but this has fallen
into disfavour because of high complication rates.
Principles of surgical therapy include evacuating the haematoma, correcting the defect in the tunica
albuginea & repairing the urethral injury.

Send us your feedback about this MCQ | Add question feedback to My Revision Notes | Rate this MCQ:

https://www.mcqbank.co.uk/testhistory.php?view=1487811 100/120
04/09/2023, 11:44 mcqbank.co.uk/testhistory.php?view=1487811

Single Best Answer Question

Maximum Mark: 1
Mark Awarded: 0
Time taken to answer: 0 minutes 23 seconds.
Main Question

Which one of the following is LEAST likely to occur in adult polycystic kidney disease?
Answers

Your answer:
E. Renal cell carcinoma

Correct Answer:
A. Aortic valve stenosis

All Answers:
A. Aortic valve stenosis
B. Mitral valve prolapse
C. Intracranial berry aneurysms
D. Increased frequency of colonic diverticula
E. Renal cell carcinoma

Feedback

Autosomal dominant polycystic kidney disease (ADPKD) is the most frequent genetic cause of renal failure in adults.

Approximately 90% have an abnormality on chromosome 16.


ADPKD is a multisystemic and progressive disorder with formation of cysts in the kidney and other organs (e.g. liver,
pancreas, spleen)
Hepatic cysts may occur in 75% of patients >60y.
They appear to be affected by female steroid hormones, and, although they may occur in both sexes, massive
cysts occur almost exclusively in women.
Non-cystic manifestations include an increased incidence of mitral valve prolapse, intracranial berry aneurysms (in up
to 40% of cases) and an increased frequency of colonic diverticula & renal cell carcinoma.
Presentation
Clinical features usually begin in the 3rd - 4th decade of life, but cysts may be detectable in childhood.
Patients usually present with
haematuria - from cyst rupture
flank pain - from cyst enlargement
hypertension
stroke - from subarachnoid haemorrhage from intracranial aneurysms
renal failure
palpable masses in the flanks
increased incidence of UTIs
Investigations
Urinalysis - check for infection, protein (microalbuminuria occurs in about a third but heavy proteinuria is rare)
and haematuria.
Bloods - FBC (polycystic kidneys can produce excess erythropoietin and hence raise Hb), U&E
Ultrasound can detect renal cysts from 1-1.5 cm in diameter.
Sensitivity is 100% over the age of 20 but false negatives can occur below this age.
It is also possible to scan other organs like the liver or pancreas for cysts.
CT is more sensitive in that it can detect cysts down to 0.5 cm diameter but the dose of radiation is quite high.
MRI is an equally sensitive alternative.
Gene testing can be done by linkage or sequence analysis:
Management
Diagnosis of ADPKD has implications both for the patient & their family.
Treatment is currently supportive only.
No treatments have, as yet, been shown to halt or even slow disease progression.

Send us your feedback about this MCQ | Add question feedback to My Revision Notes | Rate this MCQ:

https://www.mcqbank.co.uk/testhistory.php?view=1487811 101/120
04/09/2023, 11:44 mcqbank.co.uk/testhistory.php?view=1487811

Single Best Answer Question

Maximum Mark: 1
Mark Awarded: 0
Time taken to answer: 0 minutes 57 seconds.
Main Question

Which one of the following is NOT a clinical feature of genitourinary TB?


Answers

Your answer:
B. Haematuria

Correct Answer:
D. Normal fertility

All Answers:
A. Sterile pyuria
B. Haematuria
C. Painful testicular swelling
D. Normal fertility
E. Frequency & dysuria

Feedback

Genitourinary tuberculosis may involve the kidneys, ureter, bladder, or genital organs. The genitourinary system is a common
site of extrapulmonary TB.

Clinical symptoms usually develop 10-15 years after the primary infection.
Only about a quarter of patients with genitourinary tract TB have a known history of TB - about half of
these patients have normal chest radiography findings.
Commoner in males aged 30-45yrs.
Presentation
The presentation is often vague, and doctors must have a high index of suspicion to make the diagnosis.
GUTB often manifests as repeated urinary tract infections that do not respond to the usual antibiotics.
The most common symptoms of GUTB, in descending order of frequency, is
↑ frequency of urination
dysuria
suprapubic pain
blood or pus in the urine (hallmark of genitourinary tract TB is sterile pyuria)
fever
There may also be a painful testicular swelling, perianal sinus, or a genital ulcer.
Fertility maybe diminished & may be a cause of unexplained infertility.
Management
The pharmacological therapy of patients with genitourinary tract TB is analogous to lung TB.

Send us your feedback about this MCQ | Add question feedback to My Revision Notes | Rate this MCQ:

Single Best Answer Question

Maximum Mark: 1
Mark Awarded: 1
Time taken to answer: 2 minutes 1 seconds.
Main Question

A 40-year-old woman presents with sudden onset, left sided, colicky pain and nausea for the last 6 hours. On examination she is
writhing in agony. She is apyrexial and there is left loin tenderness. Urinalysis reveals a microscopic haematuria. Which of the
following is the most likely diagnosis?
Answers

Your answer:
C. Renal colic

Correct Answer:
Your answer is correct!

All Answers:
A. Mittelschmerz
B. Constipation

https://www.mcqbank.co.uk/testhistory.php?view=1487811 102/120
04/09/2023, 11:44 mcqbank.co.uk/testhistory.php?view=1487811
C. Renal colic
D. Acute pyelonephritis
E. Hydronephrosis

Feedback

A renal stone or calculus is a solid aggregation formed in the kidneys from dietary minerals in the urine.

Demographics
Annual incidence is 1:1000 & the average lifetime risk is 10%
Male to female ratio is 3:1
The peak age is between 30 - 50.
Pathology
Consist of crystal aggregates that form in the collecting ducts which can then be deposited anywhere from the renal
pelvis to the urethra.
calcium oxalate (75%)
magnesium ammonium phosphate [struvite] (10%)
urate (5%)
hydroxyapatite (5%)
cystine (1%)
Risk factors
Anatomical anomalies in the kidneys &/or urinary tract,
e.g. horseshoe kidney, ureteral stricture
Family history of renal stones
Hypertension
Gout
Hyperparathyroidism
Immobilisation
Relative dehydration
commoner in hot climates
Metabolic disorders
which increase excretion of solutes
e.g. chronic metabolic acidosis, hypercalciuria, hyperuricosuria
Deficiency of citrate in the urine
Cystinuria (an autosomal recessive aminoaciduria)
Drugs, especially thiazide diuretics
Presentation
Many stones are asymptomatic and discovered during investigations for other conditions.
Classic symptom is sudden, severe pain.
Pain starts in the loin about the level of the costovertebral angle & moves to the groin, with tenderness of the
loin or renal angle, sometimes with haematuria.
If the stone is high & distends the renal capsule then pain will be in the flank but as it moves down pain will
move anteriorly and down towards the groin.
Other symptoms which may be present include:
Rigors and fever
Dysuria
Haematuria
Urinary retention
Nausea and vomiting
Investigations
Urinalysis
The stone often causes some bleeding into the renal tract and this may produce a positive result for blood on
stick testing
a negative test does not exclude the diagnosis
Mid-stream specimen of urine for microscopy, culture & sensitivities.
Bloods
FBC, U&E, calcium & phosphate and urate.
Blood cultures & parathyroid hormone if indicated.
Encourage the patient to try to catch the stone for analysis.
This may mean urinating through a tea strainer, filter paper such as a coffee filter or a gauze.
Helical CT or CT KUB is the gold standard for the investigation of urinary stones (99% of stones visible) & has
superseded intravenous urography (IVU).
Plain X-rays (80% of stones are visible on a KUB plain film).
Renal ultrasound may also detect stones & reveal any evidence of obstruction (hydronephrosis or hydroureter)
Management

https://www.mcqbank.co.uk/testhistory.php?view=1487811 103/120
04/09/2023, 11:44 mcqbank.co.uk/testhistory.php?view=1487811
Most kidney stones do not require surgery and will pass on their own.
Surgery is necessary when the pain is persistent & severe, in renal failure and when there is a kidney
infection.
It may also be advisable if the stone fails to pass or move after 30 days.
Finding a significant stone before it passes into the ureter allows physicians to fragment it surgically before it causes
any severe problems.
In most of these cases, non-invasive extracorporeal shock wave lithotripsy (ESWL) will be used.
Otherwise some form of invasive procedure is required.
With approaches including
Ureteroscopic fragmentation (or simple basket extraction if feasible) using laser
Ultrasonic or mechanical (pneumatic, shock-wave) forms of energy to fragment the larger stones.
Percutaneous nephrolithotomy (PCNL) or rarely open surgery may ultimately be necessary for large or complicated
stones or stones which fail other less invasive attempts at treatment.
Prevention
A normal calcium intake is now recommended as low calcium diets increase oxalate excretion.
Advise to consume less oxalate by reducing tea, chocolate, nuts, strawberries, rhubarb, spinach, beans & beetroot &
to drink plenty of fluids especially in the summer.
Complications
Complete blockage of the urinary flow from a kidney decreases glomerular filtration rate (GFR) and, if it persists
for >48 hours, may cause irreversible renal damage.
If ureteric stones cause symptoms after 4 weeks, there is a 20% risk of complications, including deterioration of
renal function, sepsis, & ureteric stricture.
Infection can be life-threatening.
Persisting obstruction predisposes to pyelonephritis.

Send us your feedback about this MCQ | Add question feedback to My Revision Notes | Rate this MCQ:

Single Best Answer Question

Maximum Mark: 1
Mark Awarded: 0
Time taken to answer: 0 minutes 58 seconds.
Main Question

Which one of the following statements about phimosis is CORRECT?


Answers

Your answer:
C. Parents can be reassured that physiological phimosis will not affect intercourse in adulthood.

Correct Answer:
E. Inability to clean under the foreskin is associated with the development of cancer of the penis in adulthood.

All Answers:
A. Phimosis affects 90% of boys in childhood.
B. Circumcision is the treatment of choice for physiological or primary phimosis.
C. Parents can be reassured that physiological phimosis will not affect intercourse in adulthood.
D. Topical steroids are not an option.
E. Inability to clean under the foreskin is associated with the development of cancer of the penis in adulthood.

Feedback

Phimosis is the inability to retract the foreskin because of a narrow preputial ring.

It affects about 1% of boys.


Inability to clean under the foreskin is associated with stones in the preputial sac and the development of cancer of the
penis.
Causes
Primary (physiological) - without scarring
congenital - rare
Secondary (pathological) - due to scarring from conditions such as
recurrent balanitis
traumatic retraction of the foreskin
Balanitis xerotica et obliterans
Presentation

https://www.mcqbank.co.uk/testhistory.php?view=1487811 104/120
04/09/2023, 11:44 mcqbank.co.uk/testhistory.php?view=1487811
Poor stream
Ballooning of foreskin on micturition
'Spraying' on micturition
Recurrent attacks of balanitis
Pain on intercourse in adults
And in severe cases - hydronephrosis
Treatment
Differentiating between physiologic & pathologic phimosis is important as the former is managed conservatively and
the latter requires surgical intervention.
Physiological phimosis
1. Since the foreskin becomes retractable with time these patients can be managed conservatively.
2. Topical steroids can be applied to the preputial ring may be useful.
Pathological phimosis
1. Circumcision.
2. A short course of topical corticosteroids may be beneficial in mild scarring.

Send us your feedback about this MCQ | Add question feedback to My Revision Notes | Rate this MCQ:

Single Best Answer Question

Maximum Mark: 1
Mark Awarded: 1
Time taken to answer: 1 minutes 0 seconds.
Main Question

An 80-year-old man presents with visible haematuria and also a “dragging feeling” in his scrotum. On examination, there is a
palpable mass in his left flank and a left sided varicocele. Which SINGLE ONE of the following is the MOST likely diagnosis?
Answers

Your answer:
D. Renal cancer

Correct Answer:
Your answer is correct!

All Answers:
A. Bladder cancer
B. Testicular cancer
C. Prostate cancer
D. Renal cancer
E. Spermatic cord tumour

Feedback

This is a question that featured in a previous actual exam.

This patient most likely has a renal cancer given the presentation of haematuria, left flank mass and left varicocele.

Renal cell carcinoma (RCC)

Most common tumour of the kidney in adults, accounting for >80% of renal neoplasms.
In children, Wilms' tumour is most common.
Benign tumours of the kidney are rare.
RCC originates from the proximal renal tubular epithelium.
Renal cancers account for ~2-3% of all malignancies, with highest incidence in Western countries.
Men are 1.5x more likely to be affected than women.
Incidence rises after 40y & is highest in 60-70 year olds.
Several risk factors have been identified, particularly smoking & obesity.
~2-3% of RCCs are hereditary & several autosomal dominant syndromes are described, the most common being
von Hippel Lindau disease.
The classic triad of haematuria, loin pain & loin mass is not often seen as the use of ultrasound & CT scans has
increased detection of asymptomatic RCC.

https://www.mcqbank.co.uk/testhistory.php?view=1487811 105/120
04/09/2023, 11:44 mcqbank.co.uk/testhistory.php?view=1487811
Other presentations include fatigue, weight loss, pyrexia of unknown origin (paraneoplastic manifestation),
hypertension, varicocele & leg oedema.
The left testicular vein drains first into the left renal vein & then to the inferior vena cava (IVC) while the right
testicular vein drains directly into the IVC just below the renal vein.

A left-sided renal cancer can block the renal vein due to external compression by a large tumour or internal
obstruction by advancing tumour cells. Further tumour extension into the IVC may also cause IVC obstruction.
~30% of patients present with metastatic disease – e.g. haemoptysis, bone pain or pathological fracture.
Lungs = most common site of metastases where the classical picture of cannon ball secondaries is almost
diagnostic. It is also one of the carcinomas to metastasise to bone where it produces osteolytic lesions.

Bladder cancer
7th most common cancer in the UK.
Majority occur in patients >60y. Men outnumber women by 3:1.
Risk factors include increasing age, smoking (tobacco smoke contains aromatic amines & polycyclic aromatic
hydrocarbons, which are renally excreted) and occupational exposure to aromatic amines, polycyclic aromatic
hydrocarbons & chlorinated hydrocarbons – mainly paint, dye, metal & petroleum industries.
Squamous cell tumours usually follow chronic inflammation from stones, indwelling catheters or schistosomiasis
(common cause in Middle East).
• Presenting feature is painless haematuria that is visible in ~85%. Painless haematuria must be treated as
malignancy of the urinary tract until proved otherwise.

Testicular cancer
95% arise from germ cells (the sperm producing cells) which can be subdivided into seminoma & non-seminomatous germ
cell tumours.
Testicular cancer is relatively rare but is the most common malignancy in men aged between 20-35y.
Risk factors include cryptorchidism or testicular maldescent, Klinefelter's syndrome, family history, male infertility
(increases risk by 3x); infantile hernia; & testicular microlithiasis (small intratesticular calcifications seen on
ultrasound).
>95% present with a lump in the testis, which is usually painless. But there can be testicular pain &/or abdominal
pain or a dragging sensation. Gynaecomastia may occur from beta-hCG production.
Since the introduction of combination chemotherapy in the 1970s, survival rates have risen every year to cure rates of
>95%.

Prostate cancer
Most common cancer in men & makes up ~25% of all male cancer diagnoses in the UK.
Three well-established risk factors are increasing age, ethnic origin & genetic predisposition.
Increasing age is most important risk factor but ~25% occur in men <65y.
Higher incidence & mortality in black African-Caribbean men.
The risk is at least doubled if one first-degree relative has prostate cancer.
Prostate cancer is usually suspected on DRE &/or PSA levels. Lower urinary tract symptoms (LUTS) do not particularly
raise suspicion of prostate cancer because LUTS are common in older men & are rarely the presenting symptom of prostate
cancer. However, locally advanced tumours may cause obstructive LUTS.

Primary spermatic cord tumours


Rare but they are the most common tumours of the paratesticular region.
Generally present as asymptomatic, slow growing, firm, palpable paratesticular masses.
Although, most are benign, comprised primarily of lipomas, ~25% are potentially life-threatening malignant tumours such
as sarcomas.

References
Patient.co.uk
BMJ Best Practice

Send us your feedback about this MCQ | Add question feedback to My Revision Notes | Rate this MCQ:

https://www.mcqbank.co.uk/testhistory.php?view=1487811 106/120
04/09/2023, 11:44 mcqbank.co.uk/testhistory.php?view=1487811

Single Best Answer Question

Maximum Mark: 1
Mark Awarded: 1
Time taken to answer: 29 minutes 36 seconds.
Main Question

A 27-year-old, healthy, sexually active woman presents with pain on urination and recent onset of urinary frequency and urgency.
She has no costovertebral angle tenderness on examination. Urine dipstick is positive for nitrite and leukocytes. Which is the
SINGLE MOST likely diagnosis?
Answers

Your answer:
E. Infectious cystitis

Correct Answer:
Your answer is correct!

All Answers:
A. Bacterial vaginosis
B. Acute pyelonephritis
C. Overactive bladder
D. Candidal vaginitis
E. Infectious cystitis

Feedback

This patient most likely has a lower urinary tract infection (UTI). UTI should be suspected when a patient complains of dysuria,
frequency, urgency, changes in urine appearance or consistency, nocturia, and suprapubic discomfort/tenderness. A lower UTI is
an infection of the bladder (cystitis) usually, whereas an upper UTI is infection of the ureters and kidneys (pyelonephritis).
Lower UTIs do not have the systemic features (fevers, chills, nausea, vomiting) or the loin pain typically associated with upper
UTIs.

Bacterial vaginosis (BV) is caused by an overgrowth of predominantly anaerobic organisms in the vagina. The most common
organisms include Gardnerella vaginalis, Prevotella spp., Mycoplasma hominis, and Mobiluncus spp. They replace lactobacilli,
which are the dominant bacteria present in the normal vagina. The pH increases from less than 4.5 to as high as 6. BV is not
thought to be sexually transmitted (it can occur in virgins); however, sexual activity has been linked to development of the
infection. It presents as vaginal discharge and/or vaginal irritation. In fact, BV is the most common cause of abnormal vaginal
discharge in women of reproductive age. Urine dipstick and urine culture are negative.

Overactive bladder is characterised by urgency, often with frequency and nocturia and sometimes leakage (urge incontinence).
It is often but not always associated with detrusor muscle overactivity. Urine dipstick and urine culture are negative.

Candidal vaginitis is an infection of the lower female reproductive tract due to Candida albicans. Peak incidence age is 20-40
years with 70% of women report having had candidal vulvovaginitis at some point in their lifetime. Symptoms include pruritus
vulvae, white, 'cheesy' discharge (which is non-offensive; foul-smelling or purulent discharge suggests bacterial infection),
dyspareunia and dysuria. Urine dipstick and urine culture are negative.

[Reference – BMJ Best Practice; Patient.co.uk Professional Reference]

Urinary tract infection (lower)

Background
Lower urinary tract infection (UTI) = infection of the bladder.
Cystitis is often used as a synonym for lower UTI (particularly for women), although technically it means 'bladder
inflammation' and there may be non-infectious causes such as radiation & chemical-induced cystitis.
Upper UTI is infection of the upper urinary tract i.e. ureters & kidneys (pyelonephritis).
UTI is very common in women & accounts ~2% of all GP consultations.
50% of women will be treated for a symptomatic UTI during their lifetime.
UTI is not common in men, but incidence is higher in elderly men (likely to have additional risk factors) or
with indwelling urinary catheters.
Usually caused by gut bacteria from GI tract.
Escherichia coli accounts for 80% & Staphylococcus saprophyticus 10% of UTIs. Proteus mirabilis is more common
in men or associated with renal tract abnormalities, particularly calculi.

Definitions

Significant bacteriuria = 105 colony-forming units per ml (CFU/mL).


Asymptomatic bacteriuria = significant bacteriuria without clinical infection.
NICE Antenatal Care guidelines state that asymptomatic bacteriuria should be screened for & treated in
pregnancy as associated with premature delivery & low birthweight.
https://www.mcqbank.co.uk/testhistory.php?view=1487811 107/120
04/09/2023, 11:44 mcqbank.co.uk/testhistory.php?view=1487811
Do not treat asymptomatic bacteriuria in patients with indwelling catheters.
Uncomplicated UTI = typical pathogens with normal urinary tract & kidney function, and no predisposing co-morbidities.
Complicated UTI = increased likelihood of complications such as persistent infection, treatment failure & recurrent
infection. Assoc with ≥1 risk factors:
Abnormal urinary tract (e.g. calculus, vesicoureteric reflux, neurogenic bladder, indwelling catheter, urinary
obstruction, recent instrumentation).
Virulent organism (e.g. Staphylococcus aureus).
Immunosuppression (e.g. poorly controlled diabetes).
Impaired renal function.
Recurrent UTI = Defined as ≥2 UTI in 6mths or ≥3 UTI in 12mths. Can be:
Relapse - same organism
Reinfection - different organism

Diagnosis
Typical features of UTI = dysuria, frequency, urgency, changes in urine appearance or
consistency, nocturia, suprapubic discomfort/tenderness.
Note typical features may be absent in the elderly with underlying cognitive impairment — can present
with delirium, lethargy, ↓ ability to carry out activities of daily living & anorexia.
Women
If uncomplicated UTI in <65y.o, urine dipstick can be used as a diagnostic aid — dipstick is unreliable in women
aged >65y & those who are catheterised.
If dipstick +ve for nitrite or leukocyte & RBC ⟹ UTI is likely.
Send urine for culture if previous antibiotic treatment has failed or possibility of antibiotic resistance.
If dipstick -ve for nitrite & +ve for leukocyte ⟹ UTI is possible.
Send urine for culture to confirm diagnosis.
If dipstick -ve for nitrite, leukocyte & RBC ⟹ UTI is unlikely.
No need to send urine culture — consider other diagnoses.
Send urine for culture if:
Pregnant.
>65y.
Persistent symptoms that do not resolve with antibiotics.
Recurrent UTI.
Urinary catheter in situ or recently catheterised.
Risk factors for resistance or complicated UTI such as GU tract abnormalities, renal impairment,
residence in a long-term care facility, hospitalisation for >7d in last 6m, recent travel to a country with ↑
resistance or previous resistant UTI.
Atypical symptoms.
Visible or non-visible haematuria.
Men
Confirm diagnosis with urine culture before starting empirical drug treatment.
If catheterised - only send urine sample if features of systemic infection.
Do not use urine dipstick or microscopy to diagnose UTI in men:
Who are not catheterized — dipsticks are poor at ruling out infection.
But, may be helpful in some situations, e.g. UTI is unlikely if mild or non-specific symptoms AND -
ve urine dipstick test (i.e. both nitrite & leukocytes -ve).
Note, presence of these markers does not rule in UTI, although +ve nitrite makes UTI more
likely (PPV 96%).
With an indwelling catheter — working diagnosis should be based on clinical judgement.
Aged >65y — dipsticks become more unreliable with increasing age >65y.

Differential diagnosis
Pyelonephritis - urinary symptoms associated with fever &/or loin pain.
Drug-induced cystitis - cyclophosphamide, allopurinol, danazol, or tiaprofenic acid.
Sexually transmitted Infections e.g. chlamydia trachomatis (pyuria without bacteriuria).
Dermatological conditions e.g. psoriasis, dermatitis, lichen sclerosis or lichen planus.
Spondyloarthropathies e.g. reactive arthritis or Bechet’s syndrome.
Malignancy.
Ovarian cancer may present with persistent urinary urgency &/or frequency.
Bladder, or renal cancer may present with haematuria (visible or non-visible).
In women – consider
Urethral syndrome (aka painful bladder syndrome, interstitial cystitis & trigonitis = symptoms of cystitis in
absence of UTI - symptoms relieved by voiding & aggravated by drinking alcohol or caffeinated drinks).
Atrophic vaginitis/urethritis (menopausal woman with vaginal discharge or itch, & pain during sexual
intercourse).
In men – consider

https://www.mcqbank.co.uk/testhistory.php?view=1487811 108/120
04/09/2023, 11:44 mcqbank.co.uk/testhistory.php?view=1487811
Acute prostatitis (fever, irritative urinary symptoms, perineal/suprapubic pain, pain on ejaculation or with bowel
movements, & exquisitely tender prostate on PR).
Epididymitis (scrotal pain & epididymis is oedematous & tender).

Empirical Treatment
Women
If mild symptoms with normal immunity, renal function & renal tract, treatment can be delayed if she wishes
to see if symptoms resolve, especially if -ve urine dip (nitrites & leucocytes) indicating low probability of UTI.
For all other women especially if pregnant prescribe treatment without delay.
1st Choice
Nitrofurantoin for 3d.
If eGFR ≥45.
c/i in G6PD deficiency & acute porphyria.
Trimethoprim for 3d.
BNF advises caution if predisposed to folate deficiency.
Renal impairment:
If eGFR 15–30 - use half normal dose after 3d.
If eGFR <15 - use half normal dose.
c/i in blood dyscrasias.
2nd Choice (if no improvement in symptoms when 1st-choice antibiotic is taken for at least 48h or if first-
choice is unsuitable).
Nitrofurantoin for 3d (if not used as 1st choice).
Pivmecillinam for 3d.
Fosfomycin single dose sachet.
Pregnant
Nitrofurantoin (avoid at term due to risk of neonatal haemolysis) for 7d.
2nd choice
Amoxicillin (only if culture results available & susceptible) for 7d.
Cefalexin for 7d.
BNF states that manufacturer advises avoiding trimethoprim during pregnancy. There is a
teratogenic risk in first trimester (folate antagonist).
Men
Start empirical antibiotic drug treatment with trimethoprim or nitrofurantoin for 7d.

Duration
3d course for women with:
Uncomplicated UTI.
UTI with haematuria.
Recurrent UTI.
5–10d course for women who have:
Abnormal urinary tract.
Immunosuppression.
Impaired renal function.
7d course if:
Pregnant.
Catheterised.
Change catheter before starting antibiotics if catheter has been insitu for >7d.
Consider withholding antibiotics until culture results available to guide treatment if mild symptoms with normal
immunity & renal function.
Male.

References
NICE Clinical Knowledge Summaries - Urinary tract infection (lower) – women
NICE Clinical Knowledge Summaries - Urinary tract infection (lower) – men
British National Formulary

Send us your feedback about this MCQ | Add question feedback to My Revision Notes | Rate this MCQ:

Single Best Answer Question

Maximum Mark: 1

https://www.mcqbank.co.uk/testhistory.php?view=1487811 109/120
04/09/2023, 11:44 mcqbank.co.uk/testhistory.php?view=1487811
Mark Awarded: 0
Time taken to answer: 0 minutes 13 seconds.
Main Question

Which ONE of the following is the commonest type of renal stone?


Answers

Your answer:
A. Calcium phosphate

Correct Answer:
C. Calcium oxalate

All Answers:
A. Calcium phosphate
B. Cystine
C. Calcium oxalate
D. Urate
E. Magnesium ammonium phosphate (struvite)

Feedback

A renal stone or calculus is a solid aggregation formed in the kidneys from dietary minerals in the urine.

Demographics
Annual incidence is 1:1000 & the average lifetime risk is 10%
Male to female ratio is 3:1
The peak age is between 30 - 50.
Pathology
Consist of crystal aggregates that form in the collecting ducts which can then be deposited anywhere from the renal
pelvis to the urethra.
calcium oxalate (75%)
magnesium ammonium phosphate [struvite] (10%)
urate (5%)
hydroxyapatite (5%)
cystine (1%)
Risk factors
Anatomical anomalies in the kidneys &/or urinary tract,
e.g. horseshoe kidney, ureteral stricture
Family history of renal stones
Hypertension
Gout
Hyperparathyroidism
Immobilisation
Relative dehydration
commoner in hot climates
Metabolic disorders
which increase excretion of solutes
e.g. chronic metabolic acidosis, hypercalciuria, hyperuricosuria
Deficiency of citrate in the urine
Cystinuria (an autosomal recessive aminoaciduria)
Drugs, especially thiazide diuretics
Presentation
Many stones are asymptomatic and discovered during investigations for other conditions.
Classic symptom is sudden, severe pain.
Pain starts in the loin about the level of the costovertebral angle & moves to the groin, with tenderness of the
loin or renal angle, sometimes with haematuria.
If the stone is high & distends the renal capsule then pain will be in the flank but as it moves down pain will
move anteriorly and down towards the groin.
Other symptoms which may be present include:
Rigors and fever
Dysuria
Haematuria
Urinary retention
Nausea and vomiting
Investigations
Urinalysis
The stone often causes some bleeding into the renal tract and this may produce a positive result for blood on
stick testing

https://www.mcqbank.co.uk/testhistory.php?view=1487811 110/120
04/09/2023, 11:44 mcqbank.co.uk/testhistory.php?view=1487811
a negative test does not exclude the diagnosis
Mid-stream specimen of urine for microscopy, culture & sensitivities.
Bloods
FBC, U&E, calcium & phosphate and urate.
Blood cultures & parathyroid hormone if indicated.
Encourage the patient to try to catch the stone for analysis.
This may mean urinating through a tea strainer, filter paper such as a coffee filter or a gauze.
Helical CT or CT KUB is the gold standard for the investigation of urinary stones (99% of stones visible) & has
superseded intravenous urography (IVU).
Plain X-rays (80% of stones are visible on a KUB plain film).
Renal ultrasound may also detect stones & reveal any evidence of obstruction (hydronephrosis or hydroureter)
Management
Most kidney stones do not require surgery and will pass on their own.
Surgery is necessary when the pain is persistent & severe, in renal failure and when there is a kidney
infection.
It may also be advisable if the stone fails to pass or move after 30 days.
Finding a significant stone before it passes into the ureter allows physicians to fragment it surgically before it causes
any severe problems.
In most of these cases, non-invasive extracorporeal shock wave lithotripsy (ESWL) will be used.
Otherwise some form of invasive procedure is required.
With approaches including
Ureteroscopic fragmentation (or simple basket extraction if feasible) using laser
Ultrasonic or mechanical (pneumatic, shock-wave) forms of energy to fragment the larger stones.
Percutaneous nephrolithotomy (PCNL) or rarely open surgery may ultimately be necessary for large or complicated
stones or stones which fail other less invasive attempts at treatment.
Prevention
A normal calcium intake is now recommended as low calcium diets increase oxalate excretion.
Advise to consume less oxalate by reducing tea, chocolate, nuts, strawberries, rhubarb, spinach, beans & beetroot &
to drink plenty of fluids especially in the summer.
Complications
Complete blockage of the urinary flow from a kidney decreases glomerular filtration rate (GFR) and, if it persists
for >48 hours, may cause irreversible renal damage.
If ureteric stones cause symptoms after 4 weeks, there is a 20% risk of complications, including deterioration of
renal function, sepsis, & ureteric stricture.
Infection can be life-threatening.
Persisting obstruction predisposes to pyelonephritis.

Send us your feedback about this MCQ | Add question feedback to My Revision Notes | Rate this MCQ:

Single Best Answer Question

Maximum Mark: 1
Mark Awarded: 0
Time taken to answer: 0 minutes 18 seconds.
Main Question

According to NICE chronic kidney disease guidance which ONE of the following should be used to classify chronic kidney disease?
Answers

Your answer:
C. Estimated glomerular filtration rate (GFR)

Correct Answer:
D. Estimated glomerular filtration rate (GFR) & albumin to creatinine ratio (ACR)

All Answers:
A. Urea
B. Creatinine
C. Estimated glomerular filtration rate (GFR)
D. Estimated glomerular filtration rate (GFR) & albumin to creatinine ratio (ACR)
E. Renal biopsy

Feedback

https://www.mcqbank.co.uk/testhistory.php?view=1487811 111/120
04/09/2023, 11:44 mcqbank.co.uk/testhistory.php?view=1487811

Chronic kidney disease (CKD)

Defined as abnormalities of kidney function or structure present for >3 months, with implications for health.

This includes all people with markers of kidney damage and those with a GFR<60 ml/min/1.73m2 on ≥2
occasions separated by a period of ≥90 days.
Common causes of chronic kidney disease
diabetic nephropathy
chronic glomerulonephritis
chronic pyelonephritis
hypertension
adult polycystic kidney disease
Revised classification of CKD
NICE in 2014 advised that CKD should be classified according to estimated GFR (eGFR) & albumin:creatinine
ratio (ACR), using 'G' to denote the GFR category (G1–G5, which have the same GFR thresholds as the previous
CKD stages 1–5) and 'A' for the ACR category (A1–A3).

A person with an eGFR of 25ml/min/1.73m2 and an ACR of 15mg/mmol has CKD G4A2.

A person with an eGFR of 50ml/min/1.73m2 and an ACR of 35mg/mmol has CKD G3aA3.

GFR eGFR
Terms
category (ml/min/1.73m2)
G1 >90 Normal or high
Mildly decreased (relative to young
G2 60–89
adult level)
G3a 45–59 Mildly to moderately decreased
G3b 30–44 Moderately to severely decreased
G4 15–29 Severely decreased
G5 <15 Kidney failure
ACR
ACR (mg/mmol) Terms
category
A1 <3 Normal to mildly increased
Moderately increased (relative to
A2 3–30
young adult level)
Severely increased (Including
A3 >30 nephrotic syndrome where ACR usually
>220mg/mmol)
The following are risk factors for the progression of CKD:
Hypertension
Cardiovascular disease
Smoking
Chronic use of NSAIDs
Proteinuria
Diabetes
Black/Asian ethnicity
Urinary outflow obstruction
General Management
Avoidance of nephrotoxins, e.g. IV radiocontrast agents, NSAIDs, aminoglycosides.
Cardiovascular prophylaxis
For patients with 10-year risk of cardiovascular disease >20%, consider aspirin treatment (if blood pressure
<150/90 mm Hg) and lipid-lowering drug therapy.
Blood pressure monitoring: blood pressure should be measured at least annually.
Control of hypertension
Hypertension should be tightly controlled
The majority of patients with CKD will require more than two drugs to treat hypertension.
ACE inhibitors are first line and are particularly helpful in proteinuric renal disease (e.g. diabetic
nephropathy).
As these drugs tend to reduce filtration pressure a small fall in GFR and rise in creatinine
can be expected.
Furosemide is useful as an anti-hypertensive in patients with CKD, particularly when the GFR falls
to below 30-45 ml/min as it has the added benefit of lowering serum potassium.
T2DM
Add SGLT2 inhibitor (e.g. dapagliflozin) for T2DM patients who have albuminuria (ACR >30mg/mmol) despite
taking max doses of ARB or ACE inhibitor.

Reference
NICE Clinical Knowledge Summaries - Chronic kidney disease

https://www.mcqbank.co.uk/testhistory.php?view=1487811 112/120
04/09/2023, 11:44 mcqbank.co.uk/testhistory.php?view=1487811

Send us your feedback about this MCQ | Add question feedback to My Revision Notes | Rate this MCQ:

Single Best Answer Question

Maximum Mark: 1
Mark Awarded: 1
Time taken to answer: 11 minutes 55 seconds.
Main Question

A 50-year-old man undergoes a CT abdominal scan with contrast. Following the procedure, his renal function has deteriorated.
Which of the following is the MOST likely diagnosis?
Answers

Your answer:
A. Acute tubular necrosis

Correct Answer:
Your answer is correct!

All Answers:
A. Acute tubular necrosis
B. Renal artery stenosis
C. Nephrotic syndrome
D. Renal tract obstruction
E. Rapidly progressive glomerulonephritis

Feedback

Most acute renal failure (or now known as acute kidney injury) is caused by acute tubular necrosis which is characterised by
acute loss of renal excretory function.

Urinary sodium is classically >30 mmol/l, urea <150 mmol/l and urine plasma osmolality ratio is <1:1.
Interpretation of the above values is invalidated if loop diuretics or mannitol have been given.
Recovery is expected over days or weeks.
It is not much affected by therapeutic intervention as the kidney probably recovers by cellular regeneration, which
management is unable to speed up - hence only supportive therapy.
Classification
Ischaemic
Mainly affects the distal convoluted tubules & loop of Henle.
Due to "shock" commonly hypovolaemic, cardiogenic, or septic.
Pigment-induced ATN is a special form associated with massive haemoglobinuria (massive haemolysis) or
myoglobinuria (rhabdomyolysis).
Nephrotoxic
Mainly affects the proximal convoluted tubules.
Many agents are implicated, including
antibiotics - gentamicin
anaesthetics - methoxyflurane
heavy metals - mercury, lead, arsenic
chemotherapeutic agents - cyclosporine
organic solvents - methyl alcohol, carbon tetrachloride
poison - pesticides
radiological contrast

Send us your feedback about this MCQ | Add question feedback to My Revision Notes | Rate this MCQ:

Single Best Answer Question

Maximum Mark: 1
Mark Awarded: 1

https://www.mcqbank.co.uk/testhistory.php?view=1487811 113/120
04/09/2023, 11:44 mcqbank.co.uk/testhistory.php?view=1487811
Time taken to answer: 4 minutes 30 seconds.
Main Question

A 30-year-old man usually fit and well presents with fever, back pain and urinary symptoms. On examination: T38ºC, HR95,
tender in the left flank, with urine dip positive for both nitrite and leucocyte esterase. Which is the SINGLE MOST likely diagnosis?
Answers

Your answer:
A. Acute pyelonephritis

Correct Answer:
Your answer is correct!

All Answers:
A. Acute pyelonephritis
B. Cystitis
C. Acute prostatitis
D. Lower lobe pneumonia
E. Urethritis

Feedback

The most likely diagnosis here is acute pyelonephritis based on the combination of urinary tract infection symptoms (e.g.
dysuria, frequency, urgency) with signs of pyelonephritis (e.g. fever, nausea, vomiting, or flank pain).

Cystitis or lower urinary tract infection does not present with systemic features (such as fevers, chills, nausea, vomiting) or back
pain.

Acute prostatitis is a painful inflammation within the prostate that is usually accompanied by evidence of bacterial infection.
Commonly caused by Escherichia coli bacteria. May present with: Sudden onset of fever, low back, suprapubic, perineal, or rectal
pain, symptoms of urinary tract infection (dysuria, frequency, urgency, or retention). Digital rectal examination reveals a tender,
often enlarged prostate.

Urethritis is usually a sexually transmitted disease that classically presents as acute urethral discharge following unprotected
sex. The two most important aetiological agents are Neisseria gonorrhoeae and Chlamydia trachomatis.

Patients with lower lobe pneumonia often complain of cough and pleuritic chest pain. Physical examination may show
decreased breath sounds, rales, or rhonchi.

[Reference – BMJ Best Practice]

Acute Pyelonephritis (Upper Urinary Tract Infection)

Background
= infection of renal pelvis ± infection of parenchyma.
Usually caused by bacteria ascending from lower urinary tract. Most common causative pathogens are gram-negative
bacteria:
Escherichia coli (70%)
Klebsiella species (20%)
Proteus mirablis
Pseudomonas species
Enterobacter species

Clinical features
Flank/renal angle pain &/or tenderness.
Myalgia.
Flu-like symptoms.
Rigors or ↑ temperature of ≥37.9°C (or <36°C in people aged >65y).
Most people have fever, although may be absent early in people with early or mild cases, frail, older people, or in
immunocompromised.
Nausea/vomiting.

Diagnosis
In all patients suspected of acute pyelonephritis, arrange collection of urine sample to determine infecting micro-
organism.
Dipstick testing of urine for nitrites & leukocytes is not necessary, however may be useful adjunct to guide diagnosis. It
should not be used in people:
With indwelling catheter — make working diagnosis based on clinical judgement.
Aged >65 years — dipsticks become more unreliable with increasing age.
There are no clinical features or routine investigations that conclusively distinguish acute pyelonephritis from
lower urinary tract infection.

https://www.mcqbank.co.uk/testhistory.php?view=1487811 114/120
04/09/2023, 11:44 mcqbank.co.uk/testhistory.php?view=1487811
However, the triad of flank pain (typically unilateral), fever, and nausea & vomiting occurs much more
often with pyelonephritis than with lower UTI.
Definitive diagnosis is made in those with loin pain &/or fever if BOTH of the following:
UTI confirmed on culture.
Other causes excluded e.g. pelvic inflammatory disease, appendicitis or renal calculi, etc.

Management in primary care


Admit to hospital if severe acute pyelonephritis or more serious condition (e.g. sepsis).
Consider same-day referral or seeking specialist advice:
Significantly dehydrated or unable to take oral fluids/medicines.
Pregnant.
Higher risk of developing complications — e.g. abnormality of genitourinary tract or underlying disease
(e.g. diabetes or immunosuppression).
Recurrent episodes of UTI (i.e. ≥2 episodes in 6-month period).
Offer an antibiotic for those who do not require admission i.e. low risk of complications.
Obtain urine sample for culture before starting empirical antibiotics & adjust when sensitivities are back
For non-pregnant women, men, & those with indwelling catheters, the following are first line:
Cefalexin for 7-10d.
Co-amoxiclav (if in line with sensitivity results) for 7-10d.
Trimethoprim (if in line with sensitivity results) for 14d.
Ciprofloxacin (consider safety issues) for 7d.
For pregnant women who do not require admission:
Cefalexin for 7-10d.
Indwelling urinary catheters:
Check catheter is draining correctly.
Consider removing or changing catheter ASAP if in situ for >7d.
Do not allow catheter removal or change to delay antibiotic treatment.
Do not give antibiotic prophylaxis for catheter changes unless history of symptomatic UTIs due to catheter
change.
Advise
Seek medical help if symptoms worsen, do not start to improve within 48h of taking antibiotic, or if becomes very
unwell.
Use paracetamol or ibuprofen for pain if required.
Drink sufficient fluids to avoid dehydration.
Review culture & sensitivity results when available, and change antibiotic if indicated.
Consider referral for investigation of underlying renal tract abnormality:
Men, following single episode without obvious cause.
Women with recurrent pyelonephritis.

Complications
Renal impairment/failure - due to scarring of renal parenchyma.
Septicaemia
Preterm labour

Reference
NICE Clinical Knowledge Summaries – Acute Pyelonephritis

Send us your feedback about this MCQ | Add question feedback to My Revision Notes | Rate this MCQ:

Single Best Answer Question

Maximum Mark: 1
Mark Awarded: 1
Time taken to answer: 0 minutes 18 seconds.
Main Question

A 30-year-old woman presents with recurrent headaches, pain in her flanks and haematuria. On examination, her BP is 180/100
and there are palpable masses in both her flanks. Which of the following is the MOST likely diagnosis?
Answers

Your answer:
D. Polycystic kidney disease

https://www.mcqbank.co.uk/testhistory.php?view=1487811 115/120
04/09/2023, 11:44 mcqbank.co.uk/testhistory.php?view=1487811
Correct Answer:
Your answer is correct!

All Answers:
A. Bilateral hydronephrosis
B. Renal artery stenosis
C. Renal amyloidosis
D. Polycystic kidney disease
E. Renal cell carcinoma

Feedback

Autosomal dominant polycystic kidney disease (ADPKD) is the most frequent genetic cause of renal failure in adults.

Approximately 90% have an abnormality on chromosome 16.


ADPKD is a multisystemic and progressive disorder with formation of cysts in the kidney and other organs (e.g. liver,
pancreas, spleen)
Hepatic cysts may occur in 75% of patients >60y.
They appear to be affected by female steroid hormones, and, although they may occur in both sexes, massive
cysts occur almost exclusively in women.
Non-cystic manifestations include an increased incidence of mitral valve prolapse, intracranial berry aneurysms (in up
to 40% of cases) and an increased frequency of colonic diverticula & renal cell carcinoma.
Presentation
Clinical features usually begin in the 3rd - 4th decade of life, but cysts may be detectable in childhood.
Patients usually present with
haematuria - from cyst rupture
flank pain - from cyst enlargement
hypertension
stroke - from subarachnoid haemorrhage from intracranial aneurysms
renal failure
palpable masses in the flanks
increased incidence of UTIs
Investigations
Urinalysis - check for infection, protein (microalbuminuria occurs in about a third but heavy proteinuria is rare)
and haematuria.
Bloods - FBC (polycystic kidneys can produce excess erythropoietin and hence raise Hb), U&E
Ultrasound can detect renal cysts from 1-1.5 cm in diameter.
Sensitivity is 100% over the age of 20 but false negatives can occur below this age.
It is also possible to scan other organs like the liver or pancreas for cysts.
CT is more sensitive in that it can detect cysts down to 0.5 cm diameter but the dose of radiation is quite high.
MRI is an equally sensitive alternative.
Gene testing can be done by linkage or sequence analysis:
Management
Diagnosis of ADPKD has implications both for the patient & their family.
Treatment is currently supportive only.
No treatments have, as yet, been shown to halt or even slow disease progression.

Send us your feedback about this MCQ | Add question feedback to My Revision Notes | Rate this MCQ:

Extended Matching Question

Maximum Mark: 3
Mark Awarded: 2
Time taken to answer: 0 minutes 44 seconds.
Main Question

For each patient below presenting with haematuria, choose the most likely cause.
Select ONE option only from the list.
Each option may be selected once, more than once or not at all.
Answers

1. A 4-year-old boy who was seen by his GP a fortnight ago with a sore throat Your Answer:
now presents with periorbital oedema, microscopic haematuria and G. Post streptococcal glomerulonephritis
Correct Answer:

https://www.mcqbank.co.uk/testhistory.php?view=1487811 116/120
04/09/2023, 11:44 mcqbank.co.uk/testhistory.php?view=1487811
proteinuria. Antistreptolysin O titre is positive and serum complement C3 is Your answer is correct!
reduced.

All answers:
A. Urinary tract infection
B. Henoch-Schönlein purpura
C. Drug induced
D. Bladder cancer
E. Renal calculus
F. Goodpasture’s syndrome
G. Post streptococcal glomerulonephritis
H. Wilms' tumour

Your Answer:
B. Henoch-Schönlein purpura
Correct Answer:
Your answer is correct!

All answers:
2. An 8-year-old boy presents with a history of a buttock rash, knee pains and A. Urinary tract infection
visible haematuria for the last couple of weeks. B. Henoch-Schönlein purpura
C. Drug induced
D. Bladder cancer
E. Renal calculus
F. Goodpasture’s syndrome
G. Post streptococcal glomerulonephritis
H. Wilms' tumour

Your Answer:
F. Goodpasture’s syndrome
Correct Answer:
C. Drug induced

All answers:
3. A 30-year-old homeless man with recently diagnosed pulmonary A. Urinary tract infection
tuberculosis complains of “blood in his urine”. B. Henoch-Schönlein purpura
C. Drug induced
D. Bladder cancer
E. Renal calculus
F. Goodpasture’s syndrome
G. Post streptococcal glomerulonephritis
H. Wilms' tumour

Feedback

Q1 - G

Post streptococcal glomerulonephritis characteristically occurs 7-14 days after infection with group A beta haemolytic
streptococcus.
Accounts for 90% of acute glomerulonephritis.
Occurs mostly in the <5y.o age group
Presentation
Usually asymptomatic but may present with
smoky urine, sometimes described as coca-cola coloured by parents.
oedema on the face around the orbits and on the backs of the hands & feet.
severe cases may cause acute hypertension, with encephalopathy & seizures, or heart failure
Investigations
Urine sample:
dipstick reveals blood & protein (but not >2+).
microscopy reveals red cell casts, but no protein casts or hyaline casts.
Blood
biochemistry - urea ↑, sodium & bicarbonate ↓, potassium normal or ↑.
antistreptolysin O titre +ve
complement - ↓ C3
Management
Treatment is supportive
Weighing twice daily, vital signs four hourly, and strict monitoring of fluid input & output.
In the first day the patient should be given one third normal maintenance fluids - in subsequent days the
previous day's output & insensible losses are replaced.

https://www.mcqbank.co.uk/testhistory.php?view=1487811 117/120
04/09/2023, 11:44 mcqbank.co.uk/testhistory.php?view=1487811
The glomerular filtration rate usually returns to normal within 10 to 14 days.
Complications
Hypertension - which may result in encephalopathy
Pulmonary oedema
Acute renal failure
Prognosis
Good - 95% of patients recover completely.

Q2 - B

Henoch-Schönlein purpura (HSP) is a systemic vasculitis characterized by deposition of immune complexes containing
IgA in the skin and kidney.

It occurs mainly in young children aged 3-10 yrs.


Commoner in boys.
Clinical features of HSP include cutaneous purpura, arthritis, abdominal pain, GI bleeding, orchitis, and nephritis.
30% of children have a preceding upper respiratory tract infection.
Antistreptolysin O titres are raised in 20-50% of patients.

It is uncertain as to whether HSP needs treatment beyond controlling the symptoms and most do not receive therapy
because of the high spontaneous recovery rate.
Nephropathy is treated supportively.
Fluid and electrolyte balance should be monitored, salt intake should be restricted, and antihypertensives
should be prescribed when needed.
Various drugs (steroids, azathioprine, cyclophosphamide) and plasmapheresis have been used to prevent renal
disease from progressing - The results have been inconsistent.

Q3 - C

The anti-tuberculous drug, rifampicin can cause urine discoloration that can be easily confused with haematuria.

Other causes of discoloured urine:

Colour Pathological causes Food and drug causes


bile pigments levodopa
Brown
myoglobin metronidazole
nitrofurantoin
some antimalarial
agents
fava beans

bile pigments Cascara


Brownish-black
melanin levodopa
methaemoglobin methyldopa
senna

pseudomonal UTI amitriptyline


Green or blue
biliverdin indigo carmine
IV cimetidine
IV promethazine

https://www.mcqbank.co.uk/testhistory.php?view=1487811 118/120
04/09/2023, 11:44 mcqbank.co.uk/testhistory.php?view=1487811
methylthioninium
chloride
triamterene

bile pigments phenothiazines


Orange
phenazopyridine

haematuria beets
Red
haemoglobinuria blackberries
myoglobinuria rhubarb
porphyria phenolphthalein
rifampicin

concentrated urine carrots


Yellow (orange to gold in cascara
dehydration)

Urinary tract infection


UTI causing inflammation of the urinary tract is a very common cause of haematuria.
Urine should be re-tested after completing treatment with an appropriate antibiotic and if haematuria is
persistent, other possible underlying causes should be considered.

Bladder cancer
Over 90% of cancers of the urinary bladder are urothelial carcinoma (previously termed transitional cell carcinoma).
More than 90% of new cases occur in people ≥55 years of age.
Haematuria (both visible and non-visible) is the primary presenting symptom of bladder cancer and is present in ≥80% of
patients.
Visible painless haematuria, present throughout the entire urinary stream, is most common.
Episodes of haematuria are typically intermittent and therefore resolution should not be attributed to treatment with,
for example, antibiotics.
Cystoscopy and urinary cytology are key to making the diagnosis.
Low-grade tumours are papillary and easy to visualise, but often have negative cytology. High-grade tumours are
often flat or in situ and difficult to visualise, but typically have a positive cytology.
Treatment
Non-muscle-invasive tumours are most common and complete transurethral resection is the treatment of
choice for these tumours but recurrence is high.
Treatment of choice for carcinoma in situ is immunotherapy using tuberculosis vaccine BCG.
Muscle-invasive tumours are treated with neoadjuvant chemotherapy, cystoprostatectomy, and extended pelvic
lymphadenectomy.
For advanced disease, options include combination cisplatinum-based chemotherapy and newer immunotherapies.

Renal calculus
Nephrolithiasis refers to the presence of crystalline stones (calculi) within the urinary system.
It is a common condition with around a 10% lifetime risk.
Multiple risk factors include chronic dehydration, diet, obesity, positive family history, some drugs(e.g. antacids,
carbonic anhydrase inhibitors, sodium- and calcium-containing medications, vitamins C and D, and protease inhibitors), and
various metabolic abnormalities (e.g. hyperoxaluria).
Patients typically present with acute renal colic, although some patients are asymptomatic.
Microscopic haematuria is present on urinalysis in up to 90% of cases.
Rarely, macroscopic haematuria can be present.
Non-contrast CT scan of the abdomen/pelvis is the imaging modality with the highest sensitivity and
specificity to diagnosis stones. Plain x-ray (KUB) and renal ultrasound can be utilised for diagnosis in some cases, such as
a desire to reduce or eliminate radiation exposure.
24-hour urine tests are recommended for most stone formers to determine cause of stone formation and optimal
treatment to help prevent future stone episodes.
Treatment consists of both medical and surgical therapies.

Goodpasture’s syndrome
Also known as anti-glomerular basement membrane antibody (anti-GBM) disease and is defined by the presence of
autoantibodies to collagen that is found in the basement membranes of alveoli and glomeruli.
It presents with rapidly progressive renal dysfunction often combined with lung haemorrhage.
Simultaneous haemoptysis and renal failure (pulmonary renal syndrome) has many causes, but where the renal
disease is a rapidly progressive glomerulonephritis, about 15% of cases will be Goodpasture's disease.
Rapid diagnosis by early renal biopsy and serological testing is essential, because appropriate treatment given before
renal damage is advanced can result in an excellent prognosis.

https://www.mcqbank.co.uk/testhistory.php?view=1487811 119/120
04/09/2023, 11:44 mcqbank.co.uk/testhistory.php?view=1487811
Treatment is by removal of circulating antibodies using plasma exchange combined with agents to prevent their continued
production.

Wilms’ tumour
Nephroblastoma is the most common form of renal malignancy in childhood.
It usually occurs in the first 2 to 5 years of life.
Presents as a unilateral, painless, abdominal/flank mass; rarely presents bilaterally.
10-25% of patients have visible or non-visible haematuria, usually not associated with dysuria.
Metastatic disease occurs in <10% of patients and needs to be carefully excluded using ultrasound and CT/MRI.
Treatment is with nephrectomy, chemotherapy, and radiotherapy.
Long-term survival approaches 90% in localised disease.

Send us your feedback about this MCQ | Add question feedback to My Revision Notes | Rate this MCQ:

©2023 MCQBank Terms & Conditions | Privacy Policy | Share this page

https://www.mcqbank.co.uk/testhistory.php?view=1487811 120/120

You might also like